crim law

Pataasin ang iyong marka sa homework at exams ngayon gamit ang Quizwiz!

FILL IN THE BLANKS. First degree murder is a _________ crime; common-law murder is a _________ crime. A malice, general intent B malice, specific intent C specific intent, general intent D specific intent, malice

D

If the question does not say otherwise about governing law assume....

common law applies.

Which of the following statements regarding voluntary intoxication is FALSE? A Voluntary intoxication can be a defense to a specific-intent crime, if the intoxication prevents the formation of the required intent. B Voluntary intoxication cannot be a defense when the defendant becomes intoxicated for the purpose of establishing the defense of voluntary intoxication. C Voluntary intoxication is the negligent taking of a substance known to be intoxicating. D Voluntary intoxication is not a defense to strict-liability crimes.

C

Which one of the following crimes does NOT require specific intent? A First-degree murder B Inchoate crimes C Arson D Theft offenses

C

Three potential exceptions to rule that mistake of law is no defense -

1) Reliance on high-level government interpretations Example 31: If a regulation states that something is permitted and you are later prosecuted for that conduct because the regulation was wrong, the regulation might negate the mens rea element. Note 5: Relying on your own lawyer's advice does not generally fall into this exception. 2) Lack of notice 3) Mistake of law that goes to an element of specific intent (applies only to the "FIAT" crimes or specific-intent crimes) can be relevant to negating defense. For Fiat crimes, for specific intent crimes mistake of law going to element of specific intent might negate the defendants mens rea. Example 32: Quentin, a solo practitioner, practices law out of his rented apartment. He drills holes in the wall of his apartment for bookshelves. The landlord tells Quentin to stop drilling and grabs Quentin's drill. Quentin pushes the landlord, causing him to fall and injure his shoulder. Quentin is charged with battery and with the statutory crime of "knowingly damaging the property of another person, [with the intent to deprive that person of the property]." Quentin argues that he thought a tenant was allowed to alter the walls of his apartment. He argues that he was defending his own property when he pushed the landlord. Quentin had the mistaken belief that he was protecting his own property. Is that a defense to a charge of battery? Answer is no because battery is not a specific intent crime. As long as you voluntarily applied force to another person, and here quentin did that, he'll be guilty as long as he was not entitled to do so. Can Quentin's mistake about the property be a defense to violating the statute? Quentins mistake about property law can be a defense to violating a statute that prohibits knowingly damaging the property of another person with the intent to deprive that person of the property. Because quentin didn't knowingly deprive a person of the property since he believed he could alter the walls. For a specific intent crime, a fiat crime, a mistake about law that negates the thing to which the specific intent attaches, can be a defense. That's mistake of law - other than with regard to fiat crimes you wont see a successful mistake about law on bar exam.

Flavors of homicide (summary)

1. 1st degree murder which is a fiat crime. Important to remember fiat defenses. Rare to see first degree murder on multi state and usually if it comes up, almost certainly will be signaled it's a first degree murder case. 2. Common law murder - involves either intent to kill, intent to inflict serious bodily injury, a depraved heart, or felony murder in the commission of one of the serious felonies. 3. Manslaughter - voluntary and involuntary. Voluntary is extreme emotional distress or heat of passion. Involuntary involves less serious felony or misdemeanor and death of a person.

Flavors of homicide summary

1. 1st degree murder which is a fiat crime. Important to remember fiat defenses. Rare to see first degree murder on multi state and usually if it comes up, almost certainly will be signaled it's a first degree murder case. 2. Common law murder - involves either intent to kill, intent to inflict serious bodily injury, a depraved heart, or felony murder in the commission of one of the serious felonies. 3. Manslaughter - voluntary and involuntary. Voluntary is extreme emotional distress or heat of passion. Involuntary involves less serious felony or misdemeanor and death of a person.

Assault (Two Forms)

1. Attempted battery - could just have crime of attempted battery but that crime is often referred to as assault o If a defendant has taken a substantial step toward completing a battery but fails, he will be guilty of assault o It is a specific-intent crime because it is an attempt; specific-intent defenses are available. 2. Fear of Harm - other kind of assault o Intentionally placing another in fear of imminent bodily harm o This form of assault is a general-intent crime. Not specific intent crime.

arson

1. Definition o Common law arson requires: § malicious § Burning § Of another person's § dwelling 2. Elements o Malice: Intent to act in a way that will cause burning, or is substantially likely to do so o Burning: § Common law—there had to be burning (fire) as opposed to an explosion or smoke damage. It also required damage to the structure, not just the contents inside. § Modern statutes—it is arson even if there is no damage to the structure of the building or if the fire was caused by an explosion. o Another Person: § Common law—aat common law had to be burning house or dwelling of another rperson, you could not torch your own house. § Modern statutes—burning your own home is arson. Because of insurance fraud so don't want people torching their own property and getting insurance proceeds. o Dwelling: § Common law—had to be a dwelling, not another commercial structure § Modern statutes—burning down a commercial building is arson.

PROPERTY CRIMES - larceny

1. Definition o Larceny requires: § Taking; § Another person's property; § Without his consent (trespassory); and § With the intent to deprive him of it permanently.

Common-Law Murder

1. Definition o Unlawful killing of another human being committed with malice aforethought o Lawful killing of another is not murder (e.g., state execution or a police officer's justifiable use of deadly force). 2. Four Kinds of Malice: a. Intent to kill b. Intent to inflict serious bodily harm c. Someone who acts with abandoned or malignant heart or a depraved heart d. Felony murder

Robbery

1. Definition and Elements o Common law robbery is a simple equation: § Robbery = larceny + assault o Robbery requires (4 first elements are basically larceny elements and then add on 2 additional elements): 1) Taking; 2) Another person's property; 3) Without his consent; 4) With intent to deprive him of it permanently; and 5) The taking occurs from the victim's person or in his presence 6) Either by violence or putting the victim in fear of imminent physical harm Violence or physical harm or intimidation has to occur either before the taking, simultaneously with the taking, or immediately following the taking in order to retain the property. Level of physical force doesn't have to be huge - slght physical force is enough.

summary of negating mens rea

2 kinds of mistakes- mistakes of law - almost never a defense except for fiat crimes. Mistakes of fact - where must be reasonable as defendant for most crimes but can be unreasonable for fiat crimes. 2 kinds of intoxication - Involuntary intoxication (almost always will be a defense) vs voluntary intoxication - which again applies only to specific intent crimes and only if it prevents the defendant from forming the mens rea. 4 kinds of insanity - all tied to some form of mental disease or defect.

accomplices

C. Accomplices • Theory for holding people other than the principal responsible for the crime committed by the principal; same degree of responsibility as the principal - imp thing about accomplice liability is its not separate crime, but rather different theory about why this d is guilty of a particular crime - murder or arson or like so accomplice is charged with same crime, but just diff theory as to why person is guilty. • People who assist the principal either before or during the commission of a crime • for someone to be liable as accomplice, Must act with the intent of assisting the principal to commit the crime; bystanders, even approving ones, are not accomplices Example 22: You go to a bar after the exam and a bar fight breaks out. If you applaud during the fight, you are not liable for assault as an accomplice because you didnt act with intent of assisting principal, you just clapped. • Liable as an accomplice for both the planned crime and any other foreseeable crimes that occur in the course of the criminal act Example 23: Irving decides to rob a bank. He asks Oscar to drive a getaway car. Irving then goes to Paul's house and asks to borrow Paul's gun, telling him he wants to go target shooting. Paul lends him the gun. During the bank robbery, Irving shoots a teller. Oscar takes his share of money from the robbery and buys a pound of heroin. Irving, Oscar, and Paul are all charged with robbery, attempted murder, and possession of heroin with intent to distribute. For which crimes can they be convicted? Irving: irving is guilty of bank robberty - easy because he went into bank and attempted to rob it. Hes principal for that. Also guilty of attempted murder because he shot teller. Another crime hes principal for. But hes not guilty of possession of heroine with intent to distribute because that's not a foreseeable consequence of bank robbery - that was just Oscar doing osmehing on his own. Oscar: guilty of all 3 crimes - guilty of heroine crime as a principal because he bought heroine - actus reus - with intent to distribute, and guilty as an accomplice to attempted murder and bank robbery, even though he didn't commit the actus reus inside the bank and he didn't shoot teller, those were both foreseeable - he acted with intent that the bank be robbed and it was foreseeable that violence would occur in the bank. Paul: paul isn't liable for any crimes - he had no intent to assist in commission of any crimes. Note 4: Irving and Oscar might also be held liable for conspiracy (discussed below). In addition to crimes above, may also be conspiracy between Oscar and irving. • An accomplice can be criminally liable even if he or she cannot be a principal or even if the principal cannot be convicted as long as accomplice acted with intent of assisting in commission of the crime. Example 24: A statute makes it a crime for a public official to take bribes. Mayor Benedict wants to take a bribe, but wants to do it where no one will see. His friend, Claudio, owns a cabin in the woods. Mayor Benedict asks Claudio if he could use his cabin as the location to exchange bribery money. Claudio agrees and offers to chain off the road. Claudio could be guilty of aiding and abetting the bribery as an accomplice. But, he could not be guilty as a principal because he is not a public official. Example 25: You assist a six-year-old child to commit murder. Your little nephew tells you that he really wants to kill his classmate Bobby. You give your nephew a gun and he shoots Bobby. Your nephew cannot be convicted of murder because he is under the age of seven, but you can be convicted of aiding and abetting as an accomplice. Example 26: If you help a diplomat commit a crime, the diplomat has diplomatic immunity, so the diplomat might not be held liable as a principal, but you can still be held liable as an accomplice. • Exception: A person protected by a statute itself cannot be an accomplice in violating the statute. Example 27: If a statute prohibits sex with an underage person, the underage person is not an accomplice to that crime. So Accomplice liability is about a theory of liability for someone who aids before or during the commission of a crime. But what about people who assist after the fact? Theyre not guilty of underlying crime - for them separate set of crimes like obstruction of justice or harboring fugitive. People who assist d after crime committed generally not guilty of primary crime but can be punished for one of the secondary crimes.

manslaughter

1. Definitions o All unlawful killings of another human being that are not first-degree murder or common law murder o Two types: voluntary manslaughter and involuntary manslaughter 2. Voluntary Manslaughter o Occurs when a defendant intends to kill the victim, but his state of mind is less blameworthy than murder o Acted in the "heat of passion" or "under extreme emotional disturbance" o Test: Is the situation one in which most people would act without thinking and without time to cool off? Example 59: Rufus comes home and finds his wife, Amy, in bed with Luke. Rufus shoots Luke and Amy. This is acting in the heat of passion or heat of the moment. People are not expected to stop, think rationally, and calm down in this situation. Note 9: Hearing about your spouse's affair is not sufficient. Heat of moment is only really if its in the actual heat of the moment. Example 60: Frank was sexually abused as a young teenager. Five years later, he testifies at the highly-publicized trial of his abuser and was forced to describe the abuse he had suffered. Several days later, a group of young teenagers surrounded Frank and began taunting him, suggesting that he had "asked for it." Frank lashes out at the group and strangles one of them. Would Frank be entitled to a manslaughter instruction? Probably not under the heat of passion defense, but maybe under the emotional disturbance defense - acting under extreme emotional disturbance. Need more facts to determine whether it was reasonable to strangle somebody for a taunt. So voluntary manslaughter - defendant actually intends to cause death but is acting under circumstances where we don't think theyre sufficiently blameworthy to find them guilty of murder. 3. Involuntary Manslaughter - someone dies but death not intended by the defendant. o A criminally negligent killing or killing of someone while committing a crime other than those covered by felony murder or "bark" crimes (i.e., misdemeanor manslaughter) o A defendant who engages in criminally negligent conduct and causes a death is guilty of involuntary manslaughter (e.g., traffic deaths). Example 61: Lisa goes jogging with her dog. A city law makes it a misdemeanor to let a dog off its leash on public property. Lisa ignores the law. While unleashed, Lisa's dog attacks and kills a small child. In this situation, Lisa may be guilty of misdemeanor manslaughter, because letting her dog off its leash was the "but-for" cause of the child's death and it was crime to let dog off the leash.

Actus Reus

1. No such thing as a "thought" crime. Wanting or hoping to commit a crime is not itself a crime. Must be some physical act involved with crime. o Act can be speech 2. Act must be voluntary (i.e., willed by the defendant) a. cant be an involuntary act like when d is asleep or cant be involuntary act as in something not under ds mode of control - that does not satisfy the actus reus requirement b. "Voluntary" does not necessarily mean the person wanted to do it. It means that he had motor control over the act. Example 6: Tom goes into a store in an attempt to rob the store. He points a pistol at Jerry and says, "Tie up your coworkers or else I'll blow your head off." Jerry, in tying up his coworkers, has committed the actus reus necessary for battery, but he almost certainly will have a defense of duress. Notice not that he didn't commit actus reus - someone who steals food to save starving children. They may have not wanted to do it but they did commit actus reus. So doesn't have to be voluntary in sense that u really wanted to do it. 3. The failure to act can be sufficient actus reus a. Failure to comply with a statutory duty can be sufficient for actus reus § E.g., the failure to file a tax return, failure to register for selective service, etc. - those can be actus reus for crimes as well as affirmatively doing something. b. sometimes actus reus can come from special relationship between defendant and victim; § E.g., parents' failure to take care of their children c. or someone who has a special relationship because they Voluntarily undertook a relationship and then allowed harm to happen to person whos victim of the crime - voluntarily assuming a duty of care that is cast aside d. The defendant causes a danger and fails to mitigate harm to the victim caused by the peril

Mens Rea - specific intent (first state of mind under common law)

1. Specific Intent - first state of mind under common law. o Defendant not only committed the actus reus but did it for the very purpose of causing the result that the law criminalizes Example 10: Burglary is defined as the entering of a dwelling with the intent to commit a felony once inside (crime that has intent or specific intent requirement - tells u what intent u need to have is - intent to commit specific felony once inside). You have a suspicion that your neighbor might be a hoarder. One day, you climb into your neighbor's house through a window to look around. Once inside, you happen to see a newspaper from the day that you were born. You take the newspaper and leave the house. Have you committed burglary? Although u actually have committed crime of larceny, u haven't commited burglary. Because u didn't enter house with intnt of commiting burglary - when u went in u were just curious about knowing what might be inside. So although you committed crime, you didn't commit common law burglary - because that is a specific intnt crime - can only commit if at time you entered house you intended to commit felony - it could be larceny, arson, assault, but you didn't have any intent to commit crime when u entered house other than trespass really.

Example 73: Olivia and Paul decide to rob a grocery store. Alex, the clerk, manages to trip the alarm and Olivia grabs Scott and puts him into the meat locker. A hostage situation ensues, and police eventually get Olivia to release Scott. Now Olivia and Paul are charged with kidnapping Alex and Scott. Are they guilty of kidnapping Scott? Are they guilty of kidnapping Alex?

1. Yes. Because he was moved - they pushed him into the locker and he was confined unlawfully without his consent. 2. If Alex wasn't moved at all, they didn't commit kidnapping simply by not letting him leave the store because they never moved him and they never hid him.

states of mind under common law

1. specific intent 2. actual malice 3. general intent 4. strict liability

Summary of defenses

2 kinds of mistakes- mistakes of law - almost never a defense except for fiat crimes. Mistakes of fact - where must be reasonable as defendant for most crimes but can be unreasonable for fiat crimes. 2 kinds of intoxication - Involuntary intoxication (almost always will be a defense) vs voluntary intoxication - which again applies only to specific intent crimes and only if it prevents the defendant from forming the mens rea. 4 kinds of insanity - all tied to some form of mental disease or defect.

Which of the following is a defense to a strict liability crime? A The defendant did not know that she was violating the law. B The defendant did not intend to violate the law. C The defendant's act was involuntary. D A reasonable person would not have known that he was violating the law.

C

Homicide -causation

2. Causation o There must be a causal relationship between the defendant's actions and what happened to the victim o Actual Causation - but for causatin: Victim would not have died "but for" what the defendant did o Proximate Causation: Defendant's act is a foreseeable cause of the victim's death (death is the natural and probable result of the conduct) Example 52: A defendant shoots the victim, and the victim dies in the hospital because the doctor was negligent. That death is a foreseeable consequence of shooting someone. So, the defendant's act was both the actual and proximate cause of the victim's death. § Independent actions by a third person are generally not a foreseeable cause Example 53: Bernie, the defendant, commits securities fraud. One of his victims—who was defrauded of money—commits suicide as a result. Bernie is not the proximate cause of the victim's death, even though he may be viewed as the actual cause. But for bernie stealing money person wouldn't have killed himself but too much intervening action to say it's a homicide. Exam Tip 7: Consent is not a defense to any type of homicide. Assisted suicide is a homicide by the assister, except in jurisdictions that permit assisted suicide. 3 big kinds of homicide: first degree murder, common law murder, and manslaughter

larceny

2. Elements Property—tangible personal property (e.g., wristwatch, goods from a store) § Not intangible property (e.g., copyright), real property, or services Note 10: There are modern "theft of services" statutes that criminalize obtaining a service, e.g., a massage, and then not paying for it. That conduct does not fall under common law larceny because haven't stolen someones property but rather their services. Taking—involves any movement of property, however slight Example 62: Hank goes into an electronics store to shoplift an MP3 player. He lifts the player off of the counter and heads for the door. Even before he leaves the store, he has satisfied the "taking" element of larceny - it just involves moving property, however slightly. Trespassory taking (without consent) - 2 ways of taking something without consent - 1. Not asking them and 2. Tricking them. § Consent must be real for something not to be a larceny, not obtained by trick (discussed below). § as long as the prosecutor shows the taking was wrongful, the defendant bears the burden of proving that there was consent Intent—an intent to deprive the person of the property permanently § "Borrowing" property, even without the owner's consent, is not larceny, as long as you intend to give the property back. Might be crime but not common law larceny. § If the property is destroyed in your care when you borrowed it, you have not committed larceny as long as you intend to give the property back. Example 63: A teenager sees a fancy car with the keys in it. The teen jumps in, thinking he will just drive the car around the block a few times and then return it. But he crashes the car. Is he guilty of larceny? Answer is no because teenager intended to return car to parking lot. (Note: he may be guilty of other crimes, like joyriding. But just not common law larceny) Example 64: One day before class, you realize that you forgot your textbook. In the law school library, you see someone else's textbook lying on a desk. You borrow the book with the intention of returning it after class. During your class, there is a fire alarm and you leave the book behind. The sprinklers malfunction and start spraying water inside the classroom, ruining the borrowed textbook. Are you guilty of larceny? No. you didn't intend to keep the textbook permanently. o Larceny is a specific-intent crime. T category of fiat - theft crime. § As long as the defendant thinks it's his property—however unreasonably—he is not guilty of larceny. For specific intent crimes, mistake of fact whether reasonable or unreasonable can negate the intent. Example 65: You take the wrong black umbrella from an umbrella stand in a restaurant. Even if you intend to keep that umbrella forever, you are not guilty of larceny because you were mistaken (this is true even if the mistake was unreasonable). So long as you thought it was ur umbrella not larceny, even if unreasonable for you to think that.

scope of conspiracy

2. Scope of a Conspiracy o At common law, each co-conspirator can be convicted both of: 1) Conspiracy; and 2) All substantive crimes committed by any other conspirator acting in furtherance of the conspiracy. Not just crime u agreed to commit, but all other crimes committed in furtherance of the crime. Example 40: Goldie, Frank, and Myrtle all agree to steal goods from a sporting goods store in the mall. Goldie will enter the store to steal the goods, Frank is the lookout, and Myrtle is driving the getaway car. Goldie tells Frank to blow a whistle if a security guard comes by. After Goldie enters the store, Frank panics when he sees a guard and shoves the guard, who falls and breaks his arm. Goldie comes out of the store with the goods, including some starter pistols. Myrtle later uses the starter pistol to rob a convenience store. Who is liable for what? Conspiracy to rob the store: goldie, frank, and myrtle all liable Battery of the guard: goldie, frank, and myrtle all liable - even though they didn't agree to hit guard because the crime was in furtherance of the conspiracy which was getting away from the store with the robbery goods. Robbery of the convenience store: only myrtle is liable because that wasn't within scope of conspiracy. Conspiracy about robbing store and getting a bunch of stuff - not about what they did with the stuff afterwards. o Relationships of co-conspirators § Chain Conspiracy: Co-conspirators are engaged in an enterprise consisting of many steps; each participant is liable for the substantive crimes of his co-conspirators Example 41: A conspiracy to distribute drugs involves many people in a causal chain (manufacturer—distributer—dealer). The conspirators need not know each other, but they have all agreed to participate in the same conspiracy and if so, each can be held liable (1) for the conspiracy and (2) for the substantive offenses committed along the way. § Spoke-Hub Conspiracy: Involves many people dealing with a central hub; participants are not liable for the substantive crimes of their co-conspirators because each spoke in the wheel is treated as a separate agreement rather than one larger general agreement Example 42: A bank employee agrees to process fraudulent loan applications. This employee would serve as the "hub." Each individual seeking a fraudulent loan is a "spoke." But the spokes are not generally responsible for the actions of the other spokes, unless they have much broader agreement amongst themselves. Example 43: A pawn shop operates as a "fence" to sell stolen goods. Each robber is responsible for conspiracy to sell stolen property, but not for the goods from other robbers - those are different spokes attached to the hub.

3 big kinds of homicide

3 big kinds of homicide: first degree murder, common law murder, and manslaughter

general intent

3. General Intent o Catch-all category o The intent to perform an act, and the act is unlawful o The defendant does not need to know that the act is unlawful; it is sufficient to intend to perform the act that the law condemns o Generally, acts done knowingly, recklessly, or negligently under the Model Penal Code (MPC) are general-intent crimes. § Examples include: battery, kidnapping, rape, and false imprisonment. Exam Tip 3: General intent crimes most likely to be tested on the MBE include manslaughter and battery.

actus reus - failure to act

3. The failure to act can be sufficient actus reus o Failure to comply with a statutory duty can be sufficient for actus reus; § E.g., the failure to file a tax return, failure to register for selective service, etc. - those can be actus reus for crimes as well as affirmatively doing something. o sometimes actus reus can come from special relationship between defendant and victim; § E.g., parents' failure to take care of their children o or someone who has a special relationship because they Voluntarily undertook a relationship and then allowed harm to happen to person whos victim of the crime - voluntarily assuming a duty of care that is cast aside Example 7: Anna and her friends are on the beach when they see you drowning. Anna shouts, "I'll save you!" She starts swimming out to you when she looks at her watch and realizes that it's time for her favorite TV show. She turns around and swims back to shore. If you drown, can Anna be held criminally liable? Yes because she started rescuing you and then abandoned you. o The defendant causes a danger and fails to mitigate harm to the victim caused by the peril Example 8: On the way into the bar exam, you see a student spill his drink all over himself. You notice that he is about to touch a live wire. If you do nothing and let the student get electrocuted, you cannot be held criminally liable because you didn't create the peril. However, if it were your wire and you should have grounded it, you could be held criminally liable because theres sufficient actus reus. Example 9: Nancy is a home healthcare worker who cares for an elderly woman named Mona. One evening, while Nancy's friend Dave is visiting Nancy at Mona's house, Mona begins to choke on her dinner. Dave hears Mona choking. Nancy is in the kitchen listening to her iPod. Dave says, "Hey, that old lady is turning blue and gasping." Nancy decides to wait until the end of a track to check on Mona. Mona chokes to death. Can Dave be charged with any crime related to Mona's death? Dave cant be charged with any crime related to monas death because hes just a callous by stander - and callous by standers not generally guilty of any crime. Can Nancy? But Nancy can be charged because she had a special relationship to mona - she was her caregiver and failed to give her the appropriate amount of care.

Which of the following is a requirement for conspiracy under many modern conspiracy statutes but not under the common law? A An agreement B An unlawful purpose C Performance of an overt act D The involvement of two or more people

C

withdrawal from a conspiracy

3. Withdrawal from a Conspiracy o Common law—it's impossible to withdraw from a conspiracy, because the crime is completed the moment the agreement is made. Example 44: You and Professor Kramer agree to commit a crime. The next day, she calls and says she no longer wants to participate in the crime. Professor Kramer can still be convicted of conspiracy because you both agreed to commit the crime and that was moment when crime occurred.. But if you go ahead and commit the crime without Professor Kramer, she will not be convicted of the actual crime because she has withdrawn. Shes guilty of conspiracy but not of substantive offenses. o Federal and MPC—a conspirator can withdraw prior to the commission of any overt act by communicating her intention to withdraw to all other conspirators or by informing law enforcement. But must be done before the overt act. § After an overt act—a conspirator can withdraw only by helping to thwart the success of a conspiracy. Example 45: In an MPC jurisdiction, if Professor Kramer withdraws from the conspiracy after an overt act—either by notifying you or law enforcement—she may not be held liable for conspiracy if the notification was timely enough to thwart the success of the conspiracy. o Even if a defendant cannot withdraw from the conspiracy (e.g., because a conspiracy had already been formed), the defendant can limit his liability for substantive crimes by informing the other conspirators of withdrawal or timely advising legal authorities.

strict liability

4. Strict Liability o There is no state of mind requirement or mens rea; the defendant must merely have committed the act. Kinds of things u find are strict liability crimes fall into 2 big categories - statutory or regulatory offenses. 1) Statutory/Regulatory offenses Example 12: A statute requires that food items be labeled with the expiration date. If a company sells those food items without an expiration date, they've committed a crime under this statute. Even if the company did not intend to sell the food without an expiration date or even if it was an honest mistake, the company is liable under strict liability. 2) Morals offenses Example 13: Humbert is attracted to younger women. He knows the age of consent in his state is 16. He meets Lolita in a bar, where patrons must show ID to enter. Lolita tells Humbert that she is 18 and she suggests they go back to her apartment to have sex. Humbert agrees and they have sex. Later, Humbert is charged with statutory rape, because Lolita is under the age of consent. At trial, Humbert states that he saw her ID and really thought that Lolita was 18. Suppose the judge believes Humbert, can he still be convicted of statutory rape? Answer is yes. Because that's a strict liability offense - only question is did he intend to have sex, and was she under the age of consent? And if those 2 questions answered yes, it doesnt matter what his intent was. Exam Tip 4: If an MBE question contains a statute, read it carefully for mens rea language. "With intent to..." = specific intent crime "Knowingly or recklessly...." = general intent crime No mens rea language = Consider if likely to be strict liability crime

FILL IN THE BLANKS. A ________ crime occurs when the defendant acted in a _________ disregard of a high risk of harm. A malice; reckless B malice; negligent C specific intent; gross D specific intent; negligent

A

For purposes of common-law murder, which of the following is NOT considered malice aforethought? A Intent to inflict bodily injury B Abandoned and malignant (depraved) heart C Intent to kill D Felony murder during the commission of a BARRK felony

A

Regarding accomplice liability, which of the following is TRUE? A A person who is a member of the class protected by a statute cannot be an accomplice. B An accomplice can be liable for any crime that was committed by the principal. C An accomplice cannot be convicted of a crime for which the principal cannot be convicted. D An accomplice who cannot be a principal of a crime cannot be convicted of the crime.

A

Regarding duress, which of the following is TRUE? A Duress is a defense to all crimes except intentional murder. B Duress is a defense to all crimes except specific-intent crimes. C Duress is a defense if injury to property is threatened. D Duress is a defense even if the defendant unreasonably believed that the only way to avoid death was to commit the crime.

A

Regarding larceny, which of the following is FALSE? A Obtaining services from another person without paying for those services constitutes larceny at common law. B Any movement of tangible, personal property may constitute larceny even though the property has not been removed from the owner's premises. C Consent obtained by fraud does not excuse conduct that otherwise would be larceny. D Borrowing tangible, personal property with the intent to return it does not constitute larceny.

A

Which of the following is FALSE regarding assault? A Voluntary intoxication is a defense to "fear of harm" assault. B Involuntary intoxication is a defense to attempted battery assault. C If there is an actual touching, attempted battery assault rises to a battery. D If a defendant takes a substantial step toward committing a battery but fails, he is guilty of assault.

A

Which of the following statements regarding actus reus is TRUE? A The act requirement may be satisfied by an omission or failure to act. B An evil thought, by itself, is sufficient as the basis for a crime. C A person may be liable for a crime even if the actus reus element has not been satisfied. D An act committed during sleep is considered a voluntary act.

A

Which of the following tests for insanity is also called the "but-for" test for insanity? A Durham rule B M'Naghten test C Model Penal Code test D Irresistible impulse test

A

mens rea

A. Mens Rea—The Common Law States of Mind 1. Specific Intent - first state of mind under common law. o Defendant not only committed the actus reus but did it for the very purpose of causing the result that the law criminalizes Example 10: Burglary is defined as the entering of a dwelling with the intent to commit a felony once inside (crime that has intent or specific intent requirement - tells u what intent u need to have is - intent to commit specific felony once inside). You have a suspicion that your neighbor might be a hoarder. One day, you climb into your neighbor's house through a window to look around. Once inside, you happen to see a newspaper from the day that you were born. You take the newspaper and leave the house. Have you committed burglary? Although u actually have committed crime of larceny, u haven't commited burglary. Because u didn't enter house with intnt of commiting burglary - when u went in u were just curious about knowing what might be inside. So although you committed crime, you didn't commit common law burglary - because that is a specific intnt crime - can only commit if at time you entered house you intended to commit felony - it could be larceny, arson, assault, but you didn't have any intent to commit crime when u entered house other than trespass really.

embezzlement

A Variation of Larceny o A defendant starts out having the victim's consent to have the property but commits embezzlement by converting the property to his own use after he gets it with the victims consent. For larceny its taking thing without victims consent. Here defendant has taken property with victims consent but then does something to deprive victim of the property. Example 66: Jake is a teller at King Savings Bank. One day Lou, a depositor at the bank, comes to the window and hands Jake a check, made out to Zach and endorsed over to Lou, for deposit to Lou's account. Jake knows that Lou is an enforcer for an organized crime family, and strongly suspects that Lou obtained Zach's check through force. Believing the check to be ill-gotten gains, Jake instead deposits the check to the account of the Make-a-Wish Foundation, a local charity. Is Jake guilty of larceny because he stole Lou's check? No hes not, because he had lous permission to have cheque in his hand. Is Jake guilty of embezzlement? Yes because hetook money and kept it. That's true even if money not rightfully lous in first place. You don't get to act as an agent of the law. Even though the money was not rightfully Lou's in the first place? Yes because hetook money and kept it. That's true even if money not rightfully lous in first place. You don't get to act as an agent of the law. Even though he didn't keep the money for himself? What about fact that jake didn't keep money for himself? its still embezzlement, even if putting in make a wish account because he deprived lou of his property permanently. Embezzlement adds twist to larceny which is u got property in first place with the victims consent but then converted property to another use.

extortion

A Variation of Robbery - Rather than threatening imminent physical harm or using force, extortion involves threats of future harm (can include threats of future non-physical harm) Example 68: Xenia approaches Chris at a party and admires her necklace. Xenia tells Chris, "Give me that necklace or else I'll tell your husband that I saw you coming out of a hotel room last weekend with Ben." Chris turns to walk away and Xenia grabs the necklace, pulling it off Chris's neck. Of which crimes is Xenia guilty? Robbery: yes because she obtained the necklace by force. Attempted extortion: yes, because she threatened to commit larceny but failed. Extortion: no, she didn't actually obtain the property through the threat. Chris turned to walk away so threat wasn't how xenia got property, but robbery was. Editorial Note 2: This conclusion relies on the minority approach to extortion, which requires both a threat and actually obtaining the property. Under modern statutes, the majority of jurisdictions would find the threat alone to be sufficient for extortion.

OTHER CRIMES - battery

A. Battery 1. Definition o Unlawful o Application of force o To another person o That causes: § Bodily harm OR § An unlawful touching 2. Elements o Unlawful: Consent is a complete defense to battery (e.g., boxing match). o Application of force: Need not be a great deal of force; the slightest touch can constitute force in some cases o Battery is a general-intent crime, so voluntary intoxication and unreasonable mistake of fact (if unreasonable in thinking someone said punch me) are not available defenses. Like example: playing poker and s omeone says hits me, he means add more cards not literally hit him. That would be unreasonable and because battery is a general intent crime, that's not a defense. o use of force Does not require actual physical contact between the defendant and the victim (e.g., throwing a rock that hits someone can be a battery)

HOMICIDE

A. In General 1. Definition o Homicide: The killing of a living human being by another human being § Animals cannot commit a homicide and killing an animal is not a homicide - maybe crime but not homicide; § Victim cannot already be dead; § Suicide is not homicide; but assisting someone to commit suicide can be a homicide in jx that forbids assisted suicide.

Which of the following persons is NOT liable to a victim for failure to act? A A person who created the peril and failed to prevent the victim from being injured by the peril B A person who assumed the duty of care to rescue a victim in peril, but then changes their mind C A bystander who is aware of the victim's peril D A parent who sees the victim, who is her child, in peril but fails to help the child

C

A(n) ___________ is a person who assists the defendant after the crime has been committed. A Principal B Accomplice C Co-principal D Accessory after the fact

D

NEGATING MENS REA - mistake of law

A. Mistake • Often, a defendant will claim that some mistake—regarding either facts in the world or the state of the law—negates his mens rea and thus he cannot be convicted of a crime for which there are both actus reus and mens rea elements. 1. Mistakes of Law o Mistakes about what the law forbids and what it permits o in general rule is ignorance of the law is no excuse. Mistake of law generally doesn't excuse defendant. Example 30: You thought the speed limit was 80 mph because last time you drove on that highway there was a sign that said 80 mph. In reality the speed limit was 65 mph. The fact that you didn't realize the speed limit was 65 mph is not a defense. o Three potential exceptions: 1) Reliance on high-level government interpretations Example 31: If a regulation states that something is permitted and you are later prosecuted for that conduct because the regulation was wrong, the regulation might negate the mens rea element. Note 5: Relying on your own lawyer's advice does not generally fall into this exception. 2) Lack of notice 3) Mistake of law that goes to an element of specific intent (applies only to the "FIAT" crimes or specific-intent crimes) can be relevant to negating defense. For Fiat crimes, for specific intent crimes mistake of law going to element of specific intent might negate the defendants mens rea. Example 32: Quentin, a solo practitioner, practices law out of his rented apartment. He drills holes in the wall of his apartment for bookshelves. The landlord tells Quentin to stop drilling and grabs Quentin's drill. Quentin pushes the landlord, causing him to fall and injure his shoulder. Quentin is charged with battery and with the statutory crime of "knowingly damaging the property of another person, [with the intent to deprive that person of the property]." Quentin argues that he thought a tenant was allowed to alter the walls of his apartment. He argues that he was defending his own property when he pushed the landlord. Quentin had the mistaken belief that he was protecting his own property. Is that a defense to a charge of battery? Answer is no because battery is not a specific intent crime. As long as you voluntarily applied force to another person, and here quentin did that, he'll be guilty as long as he was not entitled to do so. Can Quentin's mistake about the property be a defense to violating the statute? Quentins mistake about property law can be a defense to violating a statute that prohibits knowingly damaging the property of another person with the intent to deprive that person of the property. Because quentin didn't knowingly deprive a person of the property since he believed he could alter the walls. For a specific intent crime, a fiat crime, a mistake about law that negates the thing to which the specific intent attaches, can be a defense. That's mistake of law - other than with regard to fiat crimes you wont see a successful mistake about law on bar exam.

important to remember

Aidors and abettors are guilty of same crime as principal if they helped before or during the commission of the offense - by contrast, people who help after the offense are guilty of a separate offense like obstruction of justice, harboring a fugitive, money laundering, or the like.

attempt - very important A man planned to rob a city bank. Waiting until he saw the bank's security guard leave the building to take a walk around the perimeter, the man entered the bank and walked up to a teller with his hand pointed in his pocket, as if he had a gun. The teller had her back to the man and did not see him at all. Before the teller could turn around, the security guard re-entered the building. Seeing the guard, the man took his hand out of his pocket and ran out of the building.The man was arrested and charged with attempted robbery of the bank. Should the man be found guilty? A Yes, whether or not he abandoned his plan. B Yes, because robbery is a specific intent crime. C No, because he successfully withdrew from the robbery. D No, because he used no actual force on the teller.

An attempt requires a specific intent to commit a criminal act coupled with a substantial step taken toward the commission of the intended crime. At common law, once the defendant has taken a substantial step toward the commission of the offense, the defendant may not legally abandon the attempt to commit the crime because of a change of heart. Upon the completion of a substantial step, the crime of attempt is completed; there can be no abandonment or withdrawal. The facts here indicate that the man planned and intended to rob the bank, so the specific intent element is satisfied. He entered the building and pretended to have a gun in his pocket. This constitutes a substantial step toward the commission of the intended crime. Answer choice B is incorrect because, while a true statement, it has no bearing on whether the man should be found guilty of attempted robbery; the crime of attempt can be found even when the underlying crime is a general intent crime. Answer choice C is incorrect, as withdrawal would not constitute a common-law defense. Note that even in a jurisdiction that recognizes abandonment or withdrawal as a defense to attempt, the man's abandonment was not voluntary and therefore does not constitute a defense; the abandonment was motivated by a desire to avoid detection by the security guard. Answer choice D is incorrect, as it is irrelevant that the man did not use actual force on the teller. As noted above, attempt merely requires specific intent to commit the criminal act along with a substantial step toward the commission of the intended crime. While robbery requires the actual use of force or intimidation, attempted robbery does not.

false pretenses

Another Variation of Larceny - The defendant obtains title to someone else's property through an act of deception. Example 67: Paying for goods with counterfeit money or a bad check

Which of the following statements regarding mistake of law is FALSE? A Generally, ignorance of the law is not a valid defense. B Mistake of law is a valid defense if the defendant relied on an official interpretation of the law. C Mistake of the law is a valid defense even if the defendant's mistake did not negate the required intent. D Mistake of law is a valid defense if a statute defining a crime was not reasonably made available to the defendant prior to the conduct.

C

FILL IN THE BLANK. Under the _______________ approach, a person cannot withdraw from a conspiracy after an agreement has been made. A federal B Model Penal Code C federal and Model Penal Code D common law

D

A homeowner was selling illegal drugs out of his house, which was located in a well-heeled suburban neighborhood. A customer, knowing that the homeowner likely had a large amount of cash from selling drugs, knocked on the homeowner's door. When the homeowner opened the door upon recognizing the customer, the customer pulled out a toy pistol that looked like a real gun and demanded money. The customer followed the homeowner inside and over to a table in the entryway with a drawer in which the homeowner told the customer there was money. The homeowner instead pulled a gun out of the drawer and shot and killed the customer. The homeowner is arrested and charged with murder. The jurisdiction does not follow the "retreat doctrine."Should the homeowner be convicted of the customer's murder? A No, because it was reasonable for the homeowner to believe that his life was at risk. B No, because the jurisdiction does not follow the retreat doctrine. C Yes, because the customer was not armed with a deadly weapon. D Yes, because the homeowner was committing a crime by selling the drugs and therefore was not privileged to use self-defense.

Answer choice A is correct. A person who is not the aggressor is justified in using reasonable force against another person to prevent immediate unlawful harm to himself. Because the toy pistol held by the customer looked like a real gun, it was reasonable for the homeowner to believe that his life was in imminent danger. Accordingly, it was reasonable for the homeowner to use deadly force against the customer to protect his own life. Answer choice B is incorrect. Even in a jurisdiction that follows the "retreat doctrine," retreat is never required when the person employing deadly force in self-defense is in his own home (i.e., the "castle doctrine"). Answer choice C is incorrect because, even though the customer was unarmed, if it was reasonable for the man to believe his life was at risk from a toy pistol that appeared to be a real gun, he was privileged to use self-defense. Answer choice D is incorrect because the fact that the man illegally sold drugs does not affect his right to use self-defense under these circumstances.

A chemist working in a research laboratory was working late hours on a project to create a chemical substance to combat ozone-depleting substances and counteract biological consequences like skin cancer that result from the increased UV exposure due to ozone depletion. One night after work, the chemist was walking back to her car when she was attacked in the parking lot. Although the man was wearing a ski mask, she recognized the man as part of the laboratory's janitorial staff due to a distinctive tattoo on his arm. Instead of telling the police, the chemist set a trap so that a lethal mixture of chemicals would blow up when he entered one of the rooms that he normally cleaned. One of the chemist's coworkers noticed that the chemist was utilizing chemicals that she normally did not use as part of her ozone-depletion project. When he questioned the chemist, she told him to mind his own business. That night, the chemist successfully murdered the janitor. The police arrested the chemist for murder when they discovered her fingerprints on some of the chemical containers. During the course of their investigation, they asked the coworker whether he saw anything suspicious the night of the murder. The coworker did not mention the array of suspicious chemicals he saw gathered in the laboratory room, and told the police that he did not see the chemist at the laboratory on the night of the murder.If the coworker were to be charged with a crime, would he be guilty? A Yes, as an accessory after the fact B Yes, as a conspirator C Yes, as an accomplice to murder D Yes, as a principal to murder

Answer choice A is correct. An accessory after the fact is a person who aids or assists a felon in avoiding apprehension or conviction after commission of the felony. An accessory after the fact must know that a felony was committed, act specifically to aid or assist the felon, and give the aid or assistance for the purpose of helping the felon avoid apprehension or conviction. The mere failure to report a crime is not generally itself a crime. However, a person who gives false information to the police in order to prevent the apprehension of a felon can be an accessory after the fact. Here, although the coworker saw the array of suspicious chemicals in the laboratory room, he did not assist the chemist or help her to avoid apprehension. While, his failure to report the murder (which he did not witness or know about) is not a crime, giving false information to the police to protect the chemist may make him guilty as an accessory after the fact. Answer choice B is incorrect because to form a conspiracy, there must be an agreement between two people to accomplish an unlawful purpose. Here, there was no agreement between the chemist and the coworker to kill the janitor. Answer choice C is incorrect. Under the majority and MPC rule, an accomplice is a person who, with the purpose of promoting or facilitating the commission of the offense, aids or abets a principal prior to or during the commission of the crime. Here, the coworker did not aid or abet the chemist prior to or during the commission of the crime. Mere knowledge that another person intends to commit a crime is not enough to make a person an accomplice. Thus the coworker is not guilty as an accomplice. Answer choice D is incorrect. An accessory after the fact is not subject to punishment for the crime committed by the felon, but instead has committed a separate crime. Thus, although the chemist is guilty of murder, the coworker would only be guilty of a separate crime.

intentional killing such as euthanasia with consent of dying party is what?

Answer choice A is correct. Common-law murder is the unlawful killing of another human being with malice aforethought. Here, the son intentionally injected his mother with drugs at her request for the purpose of causing her death, and she died as a result. Consent is not a defense to homicide, so a "mercy killing" (i.e., euthanasia) can be a criminal homicide even if the person was willing to die because of a painful terminal illness, as in this case. On these facts, the son may properly be convicted of murder. Answer choice B is incorrect. Voluntary manslaughter is homicide committed with malice aforethought, but also with mitigating circumstances (e.g., the "heat of passion"). A defendant acts in the heat of passion if he was provoked by a situation that could inflame the passion of a reasonable person to the extent that it could cause that person to momentarily act out of passion rather than reason. Here, the son's research into the means by which to effect his mother's death does not suggest an action based on passion rather than reason. Answer choice C is incorrect because involuntary manslaughter is an unintentional homicide committed with criminal negligence. Here, the son intentionally caused his mother's death. Answer choice D is incorrect. Providing a person with the means by which that person can commit suicide generally does not make the provider guilty of murder as an accomplice but instead guilty of a lesser crime, such as assisting a suicide. Here, because the son was the actor who caused the mother's death rather than assisting her in killing herself, he did more than just providing a means and can be found guilty of murder.

A woman suffered from a debilitating disease, and her husband convinced her that having sexual intercourse with him would cure the disease. The husband knew that his statement was false. Relying on this statement, the woman gave her consent, and the two had sexual intercourse. Later, the woman learned that intercourse could not and did not cure her disease, and notified the police. Rape is statutorily defined as "sexual intercourse with a female against her will." The husband was convicted of rape.If the husband appeals the conviction, how should the appellate court rule on the appeal? A Reverse the conviction, because the intercourse was not against the woman's will. B Reverse the conviction, because the man was the married to the woman. C Affirm the conviction, because the man obtained the woman's consent through fraudulent means. D Affirm the conviction, because the man had the specific intent to rape the woman.

Answer choice A is correct. Fraudulent conduct does not negate consent in most situations. Here, the man induced the woman to consent to sexual intercourse through false promises--"fraud in the inducement"--but that, standing alone, does not negate consent. Here, the man made false promises to obtain consent, but did not conceal the actual nature of the act; consequently, the fraud was in the inducement, not in the factum. Answer choice B is an incorrect statement of the law; the applicable statute does not carve out immunity for persons accused of raping a spouse. Answer choice C is incorrect because, as discussed above, fraud in the inducement does not negate consent. Answer choice D is also an incorrect statement of the law; rape is a general intent crime.

Example 72: Thomas has a crush on Ursula. After getting drunk one night, Thomas concludes that Ursula truly loves him. He goes to her room and knocks, but she won't open the door, so Thomas breaks it down. Once inside, he tries to kiss Ursula. But she's fast and he's drunk, so she gets away. Thomas falls over and grabs Isaac—who is also in the room—injuring Isaac's arm. Is Thomas guilty of battery against Ursula for trying to kiss her?

Answer is no because he didn't actually kiss or touch her. If he did kiss her without her consent, it would be battery. Is Thomas guilty of attempted battery (assault)? No because if his intoxication prevented him from having the requisite mens rea, attempted battery which is assault is a specific intent crime. So long as the defendant cant maintain the specific intent, he cant have requiste intent for that crime. Did Thomas commit a battery against Isaac? Yes he did. Because that crime is a general intent crime for which his intoxication cant provide a defense. And he also committed assault - He also committed assault by breaking down the door because he put them in fear of imminent bodily harm and his voluntary intoxication is not a defense. Remember - For battery or putting someone in fear, those are general intent crimes. Whereas assault with intent to commit battery is a specific intent crime. So defenses applying to these are different from each other.

A woman was charged with battery for trying to stab a man with a swizzle stick at a bar. During the trial, a toxicology report was admitted by the defense proving that there was no alcohol in the woman's system when she committed the act, but that there were traces of Rohypnol (i.e. "the date rape drug") in her system—the effects of which are very similar to intoxication. The defense asked for instructions that if the jury believed the woman was under the influence of Rohypnol at the time of the crime—and that she would not have committed the crime otherwise—that the jury should acquit her.What evidence would most support the defense's argument in favor of this jury instruction? A Evidence that a man talking to defendant at a bar earlier that night had put the Rohypnol in her club soda when she was in the bathroom. B Evidence that the woman was prescribed Rohypnol by a doctor because it also acts as an effective sleeping pill, and now the defendant is addicted to the drug. C Expert testimony that a reasonable person taking Rohypnol would experience a similar level of intoxication. D Evidence that the victim was teasing and laughing at the defendant for being so intoxicated at a bar.

Answer choice A is correct. Involuntary intoxication would most strongly support an acquittal. Involuntary intoxication is a defense when the intoxication serves to negate an element of the crime, including general as well as specific-intent and malice crimes. To be considered involuntary, the intoxicating substance must have been taken without knowledge of the intoxicating nature of the substance, including substances taken pursuant to medical advice. If someone put the Rohypnol in the woman's drink without her knowledge, then she would have a defense to the general intent crime of battery. Answer choice B is incorrect because voluntary intoxication is not a defense to battery, and addiction does not qualify as involuntary. Answer choice C is incorrect for the similar reason that voluntary intoxication is not a defense to battery, regardless of the intoxicant's effect on a reasonable person. Answer choice D is incorrect because provocation is generally not a defense to battery.

A woman was out late drinking at a party, and decided to walk home rather than drive home drunk. However, she was very inebriated, and lost the keys to her house. It was very cold outside, so she decided to sneak into her neighbor's home for a few hours to stay warm until morning. He happened to be out of town, but she knew that one of the ground floor windows could be pried open easily and there was no alarm system. She entered their home through the loose window, covered up under some warm blankets, and slept for a few hours. When she woke up, she had sobered up, and as she was leaving, she grabbed her neighbor's heavy coat because it was still cold outside, planning to return it next time she saw him. The jurisdiction in question only recognizes trespassing as a tort, not as a criminal offense.What is the most serious crime, listed below in ascending order of seriousness, of which the woman can be convicted? A No crime B No crime, based upon the defense of necessity C Larceny D Burglary

Answer choice A is correct. The woman lacked the requisite intent to commit either of the crimes listed, as explained below. Accordingly, she would not be found guilty of either crime. Answer choice B is incorrect because although necessity can be used as a defense if forces of nature cause the defendant to commit what would otherwise be a crime, there was no crime in this case. Therefore, the defense of necessity is not applicable. Even if the woman's trying to escape the cold after losing her keys while inebriated constituted necessity, which is unlikely, the woman would not need to raise a defense because she did not complete the elements of either crime. Answer choice C is incorrect because larceny is the trespassory taking and carrying away of the personal property of another with the intent to permanently deprive him of it. The woman attempted only to borrow the coat, with the intent to return it, so she cannot be convicted of larceny. Answer choice D is incorrect because to be guilty of burglary, at the time of the breaking and entering, the defendant must have the intent to commit a felony (e.g., larceny, robbery, rape, murder) inside the dwelling. The defendant did not have the intent to commit larceny (or any other crime) when she entered the neighbor's home; she just wanted to stay warm.

An employee was up for a promotion but was passed over by his boss for a female colleague with more experience. After learning he had not received the promotion, he became angry with his boss and convinced himself that the colleague and the boss, who was married, were involved in a relationship. He therefore contacted the boss's wife and convinced her to shoot the boss. They stated that they would not harm the colleague, as it might make their involvement too obvious. The employee provided her with a gun. The next day, the boss's wife approached the colleague and the boss in their office parking lot. The wife, who was not an experienced shooter, shot the colleague in the arm, and then panicked and ran off. An onlooker rushed the colleague to the hospital. Although the injury was not life threatening, she contracted an infection during surgery and died the following week. A later investigation revealed that the infection was a result of medical malpractice that occurred during the surgery. The crime was eventually traced back to the employee and the boss's wife, and they were both charged in connection with the colleague's death.The employee is most likely to be convicted of which of the following crimes? A Murder, attempted murder, and conspiracy to commit murder. B Murder, conspiracy to commit murder, and solicitation. C Attempted murder and conspiracy to commit murder only. D Murder and conspiracy to commit murder only.

Answer choice A is correct. To prove a homicide, the prosecution must show that the defendant was the actual and proximate cause of the victim's death. If the victim would not have died but for the defendant's act, then the defendant's act is the actual cause of the killing. To prove proximate cause, the death must be foreseeable. A defendant's conduct is deemed to be foreseeable if death is the natural and probable result of the conduct. Actions by a third party (e.g., negligence by the doctor treating the victim) are generally foreseeable. In this case, the boss's wife was the actual and proximate cause of the colleague's death, and thus would be guilty of murder of the colleague. Although the crimes of murder and attempted murder would usually merge, they only merge as to the same person. While the boss's wife could be found guilty of the colleague's murder, she could also be found guilty of the separate crime of attempting to kill the boss. The wife had the specific intent to kill the boss, and took a substantial step towards that result (approaching the boss in the parking lot and shooting the gun). The employee would be guilty as a co-conspirator. Conspiracy is an agreement between two or more persons to accomplish an unlawful purpose with the intent to accomplish that purpose. A conspirator can be convicted of both the offense of conspiracy and all substantive crimes committed by any other co-conspirators acting in furtherance of the conspiracy. In this case, because the employee and the wife agreed to kill the boss, the employee would be guilty of both murder and conspiracy, even though the person killed was actually the colleague and not the boss. Answer choice B is incorrect because it fails to take into account the attempted murder of the boss. Answer choice C is incorrect because the employee would be guilty of murder, because the colleague's death was a natural and probable cause of the wife's actions. Answer choice D is incorrect because the employee would also be guilty of the attempted murder of the boss; because the boss was not actually murdered, that attempt would not merge with the actual conviction for the colleague's murder.

Three coworkers were employed at a jewelry store. They were always complaining to one another about the long hours, the paltry pay rate, and the lack of medical benefits. Two of the coworkers came up with a plan to steal a valuable collection of gems from the jewelry store, but it was a three-person job; they asked the third coworker to join in on their plan, but he refused. The two coworkers knew how devoted the third coworker was to his family, so they threatened to kill his wife and kids if he did not help them or if he tried to foil their plan by notifying the police. Believing that he had no other choice, the third coworker joined in on the plan. The three men entered the jewelry store that night, having received the permission of their boss in order to polish all of the jewelry in preparation for a jewelry show the next day. While the two coworkers broke into the wall safe where the gems were kept, the third coworker kept watch for the night guard on his hourly walk-through of the store premises. The night guard decided to do his walk-through ten minutes early, saw the men breaking into the safe, and pulled out his gun. The excitement of the event caused the night guard to have a heart attack, which was fatal. The police entered at that moment and arrested the three men for burglary and felony murder of the night guard.What is the third coworker's best defense against the felony murder charge? A He did not intend to kill the night guard. B He was under duress to commit the burglary. C He had permission to enter the jewelry store. D He could not have foreseen the security guard's heart attack.

Answer choice B is correct. A defendant charged with felony murder may claim duress as a defense to the underlying felony and avoid conviction for felony murder. Claiming duress is the third coworker's best defense because the other two coworkers threatened to kill his family if he told the police or refused to help them steal the gems. Answer choice A is incorrect because felony murder is an unintended killing proximately caused by and during the commission or attempted commission of an inherently dangerous felony; intent to kill is not required. Answer choice C and D are incorrect. Burglary is the breaking and entering of the dwelling of another at nighttime with the specific intent to commit a felony therein. If entry is gained with consent, a breaking can still occur if the defendant breaks into a part of the dwelling structure, such as by opening a closet door or wall safe. The jewelry storeowner gave the three men permission to enter the store, but since the men broke into the jewelry store's wall safe, this qualifies as a breaking. Thus answer choice C is incorrect. Answer choice D is incorrect because it is always foreseeable that the victim of a violent crime will have a heart attack.

Note Answer choices C and D!!!! Three coworkers were employed at a jewelry store. They were always complaining to one another about the long hours, the paltry pay rate, and the lack of medical benefits. Two of the coworkers came up with a plan to steal a valuable collection of gems from the jewelry store, but it was a three-person job; they asked the third coworker to join in on their plan, but he refused. The two coworkers knew how devoted the third coworker was to his family, so they threatened to kill his wife and kids if he did not help them or if he tried to foil their plan by notifying the police. Believing that he had no other choice, the third coworker joined in on the plan. The three men entered the jewelry store that night, having received the permission of their boss in order to polish all of the jewelry in preparation for a jewelry show the next day. While the two coworkers broke into the wall safe where the gems were kept, the third coworker kept watch for the night guard on his hourly walk-through of the store premises. The night guard decided to do his walk-through ten minutes early, saw the men breaking into the safe, and pulled out his gun. The excitement of the event caused the night guard to have a heart attack, which was fatal. The police entered at that moment and arrested the three men for burglary and felony murder of the night guard.What is the third coworker's best defense against the felony murder charge? A He did not intend to kill the night guard. B He was under duress to commit the burglary. C He had permission to enter the jewelry store. D He could not have foreseen the security guard's heart attack.

Answer choice B is correct. A defendant charged with felony murder may claim duress as a defense to the underlying felony and avoid conviction for felony murder. Claiming duress is the third coworker's best defense because the other two coworkers threatened to kill his family if he told the police or refused to help them steal the gems. Answer choice A is incorrect because felony murder is an unintended killing proximately caused by and during the commission or attempted commission of an inherently dangerous felony; intent to kill is not required. Answer choice C and D are incorrect. Burglary is the breaking and entering of the dwelling of another at nighttime with the specific intent to commit a felony therein. If entry is gained with consent, a breaking can still occur if the defendant breaks into a part of the dwelling structure, such as by opening a closet door or wall safe. The jewelry storeowner gave the three men permission to enter the store, but since the men broke into the jewelry store's wall safe, this qualifies as a breaking. Thus answer choice C is incorrect. Answer choice D is incorrect because it is always foreseeable that the victim of a violent crime will have a heart attack.

Throughout her life, the defendant had visions of an unknown woman who told her to commit inappropriate, and sometimes violent, acts. The defendant truly believed that the unknown woman had her best interests at heart and that, therefore, whatever the woman said to do was right. The defendant had a history of self-medicating with drugs and alcohol, which helped her ignore the visions. One month, the defendant was so consistently intoxicated that she inadvertently forgot to pay her rent. When the defendant's landlord came to the door to remind her to pay, the defendant began to have visions of the unknown woman who told her to kill her landlord. The defendant grabbed a letter opener that happened to be by the front door and stabbed her landlord with it.The defendant has been charged with second-degree murder in a jurisdiction that uses the M'Naghten test. A state statute defines second-degree murder as common-law murder. What is the defendant's best defense against this charge? A Involuntary intoxication B Insanity C No malice aforethought D No premeditation

Answer choice B is correct. A defendant may be found not legally responsible for a crime if her acts meet one of the definitions of criminal insanity. One such test, the M'Naghten test, requires that, because of a defect of reason due to a mental disease, the defendant did not know at the time either the nature and quality of the act or the wrongfulness of the act. Here, the defendant's honest belief that the unknown woman in her visions would not advise her to do anything wrong may be sufficient to satisfy this requirement. Without knowing that his act was wrong, the defendant could not have formed the requisite criminal intent. Thus, it is the defendant's strongest defense. Answer choice A is incorrect because the defendant was nothing indicates that the defendant was involuntarily intoxicated; she voluntarily self-medicated to deal with her visions. Answer choice C is incorrect because the defendant did have malice aforethought. She stabbed landlord despite the fact that this created an unjustifiably high risk to human life. Thus, this defense would not apply to the facts. Answer choice D is incorrect. Premeditation is the action of planning or reflecting on a crime beforehand. Is it generally the defining characteristic of first-degree murder. Here, while it is true that the defendant did not appear to have premeditated this attack, premeditation is not a requirement for second-degree murder. Therefore, lack of premeditation is not a defense to second-degree murder.

The owner of a home was awakened in the middle of the night by the sound of breaking glass. Erroneously concluding that he had been dreaming, the owner went back to sleep. Several minutes later, the owner was again awakened by a loud crashing sound coming from his living room. The owner grabbed a pistol that he kept by his bedside and went to investigate. In his living room, the owner saw a burglar place the owner's laptop computer into a bag and then watched as the burglar quickly headed for the back door of the house. Before the burglar could escape, the owner intentionally shot and killed the burglar. The owner has been charged with the murder of the burglar.Is the owner's killing of the burglar justified? A No, because the owner could not use deadly force to defend his property. B No, because the burglar was heading for the back door. C Yes, because the victim was a burglar. D Yes, because the buglar was in the owner's home.

Answer choice B is correct. Although generally there is no right to use deadly force in defense of property, a person may use deadly force to prevent or terminate forcible entry into a dwelling if the occupant reasonably believes that the intruder intends to commit a felony inside. However, the use of deadly force against an intruder exiting the dwelling is generally not permissible. Here, although the owner had the right to use deadly force against the burglar in order to prevent the burglar from stealing property from the owner's home, the owner did not have that right to prevent the burglar from exiting the home. Therefore, the owner's killing of the burglar was not justified. Answer choice A is incorrect. Although a property owner generally cannot use deadly force to defend his property, a person may use deadly force to prevent or terminate forcible entry into a dwelling if the occupant reasonably believes that the intruder intends to commit a felony inside. Answer choice C is incorrect. Although the owner had the right to use deadly force against the burglar in order to prevent the burglar from stealing property from the owner's home, the owner did not have the right to prevent the burglar from exiting the home. Thus, the fact that the victim was a burglar does not justify the owner's killing. Answer choice D is incorrect. Although a homeowner may use self-defense to protect himself or others from the threat of deadly force and is not required to retreat when in his own home before doing so, the owner here was not acting in self-defense or in defense of others, but in defense of his property.

A hair technician agreed to implant synthetic hair into the scalp of her customer. The customer explained to the hair technician that he often broke into people's houses to take jewelry, and he wanted to avoid leaving his own hair at the scene as incriminating evidence. The technician, knowing that a great deal of the synthetic hair would fall out due to implant rejection, that the strands would still have the customer's DNA at the root, and that the procedure would be no help at all in hiding forensic evidence, said nothing. The technician completed the procedure and charged her regular hourly rate. The next night, the customer broke into a house, stole a diamond ring, and was convicted of burglary based on hair evidence he left behind. The jurisdiction follows the MPC rule regarding accomplice liability.Should the hair technician be convicted of burglary as an accomplice? A No, because she did not break into the house and steal the ring. B No, because she did not have the requisite intent for accomplice liability. C Yes, because she knew that the customer broke into houses regularly. D Yes, because she failed to notify legal authorities in a timely manner.

Answer choice B is correct. An accomplice (i.e., an accessory before the fact or a principal in the second degree) is a person who, with the requisite mens rea, aids or abets a principal prior to or during the commission of the crime. Under the majority and MPC rule, a person is an accomplice in the commission of an offense if he acts with the purpose of promoting or facilitating the commission of the offense. Thus, the accomplice must intend that her acts will assist or encourage the criminal aim. Here, because she knew that the procedure would not help the customer commit a crime, the technician did not act with the requisite intent to encourage or assist the customer in the commission of the crime. Answer choice A is incorrect because an accomplice can be guilty of a crime without acting as a principal. An accomplice is responsible for the crime to the same extent as the principal. Answer choice C is incorrect. Mere knowledge that another person intends to commit a crime is not enough to make a person an accomplice. Under the majority and MPC rule, a person is an accomplice in the commission of an offense if he acts with the purpose of promoting or facilitating the commission of the offense. Thus, the accomplice must intend that her acts will assist or encourage the criminal aim. The technician's mere knowledge of the customer's plan is not sufficient to establish the requisite mens rea under the MPC. Answer choice D is incorrect because mere knowledge that a crime may occur does not obligate a person to report that information to legal authorities. In addition, because she did not possess the necessary mens rea to be an accomplice of her customer, she is not required to notify legal authorities in order to legally withdraw and thereby avoid criminal liability.

An off-duty police officer was having a drink with his wife in a bar. A man walked by their table, and made a lewd comment about the wife as he passed. The officer jumped to his feet and asked the man to repeat what he had said. The man, who was obviously intoxicated, repeated the comment. The officer then pushed the man, causing the man to fall backwards. The man rose to his feet, pulled a knife from his waistband, and walked toward the officer. As the man lunged at the officer with the knife, the officer pulled his gun from his waistband and shot the man in the leg. Although the officer did not intend to fire a lethal shot, the defendant had a disease that prevented his blood from properly clotting, and he died from blood loss due to the injury.Is the officer likely to be convicted of homicide? A No, because the officer had no knowledge of the man's disease when he shot him. B No, because the officer was entitled to use self-defense under the circumstances. C Yes, because the officer did not communicate his intent to withdraw from the altercation. D Yes, because the officer intended to do grievous bodily harm.

Answer choice B is correct. An initial aggressor gains the right to act in self-defense when an aggressor using nondeadly force is met with deadly force or the aggressor, in good faith, completely withdraws from the altercation and communicates this fact to the victim. In this case, the officer's nondeadly force was met with deadly force, and thus he was entitled to use self-defense. Answer choice A is incorrect because, if the officer had not been justified in using self-defense, he could be held liable for the death of the man because he intended to do grievous bodily injury to the man. Answer choice C is incorrect because the officer did not need to communicate his intent to withdraw in this instance because his use of nondeadly force was met with deadly force. Accordingly, the officer was entitled to use self-defense. Answer choice D is incorrect because the officer was entitled to use self-defense and thus would not be held liable for the man's death despite his intent.

Example 4: Steve can't get to sleep because he's worried about the bar exam. His doctor recommends that he take Ambien, a popular sleep medication. One of Ambien's side effects is that users sometimes sleepwalk or eat while not fully awake. One night after taking his Ambien, Steve arises from his bed, sleepwalks into the common area of the dormitory, reaches into the refrigerator, and eats Cecelia's jar of caviar. Can Steve be charged with larceny - sealing someone elses property?

Answer is no because his taking it wasn't voluntary act. Under influence of prescription drug at time he took the actus reus - so if just hungry and studying and went and stole it that would beactus reus. But because it wasn't under his conscious voluntary control, it doesn't count as actus reus.

Two women and a man were charged with conspiracy to rob a bank. No one else was involved. The three were arrested after an overt act had occurred, but before the robbery actually took place, and tried before a jury in a single proceeding. The two women were acquitted of the conspiracy, but the man was found guilty.On appeal, should the man's conviction be overturned? A Yes, because of the application of the "Pinkerton Rule." B Yes, because the three defendants were tried in a single proceeding. C No, because the prosecution need only prove the existence of a conspiracy. D No, because of the application of the "Pinkerton Rule."

Answer choice B is correct. At common law, a conspirator cannot be convicted of conspiracy if all other conspirators are acquitted at the same trial, because there must be more than one conspirator to have a conspiracy. Answer choices A and D are incorrect, as the "Pinkerton Rule" does not apply. The "Pinkerton Rule" says that every co-conspirator is guilty of any foreseeable substantive offense committed in furtherance of the conspiracy, regardless of actual knowledge of its commission. The issue here is whether a conspirator can be convicted when all other co-conspirators have been acquitted at the same proceeding. Answer choice C is incorrect because a prosecution need only prove the existence of a conspiracy when the other conspirators are never tried or apprehended. When one conspirator is tried and acquitted at the same trial, the other conspirator cannot be convicted of conspiracy.

A man's girlfriend worked the night shift at a seedy motel where many unsavory characters stayed and frequently engaged in illegal activities like drug use and prostitution. The man told his girlfriend to quit her job because it was an unsafe work environment, but she refused to listen to him. One night he decided to teach her a lesson. He put on a ski mask and brought a rubber knife with him to the motel. In a disguised voice, he demanded that his girlfriend empty the cash register. Unbeknownst to the man, the motel manager had recently hired a night patrolman to protect the motel employees. The patrolman saw the man in the ski mask, pulled out his gun and shot at the man, but his aim was poor, and he struck and killed the girlfriend instead. The man was charged with felony murder. The jurisdiction follows the proximate-cause theory of felony murder.What is the man's best defense against a felony murder conviction? A The man was not responsible for the night patrolman's act of shooting at the man and accidentally hitting the girlfriend. B The man did not intend to take property from his girlfriend by force. C The man did not complete the underlying felony because he never actually took the money from the cash register. D The man did not intend to kill his girlfriend; his intent was to demonstrate that she worked in an unsafe work environment.

Answer choice B is correct. Felony murder is an unintended killing proximately caused by and during the commission or attempted commission of an inherently dangerous felony. To convict a defendant of felony murder, the prosecution must establish the underlying felony and that the defendant committed that felony. Theft crimes, such as robbery, are specific intent crimes. If the man did not intend to actually take property from his girlfriend, then he has a defense to the crime of robbery. If the man could not be found guilty of the underlying felony, then he could not be found guilty of the crime of felony murder. Accordingly, this would be the man's best defense. Answer choice A is incorrect, as this jurisdiction applies the proximate-cause theory of felony murder, the prosecution may be able to successfully argue that the death was a natural and probable cause of the man's act. It is foreseeable that a night patrolman might have a gun and shoot at an armed robber. Accordingly, it is not as good an answer as answer choice B. Answer choice C is incorrect because the underlying felony does not need to be completed in order for a defendant to be guilty of felony murder. Answer D is incorrect because a defendant's intent regarding the death of the victim does not alleviate him from criminal liability under the felony murder rule.

A woman who lived alone in a rural area witnessed a child on a bicycle crash into a tree on the edge of her property. The crash left the child unconscious. The woman, who had neither a phone nor a car, ran to her nearest neighbor's house to borrow his car. The neighbor was not home, but the woman knew from getting rides with the neighbor in the past that the neighbor kept the keys to the car under the driver's seat. The woman used the keys to start the car and attempted to back it out of the neighbor's garage. In her excitement, she drove the car forward into the back wall of the garage, which rendered the car inoperable. Unbeknownst to the woman, the neighbor had stolen the car. The woman has been charged with larceny.Which of the following would be her best defense? A She did not take the car for her own benefit, but instead to help the unconscious child. B She intended only to borrow the car. C She did not drive the car off her neighbor's premises. D The owner's possession of the car was illegal.

Answer choice B is correct. Larceny is the (i) trespassory (ii) taking and carrying away (iii) of the personal property of another (iv) with the intent to permanently deprive that person of the property. If the defendant intends only to borrow the property with the ability to do so, then larceny does not occur because there is no intent to permanently deprive the owner of the property. Here, the woman only intended to borrow her neighbor's car to help the unconscious child, not to permanently deprive the neighbor of his car. Moreover, if property is taken with the intent to return the property and the property is accidently damaged or destroyed before it is returned, larceny has not occurred. Answer choice A is incorrect because the woman's reason for taking the car, while commendable, does not constitute a defense to larceny. Answer choice C is incorrect. Although larceny requires the defendant to carry the property away, this element of the crime, also known as asportation, requires only a slight movement of the property. Here, the woman drove the car forward and into the back of the garage wall. This would be sufficient for larceny. Answer choice D is incorrect because larceny merely requires that the property that is taken be in the possession of someone other than the defendant. Stolen property taken from a thief can constitute larceny unless the taker has a superior possessory interest in the property. Consequently, the fact that the neighbor is not the true owner of the car would not prevent the woman's action from being larceny.

A man was married to his childhood sweetheart, but after twenty years of marriage, they were no longer in love, and the man took on a mistress. One day, the man's wife disappeared. The next day, believing his wife to be dead, the man married his mistress. However, the wife was not dead, but had merely been visiting her sister. When she returned home a week later, she found her husband married to the mistress. The man told his wife that because their religion required polygamy, being married to two women was his religious duty. The wife disagreed and subsequently reported her husband to the police.Will the man likely be convicted of bigamy? A Yes, because his first wife objected to his marriage to the mistress. B Yes, because he was married to the mistress and the wife at the same time. C No, because he thought his wife was dead after she disappeared. D No, because his religion required him to be married to more than one woman.

Answer choice B is correct. Mens rea is the requirement of a guilty mind or legally proscribed mental state that a defendant must possess to commit a crime. Except for strict liability crimes, a crime is committed when a criminal act (actus reus) is coupled with a guilty mind—both the mental and physical elements exist at the same time. Strict liability crimes have no mens rea requirement and require only an actus reus. Bigamy is a strict liability crime, thus only an actus reus is required. An actus reus is a criminal act that must be a voluntary, affirmative act that causes a criminally proscribed result. In this case, even if the man thought his wife was dead and believed his marriage to the mistress to be legal, he will likely be found guilty of bigamy because he voluntarily married the mistress while still married to his wife. Answer choice A is incorrect because the wife's objection is immaterial to a bigamy charge. Even if she consented to the marriage, the man's actions are still illegal. Answer choice C is incorrect because it is only relevant that the man intended to marry the mistress. Answer choice D is incorrect because religious belief is not a defense to the crime of bigamy.

Example 63: A teenager sees a fancy car with the keys in it. The teen jumps in, thinking he will just drive the car around the block a few times and then return it. But he crashes the car. Is he guilty of larceny?

Answer is no because teenager intended to return car to parking lot. (Note: he may be guilty of other crimes, like joyriding. But just not common law larceny)

depraved heart murder question A carpenter was using a nail gun in the construction of a small outdoor deck on a house. The carpenter knew that the nail gun could fire a nail with sufficient force to kill a human being at close range. Aware of the presence of three other carpenters on the other side of the nearly finished deck, one of whom the carpenter disliked, the carpenter fired the nail gun twice at a table located between himself and the other three carpenters. The first nail from the gun struck the table but the second nail, ricocheting off the table, struck and killed the disliked carpenter. At trial, a jury, based on this evidence, found the carpenter guilty of murder.If the carpenter appeals his conviction on the grounds that the evidence was not sufficient to support his conviction, how should the appellate court rule? A Affirm the conviction, because the evidence is sufficient to establish that the carpenter acted with criminal negligence. B Affirm the conviction, because the evidence is sufficient to establish that the carpenter acted with malice. C Overturn the conviction and remand for a new trial, because the evidence is not sufficient for murder but can support an involuntary manslaughter conviction. D Overturn the conviction and remand for a new trial, because the evidence is not sufficient for murder but can support a voluntary manslaughter conviction.

Answer choice B is correct. Murder requires that the defendant act with malice in causing the death of another human being. One of the ways in which malice can be established is through proof of a reckless indifference to an unjustifiably high risk to human life (i.e., a depraved-heart murder). Here, firing a nail gun in the direction of another human being who is within the range of the gun is evidence of a reckless indifference to an unjustifiably high risk to human life. Answer choice A is incorrect because murder requires that the defendant act with malice. Criminal negligence does not rise to the level of malice. Answer choice C is incorrect. Although the evidence is clearly sufficient to support an involuntary manslaughter conviction based on criminal negligence (here, the defendant-carpenter engaged in a grossly negligent action that put the victim-carpenter at a significant risk of serious bodily injury or death), it is also sufficient to support a conviction for depraved-heart murder. Answer choice D is incorrect because the evidence does not support a voluntary manslaughter conviction. Voluntary manslaughter is homicide committed under mitigating circumstances and with malice aforethought. There are no facts that the defendant-carpenter acted with intent to kill after adequate provocation (e.g., in the heat of passion) in firing the nail gun in the direction of the victim-carpenter, even though he disliked the victim-carpenter.

A woman's husband told her that he was divorcing her, leaving the woman distraught. The woman and her sister decided to burn down the husband's new house while the husband was at work. The next day, the woman and her sister purchased the necessary supplies and drove to the husband's house. The sister waited in the car while the woman started the fire. They then drove off to avoid being noticed. Although the woman and her sister believed the husband was at work, in fact he had stayed home sick, and he was killed in the fire. The woman and her sister were both arrested and charged with both arson and homicide.May the woman and her sister be convicted of both arson and murder? A The woman and her sister may be convicted of arson only. B The woman and her sister may be convicted of either arson or murder. C The woman may be convicted of both arson and murder, but her sister may be convicted of arson only. D The woman may be convicted of both arson and murder, but her sister may not be convicted of either crime.

Answer choice B is correct. The woman could be found guilty of arson because she intentionally burned down her husband's house. Or, she could be convicted of her husband's murder, because the husband's death occurred during the commission of the arson, an inherently dangerous felony. However, because arson will merge with felony murder, she can only be convicted of one or the other. The sister is guilty as an accomplice because she aided and abetted the woman in the commission of the crimes. An accomplice is responsible for the crimes to the same extent as the principal. If the principal commits crimes other than those for which the accomplice has provided encouragement or assistance, an accomplice is liable for the other crimes if the crimes are the natural and probable consequences of the principal's conduct. In this case, the husband's death was a natural and probable consequence of burning down his house. Accordingly, both the woman and her sister would be held liable for either the arson or the husband's death. Answer choice A is incorrect because the woman and her sister would be liable for the husband's death under the felony murder rule, since arson is an inherently dangerous felony. Answer choice C is incorrect because the sister would be held liable for the husband's death since it was a natural and probable consequence of the arson. Answer choice D is incorrect because the sister would be held liable for both crimes as an accomplice.

A statute in a jurisdiction that follows the Model Penal Code provides:A person is guilty of battery if he causes bodily injury to another. An individual has been charged with violation of this statute.Which of the following describes the lowest level of culpability that the individual must possess in order to be properly charged with this crime? A Negligently B Recklessly C Knowingly or willfully D Purposely

Answer choice B is correct. Under the Model Penal Code, if the requisite mens rea is not stated in a criminal statute, it is established if the defendant acted at least recklessly. Consequently, under the statute here, an individual must act at least recklessly in causing bodily injury to another. Answer choice A is incorrect. The hierarchy of the levels of culpability from the lowest to the highest is negligently, recklessly, knowingly or willfully, and purposely. Although when a particular level of culpability applies to a statute, proof of a greater level of culpability is permitted, proof of a lower level of culpability is not. Because negligently is the lowest level of culpability, and the requisite level of culpability for this offense is at least recklessly, negligently is not an acceptable level of culpability for this offense. Answer choice C is incorrect. When a particular level of culpability applies to a statute, proof of a greater level of culpability satisfies the required level of culpability. Consequently, although the individual could properly be charged with violation of this statute if the individual acted knowingly or willfully, this is not the lowest level of culpability that the individual must possess in order to properly be charged with this crime. Answer choice D is incorrect. Similarly, although the individual could properly be charged with violation of this statute if the individual acted purposely, this is not the lowest level of culpability that the individual must possess in order to properly be charged with this crime.

good question about felony murder - note - the underlying crimes merge with murder charge during the felony murder to just be "felony murder" A convenience store clerk was complaining about his financial troubles to his best friend. The friend said that the clerk's employer had been cheating him out of a decent salary for too many years, and that the employer owed the clerk. The friend suggested that if the clerk robbed the store during another clerk's shift, he would never get caught. The friend offered to loan the clerk his gun to use to scare the clerk on duty. Both men agreed that no one would get hurt in the process. The next day, the clerk carried out the plan to rob the store while the friend waited outside in the car. During the robbery, the clerk accidentally discharged the gun, and a customer was shot and died instantly. The clerk panicked and left the store empty handed. The friend drove the clerk back to his mother's house, told him to lay low, and then drove home. The clerk later decided that he needed to get out of town quickly. He stole his mother's car, which was more reliable than his own, to drive to a nearby state.In a case against the friend, which of the following charges would most likely be successfully prosecuted? A Attempted robbery, murder, and larceny. B Felony murder. C Attempted robbery only. D Attempted robbery and murder only.

Answer choice B is correct. Under the majority and MPC rule, an accomplice is a person who, with the purpose of promoting or facilitating the commission of the offense, aids or abets a principal prior to or during the commission of the crime. The accomplice's assistance to the principal may be verbal encouragement, financial assistance, or physical assistance, provided that the accomplice has the requisite intent to encourage or assist in the commission of the crime. An accomplice is responsible for the crime to the same extent as the principal. If the principal commits crimes other than the crimes for which the accomplice has provided encouragement or assistance, an accomplice is liable for the other crimes if they are the natural and probable consequences of the accomplice's conduct. In this case, the friend aided and abetted the clerk in the robbery attempt, and thus would be liable for that crime. Under the felony murder rule, the accidental shooting of the customer would be deemed murder. Because the shooting was a natural and probable consequence of the friend's conduct, the friend would be liable for murder as well. The underlying felony will generally merge into the crime of felony murder. Thus, the friend will be successfully prosecuted for felony murder. Answer choice A is incorrect because although the clerk would likely be convicted of larceny for taking his mother's car, this action was not a natural and probable cause of the friend's conduct. Thus, the friend would not likely be held liable for larceny. Answer choice C is incorrect because the friend would be held liable for the shooting of the customer. Answer choice D is incorrect because the underlying felony of attempted robbery would merge into the felony-murder conviction. Friend cannot be convicted of both attempted robbery and murder.

FILL IN THE BLANK. Necessity is a defense available in response to ____________. A deadly force B natural force C nondeadly force D any force that the defendant believed would cause imminent bodily injury or death

B

FILL IN THE BLANK. When a defendant acts with the intent to cause harm to one person and that act directly results in harm to another person, the defendant can be liable for the harm caused under the doctrine of ___________________________. A Double jeopardy B Transferred intent C Merger D Vicarious liability

B

From across the street, a woman watched as a man exited a bank with a drawn gun and a bag. As she continued walking home, she realized that she knew the man. From subsequent news reports, she confirmed her suspicions that the man had robbed the bank and learned that the police were requesting that anyone with information about the robbery contact the police. The woman did not contact the police.Is the woman an accessory after the fact to the robbery? A Yes, because she witnessed the man leave the scene of the crime and was subsequently aware that the man had committed a felony. B Yes, because, by opting not to inform police of the man's identity, she effectively aided the man in avoiding apprehension. C No, because she did not help the man to escape arrest or conviction. D No, because she did not aid the man in the commission of the robbery.

Answer choice C is correct. An accessory after the fact is a person who aids or assists a felon in avoiding apprehension or conviction after commission of the felony. An accessory after the fact must know that a felony was committed, act specifically to aid or assist the felon, and give the aid or assistance for the purpose of helping the felon avoid apprehension or conviction. Here, the woman did not affirmatively act to hinder the man's capture or arrest or otherwise to give aid to the man, and did not act intending to help him. Answer choice A is incorrect because knowledge that a felony has been committed and that a particular person has committed the felony is not sufficient make a person an accessory after the fact. Answer choice B is incorrect because the mere failure to give information regarding a felony is not sufficient to make a person an accessory after the fact to the felony. Answer choice D is incorrect. A person who gives aid to a felon during the commission of the felony is a principal in the second degree, not an accessory after the fact. Although the woman was present as the man fled from the bank, she did not affirmatively aid the man in the felony or his escape from the scene.

A man had spent the afternoon at the beach with his girlfriend; each of them had consumed a significant quantity of alcohol. The man saw one of the town's wealthiest residents arrive at the beach, spread out a beach towel, put down a large cloth bag on the towel, drop what looked like a wallet into the bag, and run into the ocean. The man recounted his observations to his girlfriend regarding the wealthy man's actions. She didn't respond, but walked over to the bag and opened it. The bag did not contain the man's wallet, but it did contain an expensive ring. She took the ring, and closed the bag. The wealthy man's companion, who was just coming onto the beach, observed the girlfriend's action. She was arrested. Soon thereafter, the man was arrested as well. The man is charged, among other crimes, with conspiracy to commit larceny. The applicable jurisdiction has adopted a conspiracy statute based on the Model Penal Code.Which of the following is the weakest argument that the man can advance in defense of the charge? A The man's conversation with his girlfriend was inadequate to form a conspiracy agreement. B The theft of the ring was beyond the scope of the conspiracy. C Since the bag did not contain a wallet, it was impossible for the man or his girlfriend to have taken it. D The man's intoxicated state prevented him from forming the intent necessary to commit the crime.

Answer choice C is correct. Factual impossibility is not a defense to conspiracy. Answer choice A is incorrect because, even though the jurisdiction has adopted a unilateral approach to conspiracy, in order for a defendant to be convicted of conspiracy, the defendant must agree to commit an unlawful act even if the other party does not. Here, the man's mere recounting of his observations would likely be insufficient to establish such agreement. A conspirator can be convicted of both the offense of conspiracy and all substantive crimes committed by any other co-conspirators acting in furtherance of the conspiracy. Answer choice B is incorrect because, here, the man could argue that the theft of the ring was an independent crime, and was not in furtherance of the conspiracy to steal the man's wallet. Answer choice D is incorrect because conspiracy is a specific intent crime for which voluntary intoxication may be a defense.

A manager runs a soup kitchen for malnourished children whose parents cannot afford to provide their children with freshly cooked, nutritious meals. So that the soup kitchen can remain financially viable, the manager often accepts free ingredients from a local food bank. One day, the manager received 400 pounds of potatoes from the food bank. He had very few other ingredients on hand that day, so he decided to serve a healthy potato soup for the children. When he started opening the potato bags, he noticed that a majority of the potatoes had green eyes and greenish skin. When he gave the potatoes to his volunteer chef, the chef mentioned that green potatoes were unsafe and potentially toxic, especially when fed to children. The manager brushed her off, and said that the green potatoes were just unripe. The volunteer chef refused to cook with the green potatoes, so she left and the manager made the potato soup by himself. A few weeks later, many of the children who ingested the potato soup ended up in comas. The doctors were able to trace the cause of the comas back to the toxic green potatoes. The doctors saved all but two of the children, who never came out of the comas and eventually died.What is the most serious offense of which the manager can be properly convicted? A Murder B Voluntary manslaughter C Involuntary manslaughter D Negligence or no form of criminal homicide

Answer choice C is correct. Involuntary manslaughter is an unintentional homicide committed with criminal negligence or during an unlawful act. Criminal negligence is grossly negligent action that puts another person at a significant risk of serious bodily injury or death. It requires more than ordinary negligence for tort liability and something less than the extremely negligent conduct required for depraved-heart murder. Here, it was grossly negligent for the manager to cook with the green potatoes, especially when the volunteer chef warned him that the green potatoes were potentially toxic when fed to children. Answer choice A is incorrect because common-law murder is the unlawful killing with malice aforethought, which includes a reckless indifference to an unjustifiably high risk to human life (depraved heart). There is a split among jurisdictions as to whether the requisite depravity exists when a defendant is actually unaware of the risk involved in the conduct, but the majority of states and the MPC impose liability only when the defendant actually realizes the danger. Here, the manager's conduct did not rise to the level of reckless indifference because although he was grossly negligent by cooking the green potatoes, he believed that they were just unripe. Thus, he lacked the malicious intent necessary to be found guilty of murder. Answer choice B is incorrect because voluntary manslaughter involves an intentional killing committed under mitigating circumstances. Here, there was no intent to kill. Answer choice D is incorrect because the manager's conduct was greater than mere civil negligence.

An employee at a fast food restaurant was held up at gunpoint by two criminals. The criminals ordered the employee to empty the cash register. The employee did so, and handed over $55 in cash to them. The criminals took the money and ran off before the police arrived. After they left, a restaurant customer immediately approached the employee to see if she was all right. The employee was visibly shaken, but she reached under the counter, pulled out a bank deposit envelope, set it directly in front of her on the counter, and told the customer that luckily she had recently emptied the cash register and put it into the envelope. The customer grabbed the envelope from the counter and ran out of the restaurant. The employee was so stunned that she was unable to react. The police eventually caught the customer, and charged him with robbery.Is the customer guilty of robbery? A Yes, because the customer took the envelope from the employee with the intent to permanently deprive him of it. B Yes, because the employee was already intimidated from the first incident of robbery when the customer took the envelope. C No, because the customer did not use force or intimidation when he grabbed the envelope from the employee. D No, because the customer did not take the envelope from her person, just grabbed it off of the counter.

Answer choice C is correct. Robbery is larceny from the person or presence of the victim by force or intimidation. Larceny is the trespassory taking and carrying away of the personal property of another with the intent to permanently deprive the person of the property. Here, the grabbing of the bank deposit envelope does not qualify as force or intimidation. The customer grabbed the envelope so quickly that the employee could not react, and the employee was not touching the envelope, so there was neither force nor intimidation needed to take it. Note that this does not mean the customer would not be guilty of any theft crime, just not the crime of robbery. Answer choice A is incorrect because although the customer had the intent to permanently deprive the employee of the envelope, this is just one prong of the rule for robbery. There is no force or intimidation, so the employee cannot be guilty of robbery. Answer choice B is incorrect because the force used by other parties (i.e., the criminals) does not satisfy the requirement of force or intimidation for the customer to be found guilty of robbery. Answer choice D is incorrect because grabbing the envelope off of the counter, in the employee's presence, qualifies as a taking.

One afternoon, a woman was walking down the street when a man walked up behind her, grabbed her purse, and ran away. That evening, as the woman and her husband dined at a restaurant, she recognized an identical purse hanging from the back of a chair upon which another woman was sitting at a table a few feet away. She pointed the purse out to her husband before going up to the table to confront the couple sitting there. The couple claimed that the purse was certainly not stolen and refused to give it to her, so the woman violently yanked it from the chair, knocking it and the woman sitting on it over, and returned to the table with her husband. Robbery charges were subsequently brought against her, and during trial, evidence indicated the purse did actually belong to her. Is the woman likely to be successfully convicted of robbery? A Yes, because she stole the property at night. B Yes, because she took the purse by force. C No, because the purse had been stolen from her earlier in the day. D No, because she took the purse from the chair, not the other woman.

Answer choice C is correct. Robbery is larceny from the person or presence of the victim by force or intimidation. The force used by the defendant must be more than the amount necessary to effectuate taking and carrying away the property. Here, the woman did use force to take the property from another, with the intent to permanently deprive her of that property, but because the property was hers to begin with, she did not take property that belonged to another. Answer choice A is incorrect because robbery need not occur at night. The answer alludes to the requirements for common law burglary. Answer choice B is incorrect because while the woman did use force to obtain the purse, that is only one element of robbery, not all of which are met here. Answer choice D is incorrect because, while the force need not be applied to the actual person if the other elements of robbery were met, those requirements were not met here.

An older man was corresponding daily with a 13-year-old girl in an online chat room called Minors for Men. The older man knew that the girl was a minor because they discussed her age and the trials and tribulations of being a teenager during their many online chats. In addition, the girl sent a picture of herself to the older man and it was clear that she was a minor. The two agreed to meet, and they ended up having sexual intercourse. The older man's wife discovered the relationship when she was looking through his Internet history. She reported him immediately to the authorities and the older man was arrested. The girl found out, and revealed to the authorities that she was actually 23 years old, but she looked very young and she enjoyed role-playing.Can the older man be found guilty of attempt to commit statutory rape? A No, because it was factually impossible for the older man to commit statutory rape of a 23-year-old woman. B No, because it was legally impossible for the older man to commit statutory rape of a 23-year-old woman. C Yes, because the older man sincerely believed that the girl was 13 years old. D Yes, because mistake of fact is a defense to statutory rape.

Answer choice C is correct. Statutory rape is sexual intercourse with a person under the age of consent. The man intended to have sexual intercourse with a minor. Impossibility is not a defense to attempt if the crime attempted is factually impossible to commit due to circumstances unknown to the defendant. If the girl had actually been only 13 years old, then there would have been a crime committed. Further, the MPC crime of attempt (as well as many jurisdictions) requires the performance of a substantial step to support a conviction of attempt, which the man performed here. Answer choice A is therefore incorrect. Answer choice B is incorrect because legal impossibility does not apply to this situation. Legal impossibility is a defense if the act intended is not a crime; the defendant cannot then be guilty of attempt. Statutory rape is a crime, so the older man is guilty of attempt. Answer choice D is incorrect because statutory rape is a strict-liability crime with respect to the age of the victim. Thus, a defendant's reasonable mistake of fact concerning the victim's age is not a defense.

good example of statutory mens rea requirement question By federal statute, it is unlawful to knowingly possess any endangered species of fish or wildlife. Any person who violates this statute is guilty of a misdemeanor and is subject to a fine of not more than $50,000 or imprisonment of not more than six months, or both. Congress indicated that the statute should be applied as broadly as possible in order to accord the endangered species maximum protection.Which of the following is likely the minimum that the prosecution must establish in order to obtain a conviction of a defendant found in the possession of an endangered bird? A The defendant knew that the bird in her possession was a member of an endangered species. B The defendant knew that there was a federal statute that made possession of the bird a crime. C The defendant knowingly possessed the bird. D The defendant possessed the bird.

Answer choice C is correct. The crime in question is a regulatory offense. Such offenses, particularly when public health or safety is concern, are usually strict-liability offenses that do not require a specific mens rea. Here, however, the wording of the statute specifically requires that the violator act knowingly. Therefore, the prosecution must establish that the defendant knowingly possessed the bird in order to obtain a conviction. Answer choice A is incorrect because a requirement that the violator have knowledge that the animal possessed was on the endangered species list would undermine the Congressional intent that the statute be applied as broadly as possible. Answer choice B is incorrect because ignorance of the law is rarely a valid defense to the enforcement of a criminal statute, and the prosecution need not prove that the defendant knew about the regulation. Answer choice D is incorrect because mere possession of the bird is insufficient to establish culpability (i.e., a mens rea) on the part of the defendant under the statute, which requires the defendant act "knowingly."

A gardener entered a garden center through a side entrance during regular business hours. He intended to steal some heirloom parsley plants for his garden. He noticed a sales display of regular parsley plants at the back of the store, but mistakenly believed they were a very valuable type of heirloom parsley. When no one was looking, he took two small parsley plants from the sales display, put them under his sweatshirt, and left the store. At the register, there was a sign that advertised, "Like parsley? We like it so much that we grew more than we need this season! Ask about our overstock! Today only, we are offering each customer two free parsley plants from the surplus in our greenhouse!" The gardener never saw the sign. Burglary in the jurisdiction is defined as "entering any building unlawfully with the intent to commit a crime." The jurisdiction follows the common-law definition of larceny.The following crimes are listed in descending order of seriousness. Which is the most serious crime of which the gardener may be convicted? A Burglary and larceny B Burglary only C Larceny only D Attempted larceny only

Answer choice C is correct. The gardener is guilty of larceny because he took and carried away the property of the garden center with the intent to steal. Mistake of fact is only a defense if it negates the defendant's state of mind. Therefore, the gardener's mistaken belief that he was stealing a more valuable kind of heirloom parsley is not a defense because it did not negate his intent to steal. Additionally, even though surplus parsley plants from the greenhouse were being given away for free, the plants on display in the store were not. Therefore, the gardener has committed a larceny regardless of whether he saw the sign because he was unlawfully taking the property of the garden center with the intent to permanently deprive the store of the parsley plants. He is not guilty of burglary because there was no unlawful entering as required under the statute. The gardener entered the garden center lawfully as an invitee at a time when the garden center was open to the public. Thus, answer choices A and B are incorrect. Answer choice D is incorrect because, although the gardener did commit the crime of attempted larceny by taking a substantial step towards the completion of the crime of larceny, the crime of attempted larceny is a less serious crime than the completed crime of larceny.

A teenage girl wanted to be popular in high school. A group of "mean girls" approached her and asked her to join their clique. The teenage girl quickly agreed. That weekend, the mean girls held a party at which they told the teenage girl that she had to kill their math teacher to be in the group. The teenage girl refused and said she did not want to be in the clique after all. The mean girl leader then said to the other members, "She's already in, right? We can't let her out. We'll kill her if she tries to leave." The other members all orally agreed. On Monday morning, the teenage girl put poison in the math teacher's coffee cup, and he died while explaining quadratic equations.The teenage girl is charged with deliberate and premeditated murder.If the teenage girl establishes that she killed the math teacher because the mean girls threatened to kill her, should she be convicted? A No, because she was forced to act under duress. B No, because the killing of the math teacher was a legal necessity. C Yes, because duress is never a defense to intentional murder. D Yes, because the mean girls threatened the teenage girl with future, not immediate harm.

Answer choice C is correct. The teenage girl is guilty of deliberate and premeditated murder because she killed the math teacher intentionally and with premeditation. She cannot claim duress because duress is never a defense to intentional murder. Answer choice A is incorrect. Even though the teenage girl was coerced into committing the crime, duress is never a defense to intentional murder. Answer choice B is incorrect because the defense of necessity only applies when natural, non-human forces (e.g., weather) require the defendant to commit the criminal act. Answer choice D is incorrect because it refers to the requirements of self-defense. Self-defense does not excuse the use of deadly force against a third party in lieu of an actual aggressor. Therefore, even a threat of immediate harm by the mean girls would not defend the teenage girl's violence against the teacher.

Regarding attempt, which of the following is FALSE? A Attempt is a specific intent crime. B Attempt is a malice crime if the targeted crime is a malice crime. C Attempt merges into the completed crime. D Attempt requires taking a substantial step towards perpetrating the crime.

B

Regarding embezzlement, which of the following is TRUE? A The property embezzled does not need to belong to another person. B The embezzler needs to be in lawful possession of the property at the time of the embezzlement. C Embezzlement occurs when a person obtains title to someone else's property through an act of deception. D A person that is guilty of embezzlement is also guilty of larceny.

B

Regarding the agreement required for a conspiracy, which of the following is FALSE? A An agreement may be implicit as well as explicit. B All conspirators must know of, and agree with, each other. C Under the common law, at least two persons had to agree to commit an unlawful act. D Under the Model Penal Code, a person who agrees to a commit an unlawful act with a government agent can be criminally liable for conspiracy even though the agent lacked the subjective intent to agree to commit the unlawful act.

B

An attorney represented a client in a legal battle over a valuable necklace that had belonged to the client's deceased grandmother. The attorney told the client, who had possession of the necklace, that the client was legally required to leave the necklace with the attorney until the legal issues were resolved. In fact, there was no such requirement. Rather, the lawyer intended to sell the necklace and retire on a small island where the lawyer believed she would never be found. After the client gave the necklace to the lawyer, the lawyer sold the necklace to a jeweler. The jeweler, who had known the grandmother, later recognized the necklace as the grandmother's, and he called the police. The attorney was arrested at the airport later that day.The attorney is guilty of which of the following crimes? A Embezzlement. B False pretenses. C Larceny. D Larceny by trick.

Answer choice D is correct. A defendant is guilty of larceny by trick if she obtains possession (but not title) to property owned by another through fraud or deceit, with the intent to permanently deprive the victim of that property, resulting in the conversion of the property. In this case, the attorney gained possession of the necklace through fraud with the intent to sell it, which she did. Answer choice A is incorrect because embezzlement requires that a defendant have lawful possession of the property of another when forming the intent to defraud. In this case, the defendant formed the intent to defraud her client before she had possession of the property. Answer choice B is incorrect because the client did not pass title to the necklace, as required for the crime of false pretenses. Answer choice C is incorrect because larceny requires that the taking of property be without the owner's consent. In this case, the attorney had consent to take the property, although this consent was obtained through fraud.

A daughter was homeschooled by her bohemian parents for most of her life. The parents did not believe in mainstream medicine, so they taught the daughter about homeopathic remedies as part of her homeschooling. She helped her parents with their extensive garden as part of her daily chores. Unfortunately, the daughter was lonely, she did not believe in homeopathy, and she wanted to eat meat, so she resolved to kill her parents by poisoning them, but making it look like an accident. Amongst the plants she was growing was the castor oil plant, which her parents used to treat a variety of skin conditions and to stimulate the immune system. However, the seeds of the castor oil plant contain the toxin called ricin, which is deadly if ingested by humans. The daughter extracted the seeds from a castor oil plant, crushed them, and then added the ricin powder to her parents' oatmeal. Unbeknownst to the daughter, she had actually picked the wrong plant, known as the "false castor oil plant," which was similar in appearance, but did not produce poisonous seeds. After the parents ate the oatmeal laced with the harmless seeds, they suffered some gastric distress. They went to the garden to find an herbal remedy, and noticing that all of the false castor oil plants were missing, they suspected that someone untrained in plant species tried to poison them, so they called the police. The police were able to piece together what actually happened, and they arrested the daughter for attempted murder.Could the daughter be found guilty of attempted murder? A No, because factual impossibility is a defense to the crime of attempted murder. B No, because it was legally impossible for the daughter to kill her parents. C Yes, because mistake of fact is not a defense to the crime of attempted murder. D Yes, because factual impossibility is not a defense to the crime of attempted murder.

Answer choice D is correct. A factual impossibility occurs when, at the time of the attempt, the facts make the intended crime impossible to commit although the defendant is unaware of this when the attempt is made. However, factual impossibility is not a defense to the crime of attempt. Thus, although it was factually impossible for the daughter to poison her parents with the innocuous seeds from the false castor oil plant, she is still guilty of attempted murder, because if the facts had been as she believed them to be, she would have committed a crime. For this reason, answer choice A is incorrect. Answer choice B is incorrect because legal impossibility is not applicable to this fact pattern. If the act intended is not a crime, then the defendant is not guilty of attempt. Here, the act intended by the daughter is murder—which is a crime, so legal impossibility does not apply. Answer choice C is incorrect. A mistake of fact is a defense to a specific-intent crime, even if the mistake is unreasonable, because it negates the element of mens rea. However, in this case, the daughter's mistake actually prevented her parents from being killed, so mistake of fact does not apply to this situation.

A daughter was homeschooled by her bohemian parents for most of her life. The parents did not believe in mainstream medicine, so they taught the daughter about homeopathic remedies as part of her homeschooling. She helped her parents with their extensive garden as part of her daily chores. Unfortunately, the daughter was lonely, she did not believe in homeopathy, and she wanted to eat meat, so she resolved to kill her parents by poisoning them, but making it look like an accident. Amongst the plants she was growing was the castor oil plant, which her parents used to treat a variety of skin conditions and to stimulate the immune system. However, the seeds of the castor oil plant contain the toxin called ricin, which is deadly if ingested by humans. The daughter extracted the seeds from a castor oil plant, crushed them, and then added the ricin powder to her parents' oatmeal. Unbeknownst to the daughter, she had actually picked the wrong plant, known as the "false castor oil plant," which was similar in appearance, but did not produce poisonous seeds. After the parents ate the oatmeal laced with the harmless seeds, they suffered some gastric distress. They went to the garden to find an herbal remedy, and noticing that all of the false castor oil plants were missing, they suspected that someone untrained in plant species tried to poison them, so they called the police. The police were able to piece together what actually happened, and they arrested the daughter for attempted murder.Could the daughter be found guilty of attempted murder? A No, because factual impossibility is a defense to the crime of attempted murder. B No, because it was legally impossible for the daughter to kill her parents. C Yes, because mistake of fact is not a defense to the crime of attempted murder. D Yes, because factual impossibility is not a defense to the crime of attempted murder.

Answer choice D is correct. A factual impossibility occurs when, at the time of the attempt, the facts make the intended crime impossible to commit although the defendant is unaware of this when the attempt is made. However, factual impossibility is not a defense to the crime of attempt. Thus, although it was factually impossible for the daughter to poison her parents with the innocuous seeds from the false castor oil plant, she is still guilty of attempted murder, because if the facts had been as she believed them to be, she would have committed a crime. For this reason, answer choice A is incorrect. Answer choice B is incorrect because legal impossibility is not applicable to this fact pattern. If the act intended is not a crime, then the defendant is not guilty of attempt. Here, the act intended by the daughter is murder—which is a crime, so legal impossibility does not apply. Answer choice C is incorrect. A mistake of fact is a defense to a specific-intent crime, even if the mistake is unreasonable, because it negates the element of mens rea. However, in this case, the daughter's mistake actually prevented her parents from being killed, so mistake of fact does not apply to this situation.

While climbing a steep mountain, a woman's climbing rope failed, and she fell off the side of the mountain. She landed on a small ledge 30 feet below. Behind her, a man continued climbing. The woman shouted, "Hey! I'm here! Throw me a rope so I can get up!" The man, who did not know the woman but had all the gear that would be necessary to safely rescue her, looked down at her and said, "Sorry, I'd like to help, but I have to reach the summit before dark." He continued on to the summit. On the way back down, he looked down to see that the woman had fallen off the ledge to her death.What is the most serious crime for which the man can be convicted? A Murder. B Voluntary manslaughter. C Involuntary manslaughter. D No crime.

Answer choice D is correct. A legal duty to act and the failure to do so results in criminal liability in certain instances, such as when there is a special relationship between the parties (e.g., a parent's duty to her child) or when a party fails to aid the victim after causing the victim's peril. When there is no duty to act, a defendant is not criminally liable just because he fails to help others in trouble. In this case, the woman and the man do not have a special relationship, as it appears they are strangers. Neither did the man cause the woman to fall down the side of the mountain onto the small ledge or voluntarily assume to render aid. Thus, the man had no legal duty to act. Although the woman died and the man could have prevented her death, he is not liable for her death. Answer choices A, B, and C are therefore incorrect.

Under which of the following circumstances can a state NOT prosecute an act? A When the act occurred outside the state and the act constituted an attempt to commit a crime inside the state B When the whole crime is committed outside the state C When only part of the crime occurred inside the state D When there was a conspiracy to commit a crime and an overt act occurred within the state

B

A man planned to assault the female manager of a convenience store by hiding in the store's bathroom just before closing time and then attacking her after she locked up for the night. On the chosen night, the man happened to enter the store as the last customer was exiting. The customer held the door open and the man walked through the open door. The man hid in the bathroom, but, having a change of heart, exited the store when he heard the manager announcing, "Closing time." On his way out of the store, he took a candy bar and shoved it in his pocket.Burglary is defined by statute as "unlawfully entering a building or occupied structure with the purpose of committing a felony therein."The man is charged with burglary. Of the following, which is likely his best defense? A He did not have the intent to steal the candy bar until he was leaving the store. B He did not break into the store. C He did not commit the planned assault. D The store was open to the public.

Answer choice D is correct. As defined by the statute, burglary requires that the entry into the building or occupied structure be unlawful. Here, because the man entered the store during normal business hours, he can argue that as a member of the public he had the implied consent of the store owner to be on the premises and thus his entry was not unlawful. Answer choice A is incorrect. Although the man did not form the intent to steal the candy bar until he was exiting the store, he nevertheless entered the store with the intent of committing a felony—assault. (Note: In addition, the theft of a candy bar likely would be treated as a misdemeanor rather than a felony.) Answer choice B is incorrect. Although the man did not break into the store as he entered through a door that was held open by a customer, the statute does not require a defendant to break into a building or occupied structure. Instead, the statute merely requires a person to enter the building or occupied structure, which the man did. Answer choice C is incorrect because this burglary statute does not require the defendant to complete the intended felony, but only to enter the building or occupied structure with the purpose of doing so. Here, the man entered the store with the purpose of assaulting the female manager, which would be sufficient.

A retailer sponsored a game that required individuals to collect game pieces in order to win various monetary prizes. An employee of the retailer embezzled the rare game pieces necessary for the significant monetary prizes. The employee then separately contacted various individuals, each of whom was given a rare game piece. The employee did not disclose and the individuals did not learn that the employee gave game pieces to anyone else. Each individual then submitted the necessary game pieces, including the rare game piece provided by the employee, to the retailer for a monetary prize. In order to obtain a prize, each individual falsely stated that he had obtained all of the game pieces through authorized, legitimate channels. Upon receipt of the prize money, each individual paid 50 percent of the money to the employee, as previously agreed.Among other offenses, the employee and all of the individuals were charged with entering into a single conspiracy to obtain money from the retailer by false pretenses.Can the defendants properly be convicted of this conspiracy? A Yes, because each individual falsely stated that he had obtained all of the necessary game pieces through authorized, legitimate channels. B Yes, because each individual shared 50 percent of the prize received from the retailer with the employee. C No, because only the employee embezzled the games pieces from the retailer. D No, because the individuals did not know of the participation of the other individuals in the scheme.

Answer choice D is correct. At common law, conspiracy is (i) an agreement (ii) between two or more persons (iii) to accomplish an unlawful purpose (iv) with the intent to accomplish that purpose. Here, the unlawful purpose was to obtain money from the retailer by false pretenses. False pretenses is (i) obtaining title to the property (ii) of another person (iii) through the reliance of that person (iv) on a known false representation of a material past or present fact, and (v) the representation is made with the intent to defraud. In this case, each individual and the employee had the intent to obtain money from the retailer through the retailer's reliance on each individual's false representation that he had obtained all of the game pieces through authorized, legitimate channels. However, there was not an agreement among all of the individuals to participate in the employee's scheme, but only separate agreements between the employee and each individual to obtain money from the retailer by false pretenses. Consequently, the employee and all of the individuals cannot properly be convicted of entering into a single conspiracy. Answer choice A is incorrect. Although each individual satisfied the requirements for the crime of obtaining money by false pretenses, there was not an agreement among all of the individuals to participate in the employee's scheme, but only separate agreements between the employee and each individual to obtain money by false pretenses. Answer choice B is incorrect. Although the fact that each individual shared 50 percent of the prize received with the employee demonstrates the existence and furtherance of a conspiracy, that conspiracy only existed between the employee and each individual, not between the employee and all of the individuals in a single conspiracy. Answer choice C is incorrect because each person in a conspiracy need not engage in every illegal act necessary for the accomplishment of the purpose of the conspiracy.

A woman broke into her ex-husband's house late one night when she knew he was away on business, intending to take a sculpture that he had been awarded in their divorce settlement. She searched the entire house, but was unable to find the sculpture. She figured he had probably sold the sculpture, which made her furious because he knew how much she loved it. In a rage, she slashed a painting of his new girlfriend before leaving the house.For which of the following crimes is the woman guilty? A Attempted larceny only. B Larceny only. C Burglary only. D Burglary and attempted larceny.

Answer choice D is correct. Burglary is the breaking and entering of the dwelling of another at nighttime with the specific intent to commit a felony therein. At the time of the breaking and entering, the defendant must have the specific intent to commit a felony inside the dwelling. A defendant who fails to commit the underlying felony may nevertheless be guilty of burglary as well as attempt to commit the underlying felony (so long as she took a substantial step toward completing the felony). In this case, the woman broke into her ex-husband's home at night specifically intending to commit larceny. Because she failed to complete the larceny, she is guilty of attempted larceny and burglary. Answer choice A is incorrect because the woman is guilty of burglary as well as attempted larceny. Answer choice B is incorrect because the woman did not commit larceny. Larceny is the trespassory taking and carrying away of the personal property of another with the intent to permanently deprive that person of the property. The destruction of property while in the owner's possession is not a taking. Because the woman did not take anything from her ex-husband, there is no larceny. Answer choice C is incorrect because the attempt to commit the intended felony, in this case larceny, does not merge into the completed crime of burglary. Thus, the woman is guilty of both crimes.

A student in a culinary arts program was extremely upset after receiving poor feedback from his instructor on a meal he had just cooked. The student went to a bar and consumed a few beers. Afterwards, the student purchased some ingredients from a grocery store and returned to his apartment complex. Due to his intoxication, he did not realize that he had actually entered the unlocked apartment of a neighbor rather than his own. While cooking, the student grabbed a bottle of oil mislabeled as vinegar and poured it all over a lit pan on the stovetop. The neighbor's apartment subsequently caught on fire, and the student was arrested and charged with common-law arson.Is the student guilty of common-law arson? A Yes, because the student's mistake was unreasonable. B Yes, because mistake of fact is not a defense to arson. C No, because the student's voluntary intoxication is a valid defense to the arson. D No, because the student did not have the requisite mental state.

Answer choice D is correct. Common-law arson is the malicious burning of the dwelling of another. The crime of arson is a malice crime requiring a reckless disregard of a high risk of harm. Here, the student's use of a mislabeled bottle does not indicate a reckless disregard that creates a substantial risk of fire. Therefore, the student did not have the requisite mental state of malice. Answer choice A is incorrect. A mistake of fact must be reasonable in order to be a defense to a general-intent or malice crime. Although it was an unreasonable mistake for the student to think that his neighbor's apartment was his own, it was not unreasonable to think that a bottle labeled as vinegar was not oil. Answer choice B is incorrect because a mistake of fact can be a defense to a malice crime like arson if the mistake is reasonable. Answer choice C is incorrect because, although voluntary intoxication is a defense to specific-intent crimes if the intoxication prevents the formation of the required intent, it is not a defense to general intent crimes or crimes involving malice, recklessness, negligence, or for strict-liability crimes. Arson is a malice crime. Therefore, the student's voluntary intoxication is not a defense to arson.

Which of the following crimes is NOT a felony on which a felony murder charge can be based? A Rape B Larceny C Kidnapping D Arson

B

Which of the following generally CANNOT constitute the actus reus for a crime? A Failure to comply with a statutory duty B An involuntary act C Failure to act when there is a special relationship between the defendant and the victim D Voluntary assumption of a duty of care that is set aside

B

Which of the following is NOT an element of robbery? A The taking must be from the person or presence of the victim B Taking the real property of another C By force or intimidation D The taking must be trespassory

B

As a car came to a stop at a stop sign, a pedestrian watched as a box fell off the roof of the car. The pedestrian tried unsuccessfully to alert the driver about the box before the car sped away. The mailing label on the box indicated that, although the intended recipient of the box lived out of state, the sender of the box lived only a couple of blocks away. As required by state law with regard to lost property, the pedestrian decided to return the box to its sender. As the pedestrian reached the sender's residence, the pedestrian had a change of heart and decided to keep the unopened box and its contents for herself. Although the outside of the box gave no indication of its contents, there was a controlled substance inside the box. The applicable law provides that knowing possession of a controlled substance is a criminal offense.If the pedestrian were to be arrested before she arrived at her home with the unopened box, with which of the following offenses can she be charged? A Both larceny and possession of a controlled substance B Only larceny C Only possession of a controlled substance D Neither larceny nor possession of a controlled substance

Answer choice D is correct. Larceny is the trespassory taking and carrying away of the personal property of another with the intent to permanently deprive that person of the property. Under the continuing trespass rule, a trespass is deemed to be continuing when the defendant does not possess the necessary intent at the time of the taking but later develops the intent to permanently deprive the owner of the personal property. However, the initial taking of the personal property must have been wrongful. Here, the pedestrian's original taking of the box was not wrongful. State law required the pedestrian to return lost property to its owner, and that was her intent at the time she took possession of the box. Consequently, the taking was not wrongful, and the pedestrian has not committed larceny. Therefore, answer choices A and B are incorrect. Although the pedestrian clearly had possession of the controlled substance, the applicable law requires that her level of culpability must be "knowing." Because the box has not been opened, the pedestrian is not aware of the contents of the box and is therefore unaware that she possesses a controlled substance. Consequently, the pedestrian has not committed the offense of possession of a controlled substance. For these reasons, answer choices A and C are incorrect.

The defendant decided to rob a convenience store. One night, he watched the store from across the street until it appeared that there were no customers inside. He went in, walked up to the clerk who was standing at the cash register, took out a gun, and said, "I will hurt your family if you don't give me the money in the cash register." The clerk immediately fainted. The defendant put away his gun and pulled out a screwdriver to pry open the cash register. At that moment, a customer walked into the store. The defendant immediately fled before being able to open the register.The charges below are listed in descending order of seriousness.On this evidence, what is the most serious charge(s) for which the defendant can be convicted? A Assault and robbery B Robbery C Assault and attempted robbery D Attempted robbery

Answer choice D is correct. Robbery is larceny from the person or his immediate presence by force or intimidation. An attempt is a substantial step towards the commission of a crime coupled with the intent to commit the crime. Here, the defendant entered the store with the intent to steal money from the store's cash register and threatened the clerk who was standing at the register. Consequently, the defendant can be convicted of attempted robbery. However, the defendant failed to complete the crime of robbery because he did not take or carry away the money from the register. Therefore, answer choices A and B are incorrect. Answer choice C is incorrect. Although these facts indicate that the defendant has committed an assault, as he threatened the clerk with the use of force, double jeopardy prevents him from being convicted of this underlying lesser-included offense of robbery. Larceny, assault, and battery all merge into robbery or attempted robbery as lesser-included offenses. Therefore, because the assault here was the threat of force used to accomplish the attempted robbery, an assault conviction would merge into the attempted robbery conviction. Consequently, of the offenses listed, the defendant can only be convicted of attempted robbery.

A man who worked nights was unable to sleep during the day because of the persistent barking of a neighbor's dog. Despite repeated requests to the neighbor to address the problem, the barking persisted. The man decided to purchase a pistol to kill the dog. Unfamiliar with firearms, the man bought a pistol, but accidentally bought blank ammunition. Standing in his own yard, the man fired several shots at the barking dog that was about 10 feet away in his neighbor's yard, thinking he was firing real bullets. The dog was unharmed because the blanks were incapable of inflicting harm. In the applicable jurisdiction, malicious destruction of property is a statutory crime. The highest court of this jurisdiction has ruled that this statutory crime requires the reckless destruction, injury, or defacement of the property of another. Is the man likely to be found guilty of attempted malicious destruction of property? A No, because a reasonable person would have been aware that the blank bullets would not harm the dog. B No, because it was impossible for the man to have killed the dog. C Yes, because the man acted recklessly in shooting at the dog. D Yes, because the man was unaware that the blank bullets would not harm the dog.

Answer choice D is correct. The crime of attempt requires that the defendant act with the specific intent to commit the attempted crime. Here, because the man was unaware that the blank bullets would not harm his neighbor's dog, the man possessed the necessary specific intent to commit the crime of attempted malicious destruction of property. Answer choice A is incorrect. In determining whether a defendant possesses the necessary mens rea, it is the defendant's subjective mental state that is relevant, not the mental state of a reasonable person in the defendant's situation. Answer choice B is incorrect because impossibility is not a defense to an attempt charge if the crime attempted is factually impossible to commit due to circumstances unknown to the defendant. Here, the man's lack of awareness that his ammunition would prevent him from harming the dog does not serve as a defense to the crime of attempted malicious destruction of property. Answer choice C is incorrect. Although the crime of malicious destruction of property requires the defendant to act recklessly, attempt requires the defendant to act with the specific intent to commit the attempted crime. Therefore, the man's recklessness is not determinative of his guilt for attempted malicious destruction of property.

A woman broke into her former lover's house at night with the intent to take back various items of her clothing that the former lover had refused to return to her. After conducting a search and being unable to find the clothing, she came across another woman's clothing. Extremely angry, she took a cigarette lighter and lit the lover's bed on fire, destroying the bed.With which of the following crimes can the woman be properly charged? A Arson. B Burglary. C Both arson and burglary. D Neither arson nor burglary.

Answer choice D is correct. Under the common law, arson is the malicious burning of the dwelling of another. Here, the woman burned only the former lover's bed, not an actual part of the structure of the dwelling. Accordingly, she cannot be guilty of arson and answer choices A and C are therefore incorrect. Answer choice B is incorrect. Common-law burglary is the breaking and entering of the dwelling of another in the nighttime with the specific intent to commit a felony therein. Here the woman did break and enter into the house of another, but not with the intent to commit a felony. Her intent was to recover her own clothing that was being wrongfully detained. Her decision to burn did not arise until after she entered the house, and would therefore not support a finding of burglary.

Which of the following statements is FALSE regarding conspiracy? A A bystander who implicitly agrees to commit a crime is considered a co-conspirator. B The overt act to further a conspiracy must be an unlawful act. C If what the conspirators agree to do is not a crime, then there is no conspiracy. D Under the hub-spoke relationship for conspiracy, the spoke members are not liable for the acts of other conspirators.

B

Which of the following statements regarding involuntary intoxication is FALSE? A Involuntary intoxication is a valid defense to specific intent crimes when it negates the necessary mens rea. B Involuntary intoxication is never a valid defense to general intent crimes. C Involuntary intoxication occurs when a person doesn't realize she's received an intoxicating substance. D Involuntary intoxication occurs when a person is coerced into ingesting a substance.

B

actus reus

B. Actus Reus • No such thing as a "thought" crime. Wanting or hoping to commit a crime is not itself a crime. Must be some physical act involved with crime. 1. There must be some physical act in the world Example 2: A thief shoves a gun into the side of a victim and says, "Your money or your life." The shoving of the gun is the actus reus. o Act can be speech - But so too can speech be actus reus - maybe someones tongue moed and words came out. Example 3: A thief walks up to a victim and says, "Give me $100,000 or I will break your mother's legs next week." The act of speaking is the actus reus there for crime of extortion. 2. Act must be voluntary (i.e., willed by the defendant) o cant be an involuntary act like when d is asleep or cant be involuntary act as in something not under ds mode of control - that does not satisfy the actus reus requirement Example 4: Steve can't get to sleep because he's worried about the bar exam. His doctor recommends that he take Ambien, a popular sleep medication. One of Ambien's side effects is that users sometimes sleepwalk or eat while not fully awake. One night after taking his Ambien, Steve arises from his bed, sleepwalks into the common area of the dormitory, reaches into the refrigerator, and eats Cecelia's jar of caviar. Can Steve be charged with larceny - sealing someone elses property? Answer is no because his taking it wasn't voluntary act. Under influence of prescription drug at time he took the actus reus - so if just hungry and studying and went and stole it that would beactus reus. But because it wasn't under his conscious voluntary control, it doesn't count as actus reus. Example 5: A husband is in bed one night with his wife. In the middle of a dream, the husband rolls over on top of the wife and suffocates her to death. Is the husband guilty of homicide? No. no actus reus because he didnt voluntarily do something. He was asleep. If it actually was someone asleep, then not an actus reus. o "Voluntary" does not necessarily mean the person wanted to do it. It means that he had motor control over the act. Example 6: Tom goes into a store in an attempt to rob the store. He points a pistol at Jerry and says, "Tie up your coworkers or else I'll blow your head off." Jerry, in tying up his coworkers, has committed the actus reus necessary for battery, but he almost certainly will have a defense of duress. Notice not that he didn't commit actus reus - someone who steals food to save starving children. They may have not wanted to do it but they did commit actus reus. So doesn't have to be voluntary in sense that u really wanted to do it.

other crimes - assault

B. Assault (Two Forms) 1. attempted battery - could just have crime of attempted battery but that crime is often referred to as assault o If a defendant has taken a substantial step toward completing a battery but fails, he will be guilty of assault o It is a specific-intent crime because it is an attempt; specific-intent defenses are available. 2. Fear of Harm - other kind of assault o Intentionally placing another in fear of imminent bodily harm o This form of assault is a general-intent crime. Not specific intent crime. Example 72: Thomas has a crush on Ursula. After getting drunk one night, Thomas concludes that Ursula truly loves him. He goes to her room and knocks, but she won't open the door, so Thomas breaks it down. Once inside, he tries to kiss Ursula. But she's fast and he's drunk, so she gets away. Thomas falls over and grabs Isaac—who is also in the room—injuring Isaac's arm. Is Thomas guilty of battery against Ursula for trying to kiss her? Answer is no because he didn't actually kiss or touch her. If he did kiss her without her consent, it would be battery. Is Thomas guilty of attempted battery (assault)? No because if his intoxication prevented him from having the requisite mens rea, attempted battery which is assault is a specific intent crime. So long as the defendant cant maintain the specific intent, he cant have requiste intent for that crime. Did Thomas commit a battery against Isaac? Yes he did. Because that crime is a general intent crime for which his intoxicatin cant provide a defense. And he also committed assault - He also committed assault by breaking down the door because he put them in fear of imminent bodily harm and his voluntary intoxication is not a defense. Remember - For battery or putting someone in fear, those are general intent crimes. Whereas assault with intent to commit battery is a specific intent crime. So defenses applying to these are different from each other.

burglary - breaking into part of store?

Burglary is the breaking and entering of the dwelling of another at nighttime with the specific intent to commit a felony therein. If entry is gained with consent, a breaking can still occur if the defendant breaks into a part of the dwelling structure, such as by opening a closet door or wall safe. The jewelry storeowner gave the three men permission to enter the store, but since the men broke into the jewelry store's wall safe, this qualifies as a breaking.

At common law, which of the following is FALSE regarding the commission of a crime by a child? A A child under the age of seven was irrebuttably presumed to be incapable of committing a crime. B A child at least 14 years old could be charged with and convicted of committing a crime as an adult. C Children between the ages of 7 to 14 were irrebuttably presumed to be capable of committing crimes. D Children between the ages of 7 to 14 were rebuttably presumed to be incapable of committing crimes.

C

FILL IN THE BLANKS. Robbery equals _________ plus _________. A false pretenses, assault B false pretenses, battery C larceny, assault D larceny, battery

C

FILL IN THE BLANKS. The United States can criminalize conduct engaged in by a United States citizen _________. A state can punish its citizens for criminal conduct ________. A only in the United States, that has some connection with the state B only in the United States, anywhere in the world C anywhere in the world, that has some connection with the state D anywhere in the world, anywhere in the world

C

FILL IN THE BLANKS. The _________ test requires that a defendant did not know either the nature of his act or that it was wrong. The _________ test requires that a defendant did not have substantial capacity to appreciate the wrongfulness of his act or to conform his conduct to the law. A Model Penal Code, M'Naghten B Model Penal Code, irresistible impulse C M'Naghten, Model Penal Code D M'Naghten, irresistible impulse

C

Regarding a homicide, which of the following is FALSE? A A homicide is the killing of another human being. B A person is not guilty of homicide if the victim was already dead. C To prove a homicide, the prosecution must only show that the defendant was the actual cause of the victim's death, not the proximate cause. D A person is not guilty of homicide if he kills himself.

C

Regarding battery, which of the following is FALSE? A Consent is a defense to battery. B Battery is a general-intent crime. C The application of force requires an actual physical connection between the defendant and the victim. D A slight touching can constitute force

C

Regarding self-defense, which of the following is FALSE? A A person may use non-deadly force in self-defense anytime he fears an imminent unlawful harm. B A person may generally use deadly force in response to an imminent threat of deadly force. C Under the majority rule, there is a duty to retreat unless it is not safe to do so. D Retreat is not required when the person employing deadly force is in his own home.

C

Which of following is NOT an element of common-law rape? A Unlawful sexual intercourse B Against a female C Against a male D By force or threat of force

C

Which of the following elements does the modern law definition of burglary NOT require? A Structure owned by another B Entering C At nighttime D Specific intent to commit a felony inside

C

Which of the following is NOT an element of kidnapping? A Confinement of the victim B Against the victim's will C Ransom D Either by moving or hiding the victim

C

burglary

C. Burglary 1. Definitions o Common law burglary requires: § Breaking and § Entering § The dwelling § Of another § At night § With the specific intent to commit a felony once inside o The common law elements have been relaxed, so that modern law burglary requires: § Breaking and § Entering § The property § Of another § With the specific intent to commit a felony inside 2. Elements o Breaking can involve pushing open or smashing a door or window, or obtaining entry by fraud. Basically about how you get in. Note 11: Someone who comes inside with the owner's consent and then steals something is guilty of larceny (and possibly robbery if he uses violence or threat of violence) but not burglary because there was no "breaking." No coming in by force. o Entering involves breaking the plane of the dwelling. Example 69: Sticking your hand through a window constitutes "entering." o A "dwelling" at common law is a structure regularly lived in § Modern statutes can also include a commercial building o Of another: Cannot burglarize yourself o With the intent to commit a felony once inside § The usual felony is larceny, but it could be another felony, such as battery. Example 70: Breaking into another's house because of curiosity is not burglary because there is no intent to commit a felony once inside. Example 71: Nick has several friends staying at his beach house. One afternoon, he notices that Mary has an expensive diamond watch. That evening, while everyone is down at the beach for the sunset, he sneaks back to the house, pries open the kitchen window, enters Mary's bedroom, forces open her jewelry box, and steals the watch. Meanwhile, Quinn, Nick's neighbor, notices the half-open window. He's always wanted to see the inside of Nick's house, but Nick has been very standoffish about inviting him in. So Quinn raises the window slightly higher to squeeze into the house. While roaming about the house, he notices Wayne's wallet on the nightstand and takes $250 from the wallet. Nick and Quinn are each charged with burglary. Are either of them guilty of burglary? Nick: nick cant be guilty of burglary because cant burglarize ur own house. Hes guilty of larceny for stealing watch but not burglary. Quinn: not guilty of burglary because didn't enter the house for the purpose of committing a felony - but he will be guilty for larceny he committed once in there.

common law murder

C. Common-Law Murder 1. Definition o Unlawful killing of another human being committed with malice aforethought o Lawful killing of another is not murder (e.g., state execution or a police officer's justifiable use of deadly force). 2. Four Kinds of Malice Intent to kill: The defendant acted with the desire that the victim end up dead. § Intent need not be premeditated; it can be formed in the moment or the secondbefore the killing. Intent to inflict serious bodily harm: The defendant intended to hurt the victim badly, and the victim died. Someone who acts with abandoned or malignant heart or a depraved heart: The defendant acted with a cavalier disregard for human life and a death resulted. § Defendant must realize that his conduct is really risky but need not have any intent regarding the outcome of his actions. He doesn't have to have more than an intent there to be guilty of an abandoned or malignant heart or depraved heart murder. Example 54: Recall the example about dropping bowling balls off an overpass. If u hit someone with balls, chances are ur guilty of abandoned or malignant heart murder. § split among jx about how much depravity you actually have to have to be guilty of an abandoned or malignant heart or depraved heart murder - Majority of states and MPC—impose liability only when defendant actually realizes that there is a danger § Minority position—a reasonable person would have recognized the danger classic example of abandoned or depraved heart murder: Russian roulette Felony murder: The death occurred during the commission or attempted commission of a dangerous felony § The "BARRK" felonies—burglary, arson, robbery, rape, and kidnapping. These give rise to felony murder. Generally not felonies that themselves involve causing death of victim. E.g., aggravated assault that resulted in a death is not felony murder; it is a different type of common law murder. Note 8: The dangerous felony must be independent of the killing itself. § Deaths caused by other felonies get the label of misdemeanor manslaughter. If someone dies in less serious felon that would not give rise to felony murder - may be crime but not felony murder. § Felony murder can involve: a) Someone who resists the felony Example 55: The defendant attempts to rob the victim and the victim resists. The defendant punches the victim and the victim dies. That's felony murder. b) When a bystander is killed during a felony Example 56: The defendant tries to rob a bank and the guard gives chase. The defendant turns around and tries to shoot the guard. Instead, the defendant hits a pedestrian on the street. That's felony murder. c) Third person killed by the resister or police officers (minority) - that counts as felony murder. Example 57: You rob a bank. After robbing the bank, you get into a shootout with the police. If a police officer misses you and shoots a bystander, you are liable in a minority of jurisdictions for felony murder there even tho police officer who did it. But in majority of jx different outcome below: o Majority—agency theory: A defendant is only responsible for the crimes of the defendant's "agents." Because the victim, police, or third party are not the defendant's agents, the defendant is not responsible for their conduct. d) If a co-felon is killed by a resister or a police officer, then the defendant is not guilty of felony murder Example 58: If the defendant is riding with the getaway driver and the police shoots and kills the getaway driver, the defendant is not liable for felony murder. But in minority jx, the so called proximate cause jx, the d might be held liable.

Negating mens rea - intoxication

C. Intoxication • Covers alcohol, drugs, and medications • Can be involuntary intoxication or voluntary intoxication 1. Involuntary intoxication o Occurs when a person: § Doesn't realize that she received an intoxicating substance (e.g., "date rape" drugs); § Is coerced into into ingesting a substance - someone holds u down and makes you drink; or § Has an unexpected or unanticipated reaction to prescription medication. o Involuntary intoxication - Can be a valid defense to general-intent crimes, specific-intent crimes, and malice crimes when it negates the mens rea necessary for the crime so note - involuntary intoxication can negate the existence of mens rea. 2. Voluntary intoxication o Occurs when a person intentionally ingests the substance, knowing it is an intoxicant. If you decide to have a beer for example or decide to pop some pills - knowing theyre an intoxicant for recreitonal use - thars voluntary intoxication. o Voluntary intoxication is a defense only to specific-intent crimes (FIAT crimes), and only if it prevented the defendant from forming the mens rea. Lay aside cases where someone gets durnk in order to commit the crime - because if you get drunk in order to commit the crime to muster up the courage to do it, then its not a defense to say but I was intoxicated at the time of the crime. § Not a valid defense if the defendant got drunk in order to commit the crime Example 36: Terry is a member of a gang. As an initiation rite, she must break into her boyfriend's house and steal one of his mother's shoes. Terry is timid and nervous, so she drinks six shots of bourbon to get her courage up. She's now quite drunk. She then climbs through an open window and begins rummaging around in the mother's closet. She gets one of the mother's shoes and leaves the house. While weaving down the driveway, the mother comes home and confronts Terry. Terry tries to hit the mother over the head with the shoe. But Terry's hand-eye coordination is not so good, so she misses. Terry then jumps into her car and drives off. Unfortunately, she runs over Ursula and kills her. Suppose Terry is charged with burglary, assault, and vehicular homicide. Starting point: She got drunk in order to commit the crime. If she'd simply gotten drunk with friends and then burglarized a house, she could not be convicted of burglary because it is a specific-intent offense. But because she drank in order to commit the burglary, she can be convicted of burglary, even though burglary is a specific intent crime. Assault (with attempt to commit battery) is also a specific-intent crime (it's the A of fiat), but it's not the crime she got drunk in order to commit, so if she didn't have the mens rea because she was drunk, she can't be convicted of assault. Finally, Vehicular homicide is not a specific-intent crime, so she's guilty even though she had no desire to run over Ursula. Fact that she was drunk cannot negate the mens rea. o Under the MPC, voluntary intoxication is only a defense to crimes for which a material element of the crime requires purpose or knowledge and the intoxication prevents the formation of that mental state.

other crimes - rape

C. Rape 1. Common-law rape requires: o Unlawful o Sexual intercourse o With a female o Against her will by force or threat of force Exam Tip 10: The MBE will not likely test on common-law rape because the elements have been relaxed or eliminated in most modern statutes. 2. Most modern rape statutes o Gender-neutral - both men and women can be raped o Require showing lack of consent rather than the force requirement. Far more likely for rape questions today its about modern rape statute and about elements or defenses of that. 3. Intent o Rape is a general-intent crime, not a specific intent crime; so voluntary intoxication by defendant cannot be used as a defense. Fact that d was voluntarily intoxicated or made mistake of fact is not sufficient to negate the elements of the crime. 4. Statutory Rape o Regulatory morals offense that involves consensual sexual intercourse with a person under the age of consent o Statutory rape is a strict liability offense § So long as the defendant knows that he is having sex, he cannot claim ignorance or mistake about the victim's age. No matter how reasonable the mistake might be.

Inchoate Crimes

CATS - Conspiracy, Attempt, Solicitation

Rape

Common-law rape requires: o Unlawful o Sexual intercourse o With a female o Against her will by force or threat of force Exam Tip 10: The MBE will not likely test on common-law rape because the elements have been relaxed or eliminated in most modern statutes. Most modern rape statutes o Gender-neutral - both men and women can be raped o Require showing lack of consent rather than the force requirement. Far more likely for rape questions today its about modern rape statute and about elements or defenses of that. Intent o Rape is a general-intent crime, not a specific intent crime; so voluntary intoxication by defendant cannot be used as a defense. Fact that d was voluntarily intoxicated or made mistake of fact is not sufficient to negate the elements of the crime.

Example 40: Goldie, Frank, and Myrtle all agree to steal goods from a sporting goods store in the mall. Goldie will enter the store to steal the goods, Frank is the lookout, and Myrtle is driving the getaway car. Goldie tells Frank to blow a whistle if a security guard comes by. After Goldie enters the store, Frank panics when he sees a guard and shoves the guard, who falls and breaks his arm. Goldie comes out of the store with the goods, including some starter pistols. Myrtle later uses the starter pistol to rob a convenience store. Who is liable for what?

Conspiracy to rob the store: goldie, frank, and myrtle all liable Battery of the guard: goldie, frank, and myrtle all liable - even though they didn't agree to hit guard because the crime was in furtherance of the conspiracy which was getting away from the store with the robbery goods. Robbery of the convenience store: only myrtle is liable because that wasn't within scope of conspiracy. Conspiracy about robbing store and getting a bunch of stuff - not about what they did with the stuff afterwards.

FILL IN THE BLANKS. For a specific-intent crime, _________ mistakes of fact can be a defense. For a general intent crime, __________ mistakes of fact can be a defense. A only reasonable; only reasonable B only unreasonable; only unreasonable C all; all D all; only reasonable

D

Regarding burglary, which of the following is TRUE? A Entry into the dwelling of another for the purposes of committing a felony is burglary even when the entry is gained with the permission of a resident. B The commission of a felony within the dwelling of another is burglary even when the intent to commit the felony did not arise until after entry. C The defendant must intend to commit a BARRK felony in order to be guilty of burglary. D A person cannot burglarize his own dwelling.

D

Regarding general intent crimes, which of the following is TRUE? A Assault with attempt to commit a battery is a general intent crime. B Acts done purposefully under the Model Penal Code are usually general intent crimes. C Larceny is considered a general intent crime. D General intent crimes require only the intent to perform the act that is unlawful.

D

Regarding the mistake of fact defense, which of the following is FALSE? A A mistake of fact is not a defense to a strict liability crime. B A mistake of fact is a defense to a general intent crime, but only if the mistake is reasonable. C A mistake of fact is a defense to a specific intent crime. D A mistake of fact is a defense to a specific intent crime, but only if the mistake is reasonable.

D

Which of the following is FALSE regarding solicitation? A Solicitation occurs when a person simply asks another to commit a crime. B Solicitation occurs when a person requests another person to commit a crime. C Solicitation does not require that the person solicited agree to commit the crime. D Solicitation does not merge into the completed offense.

D

Which of the following statements regarding common-law arson is TRUE? A Smoke damage constitutes burning. B Burning one's own dwelling constitutes arson. C Arson is a specific-intent crime. D The burning must damage the structure of the building.

D

Which of the following statements regarding merger is FALSE? A Larceny may be merged into robbery. B Attempt may be merged into the completed crime. C Solicitation may be merged into the crime solicited. D Conspiracy may be merged into the crime that was conspired.

D

Homicide

Definition - The killing of a living human being by another human being § Animals cannot commit a homicide and killing an animal is not a homicide - maybe crime but not homicide; § Victim cannot already be dead; § Suicide is not homicide; but assisting someone to commit suicide can be a homicide in jx that forbids assisted suicide.

Manslaughter

Definitions o All unlawful killings of another human being that are not first-degree murder or common law murder o Two types: voluntary manslaughter and involuntary manslaughter

Conspiracy

Definitions and Elements - Common law conspiracy requires: § An agreement - could be tacit or explicit - Agreement can be explicit (I say lets go rob a bank) or implicit (Actions manifest agreement) - Simply knowing that a crime is going to occur and doing nothing about it does not turn a bystander into a co-conspirator; there must be an agreement. § Between two or more people - so cant conspire with yourself; § To commit an unlawful act (aim of agreement is to do something that's a crime) - If what the conspirators agree to do is not a crime, there is no conspiracy even if they think what they're doing is wrong. Modern conspiracy statutes Add a fourth element: The performance of an overt act (physical act) in furtherance of the conspiracy - overt act can be lawful or unlawful, as long as it furthers the conspiracy - overt act deosnt itself have to be a crime, it can be a lawful act as long as that act furthers the conspiracy. At common law agreement itself was crime and nothing more needed. But MPC adds something else which is under Model Penal Code (MPC) - Only the defendant must actually agree to commit the unlawful act. The other people with whom the defendant agrees can be undercover agents, for example.

Summary

Different states of mind both under common law -specific intent crimes (fiat crimes), malice (I AM crimes), general intent, and strict liability. Then also in the model penal code - purpose, knowledge, recklessness, and negligence. So remember those as states of mind - remember that those states of mind can transfer. And finally remember that inchoate crimes and completed crimes merge except for conspiracy.

Example 39: Zacarias Moussaoui and other 9/11 hijackers joined a health club together in Georgia. They did it to build strength so they could overpower the pilots and crew during the hijackings. This is an overt act because it furthered the conspiracy.

Even though joining health club itself is not a crime, thatd be overt act in furtherance of the conspiracy to hijack the plans.

Example 38: Harry and Sally hear that gray-bellied sapsuckers are exotic birds and that someone will pay $10,000 for one of these birds. They know that stealing eggs from endangered species' nests is a crime. Harry and Sally agree to go to the nest of the gray-bellied sapsucker and steal the eggs to sell them. But it turns out that gray-bellied sapsuckers aren't endangered, so it's not actually a crime to catch one. So they are NOT guilty of conspiracy, because what they conspired to do was not a crime—even though they thought it was.

Even though they had agreement they thought was unlawful act, its not conspiracy even though they thought it was. Not conspiracy even though they thought it was. Agreeing to sell marijuana in state where marijuana sales are legal wouldn't be conspiracy under state law but still would be under federal law.

. Other Sex Crimes at Common Law

Exam Tip 11: These are extremely unlikely to be tested on the bar exam. o Adultery: Having sex with someone who is not your spouse o Fornication: Sex between unmarried people o Crimes against nature: Bestiality o Incest: Sex between people who are too closely related to one another o Bigamy: Marrying someone while you are still legally married to someone else o Seduction: A man tells a woman that he will marry her if she has sex with him

Example 65: You take the wrong black umbrella from an umbrella stand in a restaurant. Even if you intend to keep that umbrella forever, you are not guilty of larceny because you were mistaken (this is true even if the mistake was unreasonable). So long as you thought it was ur umbrella not larceny, even if unreasonable for you to think that.

Example 65: You take the wrong black umbrella from an umbrella stand in a restaurant. Even if you intend to keep that umbrella forever, you are not guilty of larceny because you were mistaken (this is true even if the mistake was unreasonable). So long as you thought it was ur umbrella not larceny, even if unreasonable for you to think that.

Example 8: On the way into the bar exam, you see a student spill his drink all over himself. You notice that he is about to touch a live wire. If you do nothing and let the student get electrocuted, you cannot be held criminally liable because you didn't create the peril. However, if it were your wire and you should have grounded it, you could be held criminally liable because theres sufficient actus reus.

Example 8: On the way into the bar exam, you see a student spill his drink all over himself. You notice that he is about to touch a live wire. If you do nothing and let the student get electrocuted, you cannot be held criminally liable because you didn't create the peril. However, if it were your wire and you should have grounded it, you could be held criminally liable because theres sufficient actus reus.

Example 9: Nancy is a home healthcare worker who cares for an elderly woman named Mona. One evening, while Nancy's friend Dave is visiting Nancy at Mona's house, Mona begins to choke on her dinner. Dave hears Mona choking. Nancy is in the kitchen listening to her iPod. Dave says, "Hey, that old lady is turning blue and gasping." Nancy decides to wait until the end of a track to check on Mona. Mona chokes to death. Can Dave be charged with any crime related to Mona's death? Can Nancy?

Example 9: Nancy is a home healthcare worker who cares for an elderly woman named Mona. One evening, while Nancy's friend Dave is visiting Nancy at Mona's house, Mona begins to choke on her dinner. Dave hears Mona choking. Nancy is in the kitchen listening to her iPod. Dave says, "Hey, that old lady is turning blue and gasping." Nancy decides to wait until the end of a track to check on Mona. Mona chokes to death. Dave cant be charged with any crime related to monas death because hes just a callous by stander - and callous by standers not generally guilty of any crime. But Nancy can be charged because she had a special relationship to mona - she was her caregiver and failed to give her the appropriate amount of care.

Exception to accomplice liability

Exception: A person protected by a statute itself cannot be an accomplice in violating the statute. Example 27: If a statute prohibits sex with an underage person, the underage person is not an accomplice to that crime. So Accomplice liability is about a theory of liability for someone who aids before or during the commission of a crime. But what about people who assist after the fact? Theyre not guilty of underlying crime - for them separate set of crimes like obstruction of justice or harboring fugitive. People who assist d after crime committed generally not guilty of primary crime but can be punished for one of the secondary crimes.

Four categories of crimes that are specific intent crimes under the common law.

FIAT - 1) First degree murder: not most murders; on the MBE, the question will expressly state if a defendant is charged with first-degree murder - first degree murder is a specific intent crime. 2) Inchoate crimes: "CATS"—conspiracy, attempt, and solicitation 3) Assault with attempt to commit a battery 4) Theft offenses: e.g., larceny, embezzlement, forgery, burglary, and robbery - if taking someones possessions, those theft offenses are also part of fiat and those are the specific intent crimes. Exam Tip 2: The main reason to memorize the FIAT crimes is that there are some defenses—most notably voluntary intoxication and unreasonable mistake of fact—that are available only for specific-intent crimes.

jx and actus reus summary

For any crime, remember need both jx and actus reus. Rules for jx are different for US which can sometimes have extraterritorial jx for crimes occurring wholly outside the us, like on high seas or by US citizens or nationals living abroad. Whereas states can only have jx if some nexus to state itself - either crime occurs in the state, overt act occurs in the state, or criminal actions aimed at the state but take place outside. Actus reus necessary for every crime must be some voluntary physical act in the world or some failure to act in the world.

If an MBE question contains a statute

If an MBE question contains a statute, read it carefully for mens rea language. "With intent to..." = specific intent crime "Knowingly or recklessly...." = general intent crime No mens rea language = Consider if likely to be strict liability crime

DEFENSES - intoxication

Intoxication • Ask two questions: 1) Was the intoxication voluntary or involuntary? 2) Is the charged crime a specific -intent crime or a general-intent crime? o Specific-intent crimes (FIAT or MPC statute with "purposely"): Can use either voluntary or involuntary intoxication as a defense, if the defendant because of intoxication could not maintain the state of mind necessary for the offense o General-intent crimes: Can use only involuntary intoxication as a defense

Insanity

Involves a defendant who, because of a mental disease or defect, is unable to conform his conduct to the law.

accomplice example

Irving decides to rob a bank. He asks Oscar to drive a getaway car. Irving then goes to Paul's house and asks to borrow Paul's gun, telling him he wants to go target shooting. Paul lends him the gun. During the bank robbery, Irving shoots a teller. Oscar takes his share of money from the robbery and buys a pound of heroin. Irving, Oscar, and Paul are all charged with robbery, attempted murder, and possession of heroin with intent to distribute. For which crimes can they be convicted? Irving: irving is guilty of bank robbery - easy because he went into bank and attempted to rob it. Hes principal for that. Also guilty of attempted murder because he shot teller. Another crime hes principal for. But hes not guilty of possession of heroine with intent to distribute because that's not a foreseeable consequence of bank robbery - that was just Oscar doing something on his own. Oscar: guilty of all 3 crimes - guilty of heroine crime as a principal because he bought heroine - actus reus - with intent to distribute, and guilty as an accomplice to attempted murder and bank robbery, even though he didn't commit the actus reus inside the bank and he didn't shoot teller, those were both foreseeable - he acted with intent that the bank be robbed and it was foreseeable that violence would occur in the bank. Paul: paul isn't liable for any crimes - he had no intent to assist in commission of any crimes. Note 4: Irving and Oscar might also be held liable for conspiracy (discussed below). In addition to crimes above, may also be conspiracy between Oscar and irving.

Example 37: Anna lives in a common law jurisdiction. She agrees with Bob to rob a bank, but she doesn't know that Bob is an undercover police officer. Anna also wants to steal an emerald from a jewelry store in a nearby state, which is an MPC jurisdiction. She asks Ranier for help with stealing the gem. She doesn't know that Ranier is also an undercover officer.

Is Anna guilty of conspiracy to commit bank robbery? That's crime where she agreed in a common law jx with bob whos an undercover agent. Answer is no because no actual agreement between 2 people to commit a crime - anna was only person agreeing to commit crime - bob never intended to commit a crime. So common law rule prevented certain kinds of undercover work from undergirding a prosecution. Is Anna guilty of conspiracy to commit larceny? Anna is guilty of this because in MPC jx, only anna has to actually agree. As long as she thinks shes agreeing with another person that's good enough.

It is always foreseeable that...

It is always foreseeable that the victim of a violent crime will have a heart attack.

summary - property crimes

Larceny is like building block for number of other crimes - larceny requires taking property of another without his consent with the intent to keep te property permanently. Then you can add on to that - if its done with violence at time its robbery. If done through threats of violence or other future threats - its extortion. If done after you get the property u steal, its embezzlement rather than larceny. And if you have to break or enter a dwelling in order to commit the larceny, that's burglary. But remember - burglary can occur even with regard to other felonies, just so long as at the time of the entering theres an intent to commit a felony.

Summary of chapter

Larceny is like building block for number of other crimes - larceny requires taking property of another without his consent with the intent to keep the property permanently. Then you can add on to that - if its done with violence at time its robbery. If done through threats of violence or other future threats - its extortion. If done after you get the property you steal, its embezzlement rather than larceny. And if you have to break or enter a dwelling in order to commit the larceny, that's burglary. But remember - burglary can occur even with regard to other felonies, just so long as at the time of the entering theres an intent to commit a felony.

malice

Malice - second state of mind under common law. o I "AM" certain that there are only two malice crimes: arson and murder (everything other than first degree murder which is fiat crime) o Malice exists when the defendant acts in reckless disregard of a high degree of harm. The defendant realizes the risk and acts anyway. Example 11: Arson is the malicious burning of the dwelling of another. Lynn is at Paul's house for a Fourth of July cookout. Some of the fireworks seem to be duds, so Lynn tosses them onto the grill. Some explode and catch Paul's house on fire. Lynn is charged with arson. At trial, she argues that she didn't want to cause Paul's house to burn down. Can Lynn be convicted of arson? Answer is yes, because arson isn't a specific intent crime - it's enough that she acted maliciously - malice means sufficient knowledge of a risk and going ahead anyways. Its enogh lynn was reckless and engaged in conduct with really high risk of harm.

Summary of this chapter

Modern conspiracy is agreement to commit crime and an overt act - common law required 2 to tango but no overt act. The Conspirator is liable for both conspiracy, which is a crime itself, and for any crimes committed in furtherance of the conspiracy and hard to back out or withdraw from a conspiracy. Attempt is a specific intent crime so keep the special defenses for specific intent crimes in mind - it involves taking a substantial step towards the commission of a crime. While attempt merges into the completed offense, conspiracy doesn't.

For any crime, remember

Need both jx and actus reus. Rules for jx are different for US which can sometimes have extraterritorial jx for crimes occurring wholly outside the us, like on high seas or by US citizens or nationals living abroad. Whereas states can only have jx if some nexus to state itself - either crime occurs in the state, overt act occurs in the state, or criminal actions aimed at the state but take place outside. Actus reus necessary for every crime must be some voluntary physical act in the world or some failure to act in the world.

Example 71: Nick has several friends staying at his beach house. One afternoon, he notices that Mary has an expensive diamond watch. That evening, while everyone is down at the beach for the sunset, he sneaks back to the house, pries open the kitchen window, enters Mary's bedroom, forces open her jewelry box, and steals the watch. Meanwhile, Quinn, Nick's neighbor, notices the half-open window. He's always wanted to see the inside of Nick's house, but Nick has been very standoffish about inviting him in. So Quinn raises the window slightly higher to squeeze into the house. While roaming about the house, he notices Wayne's wallet on the nightstand and takes $250 from the wallet. Nick and Quinn are each charged with burglary. Are either of them guilty of burglary?

Nick: Nick cant be guilty of burglary because cant burglarize ur own house. Hes guilty of larceny for stealing watch but not burglary. Quinn: not guilty of burglary because didn't enter the house for the purpose of committing a felony - but he will be guilty for larceny he committed once in there.

Example 66: Jake is a teller at King Savings Bank. One day Lou, a depositor at the bank, comes to the window and hands Jake a check, made out to Zach and endorsed over to Lou, for deposit to Lou's account. Jake knows that Lou is an enforcer for an organized crime family, and strongly suspects that Lou obtained Zach's check through force. Believing the check to be ill-gotten gains, Jake instead deposits the check to the account of the Make-a-Wish Foundation, a local charity. Is Jake guilty of larceny because he stole Lou's check? Is Jake guilty of embezzlement?

No hes not, because he had Lou's permission to have cheque in his hand. Is Jake guilty of embezzlement? Yes because hetook money and kept it. That's true even if money not rightfully lous in first place. You don't get to act as an agent of the law. Even though he didn't keep the money for himself? What about fact that jake didn't keep money for himself? its still embezzlement, even if putting in make a wish account because he deprived lou of his property permanently. Embezzlement adds twist to larceny which is u got property in first place with the victims consent but then converted property to another use.

Example 5: A husband is in bed one night with his wife. In the middle of a dream, the husband rolls over on top of the wife and suffocates her to death. Is the husband guilty of homicide?

No. no actus reus because he didnt voluntarily do something. He was asleep. If it actually was someone asleep, then not an actus reus.

Example 2: A thief shoves a gun into the side of a victim and says, "Your money or your life."

The shoving of the gun is the actus reus.

Example 64: One day before class, you realize that you forgot your textbook. In the law school library, you see someone else's textbook lying on a desk. You borrow the book with the intention of returning it after class. During your class, there is a fire alarm and you leave the book behind. The sprinklers malfunction and start spraying water inside the classroom, ruining the borrowed textbook. Are you guilty of larceny?

No. you didn't intend to keep the textbook permanently.

INCHOATE CRIMES

Note 7: Remember CATS: Conspiracy, Attempt, Solicitation A. Conspiracy 1. Definitions and Elements - Common law conspiracy requires: § An agreement - could be tacit or explicit; § Between two or more people - so cant conspire with yourself; § To commit an unlawful act (aim of agreement is to do something that's a crime). It had 3 pieces to it. Modern conspiracy statutes Add a fourth element: The performance of an overt act (physical act) in furtherance of the conspiracy. At common law agreement itself was crime and nothing more needed. But MPC adds something else which is under Model Penal Code (MPC) - Only the defendant must actually agree to commit the unlawful act. The other people with whom the defendant agrees can be undercover agents, for example. o Agreement can be explicit (I say lets go rob a bank) or implicit (Actions manifest agreement) § Simply knowing that a crime is going to occur and doing nothing about it does not turn a bystander into a co-conspirator; there must be an agreement. Example 37: Anna lives in a common law jurisdiction. She agrees with Bob to rob a bank, but she doesn't know that Bob is an undercover police officer. Anna also wants to steal an emerald from a jewelry store in a nearby state, which is an MPC jurisdiction. She asks Ranier for help with stealing the gem. She doesn't know that Ranier is also an undercover officer. Is Anna guilty of conspiracy to commit bank robbery? That's crime where she agreed in a common law jx with bob whos an undercover agent. Answer is No because no actual agreement between 2 people to commit a crime - anna was only peros nagreeing to commit crime - bob never intended to commit a crime. So common law rule prevented certain kinds of undercover work from undergirding a prosecution. Is Anna guilty of conspiracy to commit larceny? Anna is guilty of this because in MPC jx, only anna has to actually agree. As long as she thinks shes agreeing with another person that's good enough. o also requirement that Purpose of the conspiracy be an Unlawful act § If what the conspirators agree to do is not a crime, there is no conspiracy even if they think what they're doing is wrong. Example 38: Harry and Sally hear that gray-bellied sapsuckers are exotic birds and that someone will pay $10,000 for one of these birds. They know that stealing eggs from endangered species' nests is a crime. Harry and Sally agree to go to the nest of the gray-bellied sapsucker and steal the eggs to sell them. But it turns out that gray-bellied sapsuckers aren't endangered, so it's not actually a crime to catch one. So they are NOT guilty of conspiracy, because what they conspired to do was not a crime—even though they thought it was. Even though they had agreement they thought was unlawful act, its not conspiracy even though they thought it was. Not conspiracy even though they thought it was. Agreeing to sell marijuana in state where marijuana sales are legal wouldn't be conspiracy under state law but still would be under federal law. o Overt act requirement in modern conspiracy statutes: Can be lawful or unlawful, as long as it furthers the conspiracy - overt act deosnt itself have to be a crime, it can be a lawful act as long as that act furthers the conspiracy. Example 39: Zacarias Moussaoui and other 9/11 hijackers joined a health club together in Georgia. They did it to build strength so they could overpower the pilots and crew during the hijackings. This is an overt act because it furthered the conspiracy. Even though joining health club itself is not a crime, thatd be overt act in furtherance of the conspiracy to hijack the plans.

Felony murder expanded

Note 8: The dangerous felony must be independent of the killing itself. § Deaths caused by other felonies besides BARKK felonies (burglary, arson, robbery, rape, and kidnapping) get the label of misdemeanor manslaughter. If someone dies in less serious felon that would not give rise to felony murder - may be crime but not felony murder. § Felony murder can involve: a) Someone who resists the felony Example 55: The defendant attempts to rob the victim and the victim resists. The defendant punches the victim and the victim dies. That's felony murder. b) When a bystander is killed during a felony Example 56: The defendant tries to rob a bank and the guard gives chase. The defendant turns around and tries to shoot the guard. Instead, the defendant hits a pedestrian on the street. That's felony murder. c) (minority) Third person killed by the resister or police officers - that counts as felony murder. Example 57: You rob a bank. After robbing the bank, you get into a shootout with the police. If a police officer misses you and shoots a bystander, you are liable in a minority of jurisdictions for felony murder there even tho police officer who did it. But in majority of jx different outcome below: o Majority—agency theory: A defendant is only responsible for the crimes of the defendant's "agents." Because the victim, police, or third party are not the defendant's agents, the defendant is not responsible for their conduct. d) (majority) If a co-felon is killed by a resister or a police officer, then the defendant is not guilty of felony murder Example 58: If the defendant is riding with the getaway driver and the police shoots and kills the getaway driver, the defendant is not liable for felony murder. But in minority jx, the so called proximate cause jx, the d might be held liable

Example 6: Tom goes into a store in an attempt to rob the store. He points a pistol at Jerry and says, "Tie up your coworkers or else I'll blow your head off." Jerry, in tying up his coworkers, has committed the actus reus necessary for battery, but he almost certainly will have a defense of duress.

Notice not that he didn't commit actus reus - someone who steals food to save starving children. They may have not wanted to do it but they did commit actus reus. So doesn't have to be voluntary in sense that u really wanted to do it.

Example 1: Doug lives in Delaware and is visiting South Carolina for vacation. In South Carolina, he meets Ken from Kansas in a bar. Doug hires Ken to kill Doug's wife, Vicki. Ken goes to Delaware, kidnaps Vicki, and her body is later found in Pennsylvania (where she was killed). Can Ken be convicted of murder in Delaware?

Obviously answer is yes bc part of crime occurred there - kidnapping leading to homicide. Suppose Ken leaves the bar in South Carolina and rents a car to drive to Delaware. Can Ken be prosecuted for conspiracy to commit murder in South Carolina even if no crime occurred there? Note no other crime - because kidnapping happens in pa, not south Carolina. Yes he can be punished for conspiracy - one of the overt acts - renting car used to drive to where hed kidnap victim occurred there - even tho simply renting car wouldn't be a crime, its overt act in furtherance of the conspiracy. Can Ken be prosecuted in Kansas? No. because none of events connected to crime occurred there. Fact he lives there isn't enough to give Kansas jx over him.

principals

Principals • Defendants whose acts or omissions form the actus reus of the crime. • Can be more than one principal to a particular crime. For example, if gang assaults and kills someone, each of members who killed them can be principal to a crime. • Ask: Who committed the actus reus that gives rise to the offense? If answer is this person committed actus reus, then person is a principal.

accomplice guilt...

The sister is guilty as an accomplice because she aided and abetted the woman in the commission of the crimes. An accomplice is responsible for the crimes to the same extent as the principal.

Extortion: A Variation of Robbery

Rather than threatening imminent physical harm or using force, extortion involves threats of future harm (can include threats of future non-physical harm) Example 68: Xenia approaches Chris at a party and admires her necklace. Xenia tells Chris, "Give me that necklace or else I'll tell your husband that I saw you coming out of a hotel room last weekend with Ben." Chris turns to walk away and Xenia grabs the necklace, pulling it off Chris's neck. Of which crimes is Xenia guilty? Robbery: yes because she obtained the necklace by force. Attempted extortion: yes, because she threatened to commit larceny but failed. Extortion: no, she didn't actually obtain the property through the threat. Chris turned to walk away so threat wasn't how xenia got property, but robbery was. Editorial Note 2: This conclusion relies on the minority approach to extortion, which requires both a threat and actually obtaining the property. Under modern statutes, the majority of jurisdictions would find the threat alone to be sufficient for extortion.

states of mind summary

Remember - think about different states of mind both under common law -where specific intent crimes (fiat crimes), malice (IM crimes), general intent, and strict liability. Then also in the model penal code - purpose, knowledge, recklessness, and negligence. So remember those as states of mind - remember that those states of mind can transfer. And finally remember that inchoate crimes and completed crimes merge except for conspiracy.

3. Recklessly - MPC

Requires the defendant to act with a conscious disregard of a substantial and unjustifiable risk that constitutes a gross deviation from the standard of conduct of a law abiding person Example 16: Patricia is a Golden State Warriors fan who lives in downtown Oakland. When the Warriors win their 77th regular season game, she's so excited that she takes her semi-automatic gun out of the house and fires several dozen rounds into the air. Several of the rounds hit Vic. She didn't want to hit him, and she actually didn't know he was standing nearby, but she acted recklessly because a reasonable person would recognize risk that firing a gun into the air in a city would have.

recklessly - third MPC state of mind

Requires the defendant to act with a conscious disregard of a substantial and unjustifiable risk that constitutes a gross deviation from the standard of conduct of a law abiding person Example 16: Patricia is a Golden State Warriors fan who lives in downtown Oakland. When the Warriors win their 77th regular season game, she's so excited that she takes her semi-automatic gun out of the house and fires several dozen rounds into the air. Several of the rounds hit Vic. She didn't want to hit him, and she actually didn't know he was standing nearby, but she acted recklessly because a reasonable person would recognize risk that firing a gun into the air in a city would have.

Kidnapping

Requires: o Unlawful o Confinement of another person o Against that person's will o Either by moving or hiding the victim

Other Sex Crimes at Common Law

These are extremely unlikely to be tested on the bar exam. o Adultery: Having sex with someone who is not your spouse o Fornication: Sex between unmarried people o Crimes against nature: Bestiality o Incest: Sex between people who are too closely related to one another o Bigamy: Marrying someone while you are still legally married to someone else o Seduction: A man tells a woman that he will marry her if she has sex with him

After a woman and her roommate got into a fight, the woman and her boyfriend formulated a plan to get back at the roommate. The woman planned to distract the roommate while the boyfriend snuck into the apartment through the open back door to steal her new camera, which was on the kitchen counter. The woman and her boyfriend headed back to the apartment and put their plan into action. The plan went awry when the roommate heard a noise and went into the kitchen to find the boyfriend with her camera in hand. Panicked, the boyfriend pushed the roommate to the floor and ran out of the apartment with the camera.The boyfriend is most likely to be convicted of which of the following crimes? A Burglary only. B Burglary and battery. C Robbery only. D Robbery and battery.

Robbery is larceny from the person or presence of the victim by force or intimidation. The force need not be great, but must be more than the amount necessary to effectuate taking and carrying away the property. The elements of robbery, including the presence requirement, were satisfied here when the boyfriend pushed down the roommate and ran off with her camera. Answer choices A and B are incorrect because the elements of burglary are not satisfied. Burglary is the breaking and entering of the dwelling of another at nighttime with the specific intent to commit a felony therein. Because the woman was a resident of the apartment, and she let her boyfriend in the apartment, there was no breaking and entering into the dwelling of another. Answer choice D is incorrect because the crime of battery merges into the crime of robbery. Although a battery did occur when the boyfriend pushed the roommate to the floor, the boyfriend could not be convicted of both robbery and battery under the doctrine of merger. Rather, the battery would merge into the more serious crime of robbery.

Involuntary Manslaughter

Someone dies but death not intended by the defendant. o A criminally negligent killing or killing of someone while committing a crime other than those covered by felony murder or "bark" crimes (i.e., misdemeanor manslaughter) o A defendant who engages in criminally negligent conduct and causes a death is guilty of involuntary manslaughter (e.g., traffic deaths). Example 61: Lisa goes jogging with her dog. A city law makes it a misdemeanor to let a dog off its leash on public property. Lisa ignores the law. While unleashed, Lisa's dog attacks and kills a small child. In this situation, Lisa may be guilty of misdemeanor manslaughter, because letting her dog off its leash was the "but-for" cause of the child's death and it was crime to let dog off the leash.

Example 36: Terry is a member of a gang. As an initiation rite, she must break into her boyfriend's house and steal one of his mother's shoes. Terry is timid and nervous, so she drinks six shots of bourbon to get her courage up. She's now quite drunk. She then climbs through an open window and begins rummaging around in the mother's closet. She gets one of the mother's shoes and leaves the house. While weaving down the driveway, the mother comes home and confronts Terry. Terry tries to hit the mother over the head with the shoe. But Terry's hand-eye coordination is not so good, so she misses. Terry then jumps into her car and drives off. Unfortunately, she runs over Ursula and kills her. Suppose Terry is charged with burglary, assault, and vehicular homicide.

Starting point: She got drunk in order to commit the crime. If she'd simply gotten drunk with friends and then burglarized a house, she could not be convicted of burglary because it is a specific-intent offense. But because she drank in order to commit the burglary, she can be convicted of burglary, even though burglary is a specific intent crime. Assault (with attempt to commit battery) is also a specific-intent crime (it's the A of fiat), but it's not the crime she got drunk in order to commit, so if she didn't have the mens rea because she was drunk, she can't be convicted of assault. Finally, Vehicular homicide is not a specific-intent crime, so she's guilty even though she had no desire to run over Ursula. Fact that she was drunk cannot negate the mens rea. o Under the MPC, voluntary intoxication is only a defense to crimes for which a material element of the crime requires purpose or knowledge and the intoxication prevents the formation of that mental state.

Example 3: A thief walks up to a victim and says, "Give me $100,000 or I will break your mother's legs next week."

The act of speaking is the actus reus there for crime of extortion.

False Pretenses: Another Variation of Larceny

The defendant obtains title to someone else's property through an act of deception. Example 67: Paying for goods with counterfeit money or a bad check

Negligently - MPC

The defendant should be aware of a substantial and unjustifiable risk and acts in a way that grossly deviates from the standard of care of a reasonable person in the same situation. But defendant wasnt subjectively aware of the risk.

negligently - fourth MPC state of mind

The defendant should be aware of a substantial and unjustifiable risk and acts in a way that grossly deviates from the standard of care of a reasonable person in the same situation. But defendant wasnt subjectively aware of the risk.

1. Purposely - MPC

The defendant's conscious objective is to engage in the conduct or to cause a certain result Example 14: Patricia raises her gun, points it at Vic, screams "die, you two timing no-good piece of garbage," and fires, killing him. She has acted purposely. Because she intended that he die.

Purposely - first MPC state of mind

The defendant's conscious objective is to engage in the conduct or to cause a certain result Example 14: Patricia raises her gun, points it at Vic, screams "die, you two timing no-good piece of garbage," and fires, killing him. She has acted purposely. Because she intended that he die.

Model Penal Code and other Modern Changes

Under the MPC and in many jurisdictions, crimes such as larceny, false pretenses, and embezzlement are treated as a single statutory crime of theft (which includes both tangible and intangible property). Exam Tip 9: Be sure to note whether a question wants you to use the MPC definition (i.e., all the theft crimes are treated equivalently) or the common law definitions (in which case, you must work your way through the elements).

Transferred Intent Doctrine

When a defendant has the requisite mens rea for committing a crime against Victim A, but actually commits the crime against Victim B, the law transfers the intent from Victim A over to Victim B. Example 17: Ralph is a professional jewel thief. He is hired to steal a valuable diamond pin from Mrs. Rich at a charity ball. In preparation, Ralph studies her picture from old newspapers. However, Mrs. Rich has had a lot of work done on her face since the picture. So at the ball, Ralph approaches Miss Humble, who resembles the picture he studied, believing her to be Mrs. Rich, and steals jewelry from her instead. Can Ralph be charged with larceny—a specific-intent crime—against Miss Humble? Yes he intended to commit larceny and it transfers to ms humble from ms rich. Can he be charged with attempted larceny against Mrs. Rich? Yes, because he attempted to commit larceny against her even though he actually committed larceny against ms humble. Note 2: Transferred intent does not apply to attempted crimes, only to completed crimes.

Example 7: Anna and her friends are on the beach when they see you drowning. Anna shouts, "I'll save you!" She starts swimming out to you when she looks at her watch and realizes that it's time for her favorite TV show. She turns around and swims back to shore. If you drown, can Anna be held criminally liable?

Yes because she started rescuing you and then abandoned you.

Four Kinds of Malice:

a. Intent to kill - The defendant acted with the desire that the victim end up dead. - Intent need not be premeditated; it can be formed in the moment or the second before the killing. b. Intent to inflict serious bodily harm - The defendant intended to hurt the victim badly, and the victim died. c. Someone who acts with abandoned or malignant heart or a depraved heart - The defendant acted with a cavalier disregard for human life and a death resulted. - Defendant must realize that his conduct is really risky but need not have any intent regarding the outcome of his actions. He doesn't have to have more than an intent there to be guilty of an abandoned or malignant heart or depraved heart murder. Example 54: Recall the example about dropping bowling balls off an overpass. If u hit someone with balls, chances are ur guilty of abandoned or malignant heart murder. § split among jx about how much depravity you actually have to have to be guilty of an abandoned or malignant heart or depraved heart murder - Majority of states and MPC—impose liability only when defendant actually realizes that there is a danger § Minority position—a reasonable person would have recognized the danger classic example of abandoned or depraved heart murder: Russian roulette d. Felony murder - The death occurred during the commission or attempted commission of a dangerous felony § The "BARRK" felonies—burglary, arson, robbery, rape, and kidnapping. These give rise to felony murder. Generally not felonies that themselves involve causing death of victim. E.g., aggravated assault that resulted in a death is not felony murder; it is a different type of common law murder.

If the principal commits crimes other than those for which the accomplice has provided encouragement or assistance...

an accomplice is liable for the other crimes if the crimes are the natural and probable consequences of the principal's conduct. In this case, the husband's death was a natural and probable consequence of burning down his house. Accordingly, both the woman and her sister would be held liable for either the arson or the husband's death.

Attempt

a• Attempt is a specific-intent crime ex: attempt to commit murder, attempt to commit burglary, etc. 1. Requirements 1) Specific intent to commit a particular criminal act; and 2) Has to have made a substantial step towards perpetrating the crime - have to have moved down the road quite a way towards actually committing a crime. Example 46: You work at a bank and plan to embezzle funds. Waking up and getting dressed does not constitute a substantial step. Example 47: Bringing special equipment to the scene of the crime or lying in wait will typically constitute substantial steps. o Attempt is a specific-intent crime even when the completed offense is only a general-intent crime. Example 48: Attempted murder is specific-intent crime, but murder is a general-intent malice crime. So pay attention to whether someone is charged with attempt or being charged with completed crime because defenses for the attempt may be quite different than for underlying crime. Editorial Note 1: Common-law murder is a malice crime, not a general-intent crime. 2. Defenses o Defenses for specific-intent crimes can be used as a defense to attempt Example 49: Aaron is standing on an overpass. For a joke, he throws bowling balls onto the highway below. If a bowling ball hits and kills someone, Aaron will be guilty of murder (because he acted with a maligned and depraved heart). But if the ball doesn't hit anyone, Aaron cannot be charged with attempted murder because he lacked the specific intent required to commit murder (i.e., he didn't intend to kill). He wasn't throwing bowling balls to kill people but just doing it because hes a bad person. But note he could be convicted of something like reckless endangerment - not like hed be completely free if he keeps tossing balls and nobody gets hit, but he cant be charged with attempted murder if balls don't hit anyone even if he could be charged with murder if they did hit someone. o Certain defenses like voluntary intoxication and unreasonable mistakes of fact are available for prosecutions of attempted crimes even if they wouldn't be available had the crime been completed. 3. Merger o Attempt merges into a completed offense. o You cannot be convicted of both attempted murder and murder of the same person in the same episode. o You can be convicted of both conspiracy to commit murder and murder. So attempt and conspiracy are treated differently with regard to merger.

States can only punish crimes

having some connection to the state. For example: o A crime that occurs whole or in part inside the state o Conduct outside the state that involved an attempt to commit a crime inside the state. For example - if someone tries t ocommit fraud and attempts fraud outside of ny but is trying to defraud someone in ny, that's something ny has jx over. o A conspiracy to commit a crime if an overt act occurred within the state.

2. 2. Knowingly or willfully - MPC

little less stringent - Requires that the defendant be aware that his conduct is of the nature required to commit the crime and that the result is practically certain to occur based on his conduct. But he doenst have to affirmatively want that result. Example 15: Patricia is angry at Vic but can't stand the sight of blood. So she decides to kill Vic by putting arsenic in his coffee Thermos. She knows Vic shares his coffee with his co-worker, Virgil, but she does not care what happens to Virgil. Both men drink the coffee and die. With respect to Vic, Patricia has acted purposely (she put arsenic in coffee with intent he die), but with respect to Virgil, she has acted knowingly, since she didn't have the intent that Virgil die, although she knew that the result was practically certain to occur.

knowingly or willfully - second MPC state of mind

little less stringent - Requires that the defendant be aware that his conduct is of the nature required to commit the crime and that the result is practically certain to occur based on his conduct. But he doenst have to affirmatively want that result. Example 15: Patricia is angry at Vic but can't stand the sight of blood. So she decides to kill Vic by putting arsenic in his coffee Thermos. She knows Vic shares his coffee with his co-worker, Virgil, but she does not care what happens to Virgil. Both men drink the coffee and die. With respect to Vic, Patricia has acted purposely (she put arsenic in coffee with intent he die), but with respect to Virgil, she has acted knowingly, since she didn't have the intent that Virgil die, although she knew that the result was practically certain to occur.

Embezzlement: A Variation of Larceny

o A defendant starts out having the victim's consent to have the property but commits embezzlement by converting the property to his own use after he gets it with the victims consent. For larceny its taking thing without victims consent. Here defendant has taken property with victims consent but then does something to deprive victim of the property.

Scope of a Conspiracy

o At common law, each co-conspirator can be convicted both of: 1) Conspiracy; and 2) All substantive crimes committed by any other conspirator acting in furtherance of the conspiracy. Not just crime u agreed to commit, but all other crimes committed in furtherance of the crime

Merger

o Attempt merges into a completed offense. o You cannot be convicted of both attempted murder and murder of the same person in the same episode. o You can be convicted of both conspiracy to commit murder and murder. So attempt and conspiracy are treated differently with regard to merger.

Inchoate and Completed Offenses

o Attempt—A defendant who actually completes a crime cannot also be convicted of attempting that crime. As long as just one person. Example 19: If you attempt to rob someone and you succeed in robbing them, you can only be convicted of robbery, not also attempted robbery. Example 20: If you try to rob Person A, but actually rob Person B, you can be convicted of attempted robbery of Person A and actual robbery of Person B. o Solicitation—Merges into the completed offense. Example 21: If the defendant solicits another person to commit a murder, and the other person commits the murder, the defendant is liable for the murder, but not for solicitation because it merges into the murder. o Conspiracy and a completed substantive offense do not merge! § A defendant can be convicted of both conspiracy to commit a crime and committing the crime itself.

Burglary - Elements

o Breaking can involve pushing open or smashing a door or window, or obtaining entry by fraud. Basically about how you get in. Note 11: Someone who comes inside with the owner's consent and then steals something is guilty of larceny (and possibly robbery if he uses violence or threat of violence) but not burglary because there was no "breaking." No coming in by force. o Entering involves breaking the plane of the dwelling. Example 69: Sticking your hand through a window constitutes "entering." o A "dwelling" at common law is a structure regularly lived in § Modern statutes can also include a commercial building o Of another: Cannot burglarize yourself o With the intent to commit a felony once inside § The usual felony is larceny, but it could be another felony, such as battery. Example 70: Breaking into another's house because of curiosity is not burglary because there is no intent to commit a felony once inside.

General Intent - third state of mind under common law

o Catch-all category o The intent to perform an act, and the act is unlawful o The defendant does not need to know that the act is unlawful; it is sufficient to intend to perform the act that the law condemns o Generally, acts done knowingly, recklessly, or negligently under the Model Penal Code (MPC) are general-intent crimes. § Examples include: battery, kidnapping, rape, and false imprisonment. Exam Tip 3: General intent crimes most likely to be tested on the MBE include manslaughter and battery.

Arson - Definition

o Common law arson requires: § Malicious § Burning § Of another person's § Dwelling o Malice: Intent to act in a way that will cause burning, or is substantially likely to do so o Burning: § Common law—there had to be burning (fire) as opposed to an explosion or smoke damage. It also required damage to the structure, not just the contents inside. § Modern statutes—it is arson even if there is no damage to the structure of the building or if the fire was caused by an explosion. o Another Person: § Common law— at common law had to be burning house or dwelling of another person, you could not torch your own house. § Modern statutes—burning your own home is arson. Because of insurance fraud so don't want people torching their own property and getting insurance proceeds. o Dwelling: § Common law—had to be a dwelling, not another commercial structure § Modern statutes—burning down a commercial building is arson.

Burglary - Definitions

o Common law burglary requires: § Breaking and § Entering § The dwelling § Of another § At night § With the specific intent to commit a felony once inside o The common law elements have been relaxed, so that modern law burglary requires: § Breaking and § Entering § The property § Of another § With the specific intent to commit a felony inside

Robbery - Definition and Elements

o Common law robbery is a simple equation: § Robbery = larceny + assault o Robbery requires (4 first elements are basically larceny elements and then add on 2 additional elements): 1) Taking; 2) Another person's property; 3) Without his consent; 4) With intent to deprive him of it permanently; and 5) The taking occurs from the victim's person or in his presence 6) Either by violence or putting the victim in fear of imminent physical harm Violence or physical harm or intimidation has to occur either before the taking, simultaneously with the taking, or immediately following the taking in order to retain the property. Level of physical force doesn't have to be huge - slght physical force is enough.

Withdrawal from a Conspiracy

o Common law—it's impossible to withdraw from a conspiracy, because the crime is completed the moment the agreement is made. Example 44: You and Professor Kramer agree to commit a crime. The next day, she calls and says she no longer wants to participate in the crime. Professor Kramer can still be convicted of conspiracy because you both agreed to commit the crime and that was moment when crime occurred.. But if you go ahead and commit the crime without Professor Kramer, she will not be convicted of the actual crime because she has withdrawn. Shes guilty of conspiracy but not of substantive offenses. o Federal and MPC—a conspirator can withdraw prior to the commission of any overt act by communicating her intention to withdraw to all other conspirators or by informing law enforcement. But must be done before the overt act. § After an overt act—a conspirator can withdraw only by helping to thwart the success of a conspiracy. Example 45: In an MPC jurisdiction, if Professor Kramer withdraws from the conspiracy after an overt act—either by notifying you or law enforcement—she may not be held liable for conspiracy if the notification was timely enough to thwart the success of the conspiracy. o Even if a defendant cannot withdraw from the conspiracy (e.g., because a conspiracy had already been formed), the defendant can limit his liability for substantive crimes by informing the other conspirators of withdrawal or timely advising legal authorities.

Defenses to Attempt

o Defenses for specific-intent crimes can be used as a defense to attempt Example 49: Aaron is standing on an overpass. For a joke, he throws bowling balls onto the highway below. If a bowling ball hits and kills someone, Aaron will be guilty of murder (because he acted with a maligned and depraved heart). But if the ball doesn't hit anyone, Aaron cannot be charged with attempted murder because he lacked the specific intent required to commit murder (i.e., he didn't intend to kill). He wasn't throwing bowling balls to kill people but just doing it because hes a bad person. But note he could be convicted of something like reckless endangerment - not like hed be completely free if he keeps tossing balls and nobody gets hit, but he cant be charged with attempted murder if balls don't hit anyone even if he could be charged with murder if they did hit someone. o Certain defenses like voluntary intoxication and unreasonable mistakes of fact are available for prosecutions of attempted crimes even if they wouldn't be available had the crime been completed.

malice - second state of mind under common law

o I "AM" certain that there are only two malice crimes: arson and murder (everything other than first degree murder which is fiat crime) o Malice exists when the defendant acts in reckless disregard of a high degree of harm. The defendant realizes the risk and acts anyway. Example 11: Arson is the malicious burning of the dwelling of another. Lynn is at Paul's house for a Fourth of July cookout. Some of the fireworks seem to be duds, so Lynn tosses them onto the grill. Some explode and catch Paul's house on fire. Lynn is charged with arson. At trial, she argues that she didn't want to cause Paul's house to burn down. Can Lynn be convicted of arson? Answer is yes, because arson isn't a specific intent crime - it's enough that she acted with malice - malice means sufficient knowledge of a risk and going ahead anyways. Its enough lynn was reckless and engaged in conduct with really high risk of harm.

Mistakes of Fact

o Key starting point: Determine whether the crime is a specific intent crime (FIAT), a general intent crime, or a strict liability crime. Strict Liability: Mistake of fact is never a defense to strict liability crimes. § Must be a voluntary act, but the defendant's state of mind is irrelevant Example 33: You live in a jurisdiction where selling alcohol to a minor is a strict liability crime. If you work in a wine shop, checking IDs is not sufficient to avoid liability for selling alcohol to a minor. Even if u reasonably believe id was valid, as long as person who got liquor was underage, that's enough. So for strict liability mistakes of fact are never a defense. General Intent: Mistake of fact is a defense only if the mistake is reasonable and goes to the criminal intent. Specific intent: Mistakes of fact are a defense whether the mistake is reasonable or unreasonable. The only question is whether the defendant held the mistaken belief. Example 34: Roger is out of town on a business trip and he is very jetlagged. He picks up a rental car at the airport but he's not paying attention to what color it is. He goes to a restaurant and gives his keys to the valet. After his meeting, Roger gives his ticket to the valet and the valet goes to get his car. The valet returns, not with Roger's cheap rental car, but with a Porsche. Roger gets into the Porsche and drives off. Roger is charged with larceny (which requires taking away the property of another person with the intent to permanently deprive the other person of the property; a specific-intent crime in fiat crimes - t or theft portion of fiat) and with joyriding (which requires taking the automobile of another person without permission for a short period of time). Can Roger be convicted of larceny if the jury concludes that no reasonable person would confuse an economy car with a Porsche? Answer is no. because larceny is a specific intent crime - as long as roger thought it was his car doesn't matter whether he was reasonable or unreasonable in thinking that. Can Roger be convicted of joyriding, even if the jury believes that he thought the car was his? But he can be convicted of joyriding because so long as jury concludes that mistake was unreasonable - that no reasonable person would have confused the cheap economy car with the expensive car - it doesn't matter what roger was thinking.

NEGATING MENS REA - mistake of fact

o Key starting point: Determine whether the crime is a specific intent crime (FIAT), a general intent crime, or a strict liability crime. Strict Liability: Mistake of fact is never a defense to strict liability crimes. § Must be a voluntary act, but the defendant's state of mind is irrelevant Example 33: You live in a jurisdiction where selling alcohol to a minor is a strict liability crime. If you work in a wine shop, checking IDs is not sufficient to avoid liability for selling alcohol to a minor. Even if u reasonably believe id was valid, as long as person who got liquor was underage, that's enough. So for strict liability mistakes of fact are never a defense. General Intent: Mistake of fact is a defense only if the mistake is reasonable and goes to the criminal intent. Specific intent: Mistakes of fact are a defense whether the mistake is reasonable or unreasonable. The only question is whether the defendant held the mistaken belief. Example 34: Roger is out of town on a business trip and he is very jetlagged. He picks up a rental car at the airport but he's not paying attention to what color it is. He goes to a restaurant and gives his keys to the valet. After his meeting, Roger gives his ticket to the valet and the valet goes to get his car. The valet returns, not with Roger's cheap rental car, but with a Porsche. Roger gets into the Porsche and drives off. Roger is charged with larceny (which requires taking away the property of another person with the intent to permanently deprive the other person of the property; a specific-intent crime in fiat crimes - t or theft portion of fiat) and with joyriding (which requires taking the automobile of another person without permission for a short period of time). Can Roger be convicted of larceny if the jury concludes that no reasonable person would confuse an economy car with a Porsche? Answer is no. because larceny is a specific intent crime - as long as roger thought it was his car doesn't matter whether he was reasonable or unreasonable in thinking that. Can Roger be convicted of joyriding, even if the jury believes that he thought the car was his? But he can be convicted of joyriding because so long as jury concludes that mistake was unreasonable - that no reasonable person would have confused the cheap economy car with the expensive car - it doesn't matter what roger was thinking.

Larceny - Definition

o Larceny requires: § Taking; § Another person's property; § Without his consent (trespassory); and § With the intent to deprive him of it permanently. a. Property—tangible personal property (e.g., wristwatch, goods from a store) § Not intangible property (e.g., copyright), real property, or services Note 10: There are modern "theft of services" statutes that criminalize obtaining a service, e.g., a massage, and then not paying for it. That conduct does not fall under common law larceny because haven't stolen someones property but rather their services. b. Taking—involves any movement of property, however slight Example 62: Hank goes into an electronics store to shoplift an MP3 player. He lifts the player off of the counter and heads for the door. Even before he leaves the store, he has satisfied the "taking" element of larceny - it just involves moving property, however slightly. c. Trespassory taking (without consent) - 2 ways of taking something without consent - 1. Not asking them and 2. Tricking them. § Consent must be real for something not to be a larceny, not obtained by trick (discussed below). § as long as the prosecutor shows the taking was wrongful, the defendant bears the burden of proving that there was consent d. Intent—an intent to deprive the person of the property permanently § "Borrowing" property, even without the owner's consent, is not larceny, as long as you intend to give the property back. Might be crime but not common law larceny. § If the property is destroyed in your care when you borrowed it, you have not committed larceny as long as you intend to give the property back.

Larceny is a....

specific-intent crime. T category of fiat - theft crime. § As long as the defendant thinks it's his property—however unreasonably—he is not guilty of larceny. For specific intent crimes, mistake of fact whether reasonable or unreasonable can negate the intent.

Lesser-included Offenses - merger

o Lesser-included offense—an offense in which each of its elements appears in another offense, but the other offense has something additional. Exam Tip 5: Think of each element as a different geometric figure. If the elements of Offense # 1 are a circle and a square, And the elements of Offense # 2 are circle, square, and triangle, then Offense #1 is the lesser-included offense of Offense #2; Offense #2 is the greater-included offense of Offense #1. Remember: A defendant cannot be convicted of both Offense #1 and Offense #2. Can be tried for both but ultimately because of double jeopardy clause cant be convicted of both. Lesser included offense will be merged into the greater one. Example 18: Ralph tries to steal Miss Humble's pin by gently prying it free from her jacket. But she notices Ralph and resists. Ralph then pushes her over and runs off with the jacket. His conduct would satisfy the elements for both larceny and robbery, but since: Robbery = larceny + threat of force Larceny is a lesser-included offense of robbery. Robbery is a greater-included offense of larceny. Ralph can be convicted of robbery but not of larceny, because larceny will merge into the robbery Note 3: If there are two separate victims to a crime, the crimes against each victim do not merge together. Remember - lesser included and greater included offenses merge together and D can be convicted only of one.

specific intent crimes

o Memorize the four categories of crimes that are specific intent crimes under the common law. Note 1: Remember FIAT - helps remember specific intent crimes! 1) First degree murder: not most murders; on the MBE, the question will expressly state if a defendant is charged with first-degree murder - first degree murder is a specific intent crime. 2) Inchoate crimes: "CATS"—conspiracy, attempt, and solicitation 3) Assault with attempt to commit a battery 4) Theft offenses: e.g., larceny, embezzlement, forgery, burglary, and robbery - if taking someones possessions, those theft offenses are also part of fiat and those are the specific intent crimes. Exam Tip 2: The main reason to memorize the FIAT crimes is that there are some defenses—most notably voluntary intoxication and unreasonable mistake of fact—that are available only for specific-intent crimes.

Mistakes of Law

o Mistakes about what the law forbids and what it permits o in general rule is ignorance of the law is no excuse. Mistake of law generally doesn't excuse defendant. Example 30: You thought the speed limit was 80 mph because last time you drove on that highway there was a sign that said 80 mph. In reality the speed limit was 65 mph. The fact that you didn't realize the speed limit was 65 mph is not a defense.

The United States has the power to criminalize and to prosecute crimes that:

o Occur anywhere inside the boundaries of the US o Occur on ships and planes or high seas or high air; or o Are committed by U.S. nationals abroad - so can have jx over foreign crimes too if its authorized

Voluntary Manslaughter

o Occurs when a defendant intends to kill the victim, but his state of mind is less blameworthy than murder o Acted in the "heat of passion" or "under extreme emotional disturbance" o Test: Is the situation one in which most people would act without thinking and without time to cool off? Example 59: Rufus comes home and finds his wife, Amy, in bed with Luke. Rufus shoots Luke and Amy. This is acting in the heat of passion or heat of the moment. People are not expected to stop, think rationally, and calm down in this situation. Note 9: Hearing about your spouse's affair is not sufficient. Heat of moment is only really if its in the actual heat of the moment. Example 60: Frank was sexually abused as a young teenager. Five years later, he testifies at the highly-publicized trial of his abuser and was forced to describe the abuse he had suffered. Several days later, a group of young teenagers surrounded Frank and began taunting him, suggesting that he had "asked for it." Frank lashes out at the group and strangles one of them. Would Frank be entitled to a manslaughter instruction? Probably not under the heat of passion defense, but maybe under the emotional disturbance defense - acting under extreme emotional disturbance. Need more facts to determine whether it was reasonable to strangle somebody for a taunt. So voluntary manslaughter - defendant actually intends to cause death but is acting under circumstances where we don't think theyre sufficiently blameworthy to find them guilty of murder.

Voluntary intoxication

o Occurs when a person intentionally ingests the substance, knowing it is an intoxicant. If you decide to have a beer for example or decide to pop some pills - knowing theyre an intoxicant for recreitonal use - thars voluntary intoxication. o Voluntary intoxication is a defense only to specific-intent crimes (FIAT crimes), and only if it prevented the defendant from forming the mens rea. Lay aside cases where someone gets drunk in order to commit the crime - because if you get drunk in order to commit the crime to muster up the courage to do it, then its not a defense to say but I was intoxicated at the time of the crime. § Not a valid defense if the defendant got drunk in order to commit the crime

Involuntary intoxication

o Occurs when a person: § Doesn't realize that she received an intoxicating substance (e.g., "date rape" drugs); § Is coerced into ingesting a substance - someone holds u down and makes you drink; or § Has an unexpected or unanticipated reaction to prescription medication. o Involuntary intoxication - Can be a valid defense to general-intent crimes, specific-intent crimes, and malice crimes when it negates the mens rea necessary for the crime so note - involuntary intoxication can negate the existence of mens rea.

Statutory Rape

o Regulatory morals offense that involves consensual sexual intercourse with a person under the age of consent o Statutory rape is a strict liability offense § So long as the defendant knows that he is having sex, he cannot claim ignorance or mistake about the victim's age. No matter how reasonable the mistake might be.

Strict Liability - fourth state of mind under common law

o There is no state of mind requirement or mens rea; the defendant must merely have committed the act. Kinds of things u find are strict liability crimes fall into 2 big categories - statutory or regulatory offenses. 1) Statutory/Regulatory offenses Example 12: A statute requires that food items be labeled with the expiration date. If a company sells those food items without an expiration date, they've committed a crime under this statute. Even if the company did not intend to sell the food without an expiration date or even if it was an honest mistake, the company is liable under strict liability. 2) Morals offenses Example 13: Humbert is attracted to younger women. He knows the age of consent in his state is 16. He meets Lolita in a bar, where patrons must show ID to enter. Lolita tells Humbert that she is 18 and she suggests they go back to her apartment to have sex. Humbert agrees and they have sex. Later, Humbert is charged with statutory rape, because Lolita is under the age of consent. At trial, Humbert states that he saw her ID and really thought that Lolita was 18. Suppose the judge believes Humbert, can he still be convicted of statutory rape? Answer is yes. Because that's a strict liability offense - only question is did he intend to have sex, and was she under the age of consent? And if those 2 questions answered yes, it doesnt matter what his intent was.

Homicide - Causation

o There must be a causal relationship between the defendant's actions and what happened to the victim o Actual Causation - but for causation: Victim would not have died "but for" what the defendant did o Proximate Causation: Defendant's act is a foreseeable cause of the victim's death (death is the natural and probable result of the conduct) Example 52: A defendant shoots the victim, and the victim dies in the hospital because the doctor was negligent. That death is a foreseeable consequence of shooting someone. So, the defendant's act was both the actual and proximate cause of the victim's death. § Independent actions by a third person are generally not a foreseeable cause Example 53: Bernie, the defendant, commits securities fraud. One of his victims—who was defrauded of money—commits suicide as a result. Bernie is not the proximate cause of the victim's death, even though he may be viewed as the actual cause. But for bernie stealing money person wouldn't have killed himself but too much intervening action to say it's a homicide. Exam Tip 7: Consent is not a defense to any type of homicide. Assisted suicide is a homicide by the assister, except in jurisdictions that permit assisted suicide. 3 big kinds of homicide: first degree murder, common law murder, and manslaughter

Battery - Definition

o Unlawful o Application of force o To another person o That causes: § Bodily harm OR § An unlawful touching

Battery - Elements

o Unlawful: Consent is a complete defense to battery (e.g., boxing match). o Application of force: Need not be a great deal of force; the slightest touch can constitute force in some cases o Battery is a general-intent crime, so voluntary intoxication and unreasonable mistake of fact (if unreasonable in thinking someone said punch me) are not available defenses. Like example: playing poker and someone says hit me, he means add more cards not literally hit him. That would be unreasonable and because battery is a general intent crime, that's not a defense. o Use of force does not require actual physical contact between the defendant and the victim (e.g., throwing a rock that hits someone can be a battery)

Relationships of co-conspirators

§ Chain Conspiracy: Co-conspirators are engaged in an enterprise consisting of many steps; each participant is liable for the substantive crimes of his co-conspirators Example 41: A conspiracy to distribute drugs involves many people in a causal chain (manufacturer—distributer—dealer). The conspirators need not know each other, but they have all agreed to participate in the same conspiracy and if so, each can be held liable (1) for the conspiracy and (2) for the substantive offenses committed along the way. § Spoke-Hub Conspiracy: Involves many people dealing with a central hub; participants are not liable for the substantive crimes of their co-conspirators because each spoke in the wheel is treated as a separate agreement rather than one larger general agreement Example 42: A bank employee agrees to process fraudulent loan applications. This employee would serve as the "hub." Each individual seeking a fraudulent loan is a "spoke." But the spokes are not generally responsible for the actions of the other spokes, unless they have much broader agreement amongst themselves. Example 43: A pawn shop operates as a "fence" to sell stolen goods. Each robber is responsible for conspiracy to sell stolen property, but not for the goods from other robbers - those are different spokes attached to the hub.

Merger

• A defendant can be convicted of more than one crime arising out of the same act. Example: if diana runs over pedestrian while trying to escape after robbing a bank, she can be convicted of both reckless driving and bank robbery. • A defendant cannot be convicted of two crimes when the two crimes merge into one. • Two categories of merger: 1) Lesser included offenses; and 2) The merger of an inchoate and a completed offense

Duress

• Defendant claims he committed a crime only because he was threatened by a third party and reasonably believed that the only way to avoid death or injury to himself or others was to commit the crime. • In order for duress to be a defense, there must be a threat of death or serious bodily harm. Mere injury, particularly injury to property, is not sufficient to give you a duress defense. • Defense for all crimes other than intentional murder. Example 80: Don decides to rob a grocery store. On the way out of the store, he leaps into a passing car driven by Fred, puts a gun to Fred's head, and orders Fred to drive him out of town. "Faster, faster," Don yells. Fred knows that the speed limit is 65, but he cranks the engine up to 90 miles an hour. As a result, he is unable to stop before running over Glenda. Fred is charged with felony murder with respect to Glenda's death. Is Fred guilty? No, because hes under duress. He killed her while he was speeding - didn't commit intentional murder. And only driving 90 mph so cant even be charged with that - he has duress defense for that because the only reason he was doing that is because he feared he would otherwise be seriously injured.

Merger

• A defendant can be convicted of more than one crime arising out of the same act. Example: if diana runs over pedestrian while trying to escape after robbing a bank, she can be convicted of both reckless driving and bank robbery. • A defendant cannot be convicted of two crimes when the two crimes merge into one. • Two categories of merger: 1) Lesser included offenses; and 2) The merger of an inchoate and a completed offense 1. Lesser-included Offenses o Lesser-included offense—an offense in which each of its elements appears in another offense, but the other offense has something additional. Exam Tip 5: Think of each element as a different geometric figure. If the elements of Offense # 1 are a circle and a square, And the elements of Offense # 2 are circle, square, and triangle, then Offense #1 is the lesser-included offense of Offense #2; Offense #2 is the greater-included offense of Offense #1. Remember: A defendant cannot be convicted of both Offense #1 and Offense #2. Can be tried for both but ultimately because of double jeopardy clause cant be convicted of both. Lesser included offense will be merged into the greater one. Example 18: Ralph tries to steal Miss Humble's pin by gently prying it free from her jacket. But she notices Ralph and resists. Ralph then pushes her over and runs off with the jacket. His conduct would satisfy the elements for both larceny and robbery, but since: Robbery = larceny + threat of force Larceny is a lesser-included offense of robbery. Robbery is a greater-included offense of larceny. Ralph can be convicted of robbery but not of larceny, because larceny will merge into the robbery Note 3: If there are two separate victims to a crime, the crimes against each victim do not merge together. Remember - lesser included and greater included offenses merge together and D can be convicted only of one. 2. Inchoate and Completed Offenses o Attempt—A defendant who actually completes a crime cannot also be convicted of attempting that crime. As long as just one person. Example 19: If you attempt to rob someone and you succeed in robbing them, you can only be convicted of robbery, not also attempted robbery. Example 20: If you try to rob Person A, but actually rob Person B, you can be convicted of attempted robbery of Person A and actual robbery of Person B. o Solicitation—Merges into the completed offense. Example 21: If the defendant solicits another person to commit a murder, and the other person commits the murder, the defendant is liable for the murder, but not for solicitation because it merges into the murder. o Conspiracy and a completed substantive offense do not merge! § A defendant can be convicted of both conspiracy to commit a crime and committing the crime itself.

accomplice liability - important point

• An accomplice can be criminally liable even if he or she cannot be a principal or even if the principal cannot be convicted as long as accomplice acted with intent of assisting in commission of the crime. Example 24: A statute makes it a crime for a public official to take bribes. Mayor Benedict wants to take a bribe, but wants to do it where no one will see. His friend, Claudio, owns a cabin in the woods. Mayor Benedict asks Claudio if he could use his cabin as the location to exchange bribery money. Claudio agrees and offers to chain off the road. Claudio could be guilty of aiding and abetting the bribery as an accomplice. But, he could not be guilty as a principal because he is not a public official. Example 25: You assist a six-year-old child to commit murder. Your little nephew tells you that he really wants to kill his classmate Bobby. You give your nephew a gun and he shoots Bobby. Your nephew cannot be convicted of murder because he is under the age of seven, but you can be convicted of aiding and abetting as an accomplice. Example 26: If you help a diplomat commit a crime, the diplomat has diplomatic immunity, so the diplomat might not be held liable as a principal, but you can still be held liable as an accomplice.

Defense of Others

• An individual has the same right to defend other individuals against a criminal that she has to defend herself. Example 78: You and a friend are walking down the street. Someone approaches your friend and says, "Give me your money or I'm going to blow your head off." You are entitled to use deadly force to protect your friend, as if you were protecting yourself.

Intoxication

• Ask two questions: 1) Was the intoxication voluntary or involuntary? 2) Is the charged crime a specific-intent crime or a general-intent crime? o Specific-intent crimes (FIAT or MPC statute with "purposely"): Can use either voluntary or involuntary intoxication as a defense, if the defendant because of intoxication could not maintain the state of mind necessary for the offense o General-intent crimes: Can use only involuntary intoxication as a defense

Children

• At common law, children under the age of 7 were never capable of committing a crime. • Children ages 7 to 14 were rebuttably presumed to be incapable of committing crimes. • Children at least 14 years old could be charged as adults. Exam Tip 6: On the MBE, children are more likely to be the victims of crime than the perpetrators.

Attempt

• Attempt is a specific-intent crime ex: attempt to commit murder, attempt to commit burglary, etc. 1. Requirements 1) Specific intent to commit a particular criminal act; and 2) Has to have made a substantial step towards perpetrating the crime - have to have moved down the road quite a way towards actually committing a crime. Example 46: You work at a bank and plan to embezzle funds. Waking up and getting dressed does not constitute a substantial step. Example 47: Bringing special equipment to the scene of the crime or lying in wait will typically constitute substantial steps. o Attempt is a specific-intent crime even when the completed offense is only a general-intent crime. Example 48: Attempted murder is specific-intent crime, but murder is a general-intent malice crime. So pay attention to whether someone is charged with attempt or being charged with completed crime because defenses for the attempt may be quite different than for underlying crime. Editorial Note 1: Common-law murder is a malice crime, not a general-intent crime.

Bribery

• Common law: Corrupt payment of something of value for purposes of influencing an official in the discharge of his official duties • Modern law: Allows a bribery charge even if the person being bribed is not a public official • Offering a bribe and receiving a bribe are both felonies. • Recall that a person can be convicted of bribery even if the person could not themselves "be bribed." Example 75: If the defendant aids and abets the bribing of a public official by helping the official accept the bribe, the defendant can be guilty of bribery even though he could not himself or herself receive the bribe and did not offer the bribe.

bribery

• Common law: Corrupt payment of something of value for purposes of influencing an official in the discharge of his official duties • Modern law: Allows a bribery charge even if the person being bribed is not a public official • Offering a bribe and receiving a bribe are both felonies. • Recall that a person can be convicted of bribery even if the person could not themselves "be bribed." Example 75: If the defendant aids and abets the bribing of a public official by helping the official accept the bribe, the defendant can be guilty of bribery even though he could not himself or herself receive the bribe and did not offer the bribe.

Intoxication

• Covers alcohol, drugs, and medications • Can be involuntary intoxication or voluntary intoxication

Principals

• Defendants whose acts or omissions form the actus reus of the crime. • Can be more than one principal to a particular crime. For example, if gang assaults and kills someone, each of members who killed them can be principal to a crime. • Ask: Who committed the actus reus that gives rise to the offense? If answer is this person committed actus reus, then person is a principal.

Necessity

• Defense Available in response to natural forces; i.e., it is the lesser of two evils Example 81: After Hurricane Katrina flooded New Orleans, many people were trapped in the city. One such person is Harold, who is trapped in his house without potable water. He breaks into a neighbor's house looking for drinkable water. Harold could assert a necessity defense, because he could claim that without getting water from his neighbor's house (which was a burglary and a larceny), he might have died. Thus, breaking into the house was the lesser of two evils.

Necessity

• Defense available in response to natural forces; i.e., it is the lesser of two evils Example 81: After Hurricane Katrina flooded New Orleans, many people were trapped in the city. One such person is Harold, who is trapped in his house without portable water. He breaks into a neighbor's house looking for drinkable water. Harold could assert a necessity defense, because he could claim that without getting water from his neighbor's house (which was a burglary and a larceny), he might have died. Thus, breaking into the house was the lesser of two evils.

Insanity

• Four Different Tests: 1) M'Naghten: Defendant either did not know the nature of the act or did not know that the act was wrong because of a mental disease or defect. Example 35: A defendant did not realize that he was shooting at a human being; he thought he was shooting at a pumpkin because he was delusional. 2) Irresistible Impulse: Defendant has a mental disease or defect that prevents the defendant from controlling himself 3) Durham Rule: Defendant would not have committed the crime but for his having a mental disease or defect (rarely used because so defendant-friendly) 4) Model Penal Code: Due to a mental disease or defect, the defendant did not have the substantial capacity to appreciate the wrongfulness of his actions or to conform his conduct to the law. Note 6: All four tests require that the defendant have a mental disease or defect. Being a sociopath is not enough to constitute insanity. • Majority of jx —the defendants has the burden of proving insanity either by a preponderance of the evidence or clear and convincing evidence. o Some jurisdictions require the defendant to introduce evidence of insanity, and then the burden of persuasion shifts to the prosecution to prove sanity beyond a reasonable doubt.

NEGATING MENS REA - insanity

• Four Different Tests: 1) M'Naghten: Defendant either did not know the nature of the act or did not know that the act was wrong because of a mental disease or defect. Example 35: A defendant did not realize that he was shooting at a human being; he thought he was shooting at a pumpkin because he was delusional. 2) Irresistible Impulse: Defendant has a mental disease or defect that prevents the defendant from controlling himself 3) Durham Rule: Defendant would not have committed the crime but for his having a mental disease or defect (rarely used because so defendant-friendly) 4) Model Penal Code: Due to a mental disease or defect, the defendant did not have the substantial capacity to appreciate the wrongfulness of his actions or to conform his conduct to the law. Note 6: All four tests require that the defendant have a mental disease or defect. Being a sociopath is not enough to constitute insanity. • Majority of jx —the defendants has the burden of proving insanity either by a preponderance of the evidence or clear and convincing evidence. o Some jurisdictions require the defendant to introduce evidence of insanity, and then the burden of persuasion shifts to the prosecution to prove sanity beyond a reasonable doubt.

Vicarious Liability

• Holds a person or entity liable for an actus reus committed by someone else • A corporation can be liable for the actions of its high-level employees or the Board of Directors. Even tho actus reus was committed by some lower level official.

Aiders/Abettors and Conspiracy

• In addition to accomplice liability for the substantive crime itself, individuals who aid or abet a defendant to commit a crime may also be guilty of the separate crime of conspiracy if there was an agreement to commit the crime and an overt act was taken in furtherance of that agreement. Example 29: Oscar and Irving might also be guilty of conspiracy to commit robbery in addition to the substantive crime of robbery. That conspiracy happens even if they don't make it all the way to the bank - if tires blow out on way to bank and they don't get there till after bank has closed, they cant be guilty of robbing bank because no robbery occurred. Might be guilty of attempted robbery of the bank. And they'll be guilty of a conspiracy.

defenses - insanity

• Involves a defendant who, because of a mental disease or defect, is unable to conform his conduct to the law.

Mistake of Fact

• Is this a specific-intent crime or a general-intent crime? o General-intent crimes: Only reasonable mistakes of fact may be used as a defense. o Specific-intent crimes: All mistakes of fact are potential defenses, even unreasonable mistakes. Even if d's belief in the fact was unreasonable - As long as d honestly held the belief that's good enough.

defenses - mistake of fact

• Is this a specific-intent crime or a general-intent crime? o General-intent crimes: Only reasonable mistakes of fact may be used as a defense. o Specific-intent crimes: All mistakes of fact are potential defenses, even unreasonable mistakes. Even if d's belief in the fact was unreasonable - As long as d honestly held the belief that's good enough.

The Mental States of Accomplices

• Majority and MPC Approaches—the accomplice must act with the purpose of promoting or facilitating the commission of the offense; the accomplice must intend that her acts will assist or encourage the criminal conduct. • Minority Approach—the accomplice is liable if he intentionally or knowingly aids or causes another person to commit an offense. • Criminal Facilitation—under the majority rule, a person who is not guilty of the substantive crime (because he did not act with intent) may nevertheless be guilty of the lesser offense of criminal facilitation for simply assisting.

First-Degree Murder

• Specific-intent crime - f in FIAT. Exam Tip 8: The question will generally tell you if the case involves first-degree murder. Otherwise, assume the question involves common law murder. • first degree murder Typically defined as a deliberate and pre-meditated murder, or a killing that results during the commission of an inherently dangerous felony (i.e., felony murder is frequently classified as first-degree murder). If it doesn't say its first degree murder, its not. Instead its common law murder.

The Mental States of Accomplices

• Majority and MPC Approaches—the accomplice must act with the purpose of promoting or facilitating the commission of the offense; the accomplice must intend that her acts will assist or encourage the criminal conduct. • Minority Approach—the accomplice is liable if he intentionally or knowingly aids or causes another person to commit an offense. • Criminal Facilitation—under the majority rule, a person who is not guilty of the substantive crime (because he did not act with intent) may nevertheless be guilty of the lesser offense of criminal facilitation for simply assisting. Remember - at common law little kids were never liable. Pre-adolescents were presumed to be incapable. Principals are people whos actus reus is the actus reus of the crime - they have both the mens rea and the actus reus and can be several of them. Aidors and abettors are guilty of same crime as principal if they helped before or during the commission of the offense - by contrast, people who help after the offense are guilty of a separate offense like obstruction of justice, harboring a fugitive, money laundering, or the like.

Solicitation

• Occurs when an individual intentionally invites, requests, or commands another person to commit a crime - act of asking is crime of solicitation • If the person agrees, the crime is conspiracy instead, rather than solicitation. • If the person commits the offense, the solicitation charge will merge into the completed offense although conspiracy charge will still remain as a separate one. Example 50: Amy offers to pay Doug $1,000 to commit a murder. This is a solicitation. If Doug says yes, this is a conspiracy to commit murder. So theres just conspiracy and not solicitation. If Doug actually commits the murder Amy will be guilty of the murder and of the conspiracy to commit murder, but not of solicitation because it merges in. Example 51: Wallace goes into a bar and offers a guy $10,000 to kill her boss. That is solicitation. If the guy agrees, that is conspiracy to commit murder. If the guy actually kills the boss, Wallace is guilty of both conspiracy and murder.

Solicitation

• Occurs when an individual intentionally invites, requests, or commands another person to commit a crime- act of asking is crime of solicitation • If the person agrees, the crime is conspiracy instead, rather than solicitation. • If the person commits the offense, the solicitation charge will merge into the completed offense although conspiracy charge will still remain as a separate one. Example 50: Amy offers to pay Doug $1,000 to commit a murder. This is a solicitation. If Doug says yes, this is a conspiracy to commit murder. So theres just conspiracy and not solicitation. If Doug actually commits the murder Amy will be guilty of the murder and of the conspiracy to commit murder, but not of solicitation because it merges in. Example 51: Wallace goes into a bar and offers a guy $10,000 to kill her boss. That is solicitation. If the guy agrees, that is conspiracy to commit murder. If the guy actually kills the boss, Wallace is guilty of both conspiracy and murder. Modern conspiracy is agreement to commit crime and an overt act - common law required 2 to tango but no overt act. The Conspirator is liable for both conspiracy, which is a crime itself, and for any crimes committed in furtherance of the conspiracy and hard to back out or withdraw from a conspiracy. Attempt is a specific intent crime so keep the special defenses for specific intent crimes in mind - it involves taking a substantial step towards the commission of a crime. While attempt merges into the completed offense, conspiracy doesn't.

Mistake

• Often, a defendant will claim that some mistake—regarding either facts in the world or the state of the law—negates his mens rea and thus he cannot be convicted of a crime for which there are both actus reus and mens rea elements.

Accessories After the Fact

• People who assist the defendant after the crime has been committed (e.g., obstruction of justice or harboring a fugitive) Example 28: After the bank robbery, Irving asks his accountant for help with laundering the money from the robbery. If the accountant helps Irving, he is not guilty of aiding and abetting the bank robbery, but instead would be guilty of a separate crime as an accessory after the fact (e.g., a financial crime).

Kidnapping

• Requires: o Unlawful o confinement of another person o Against that person's will o Either by moving or hiding the victim Example 73: Olivia and Paul decide to rob a grocery store. Alex, the clerk, manages to trip the alarm and Olivia grabs Scott and puts him into the meat locker. A hostage situation ensues, and police eventually get Olivia to release Scott. Now Olivia and Paul are charged with kidnapping Alex and Scott. Are they guilty of kidnapping Scott? Yes. Because he was moved - they pushed him into the locker and he was confined unlawfully without his consent. Are they guilty of kidnapping Alex? If alex wasn't moved at all, they didn't commit kidnapping simply by not letting him leave the store because they never moved him and they never hid him.

Defense of Property

• Right to use only non-deadly force to protect property Example 79: If you see someone trying to steal your car, you cannot use deadly force to prevent the theft of property. Exam Tip 13: The bar examiners often use a scenario involving deadly force designed to protect property during the owner's absence; e.g., the use of booby traps or spring guns. This is not an appropriate use of force if booby trap involves deadly force. But can use deadly force if theres a connection between the property and a physical risk - someone trying to steal ur necklace off ur neck - u can use deadly force if theyre using deadly force as well. If they have gun to ur neck and theyre trying to steal ur necklace u can shoot them

Defense of Property

• Right to use only non-deadly force to protect property Example 79: If you see someone trying to steal your car, you cannot use deadly force to prevent the theft of property. Exam Tip 13: The bar examiners often use a scenario involving deadly force designed to protect property during the owner's absence; e.g., the use of booby traps or spring guns. This is not an appropriate use of force if booby trap involves deadly force. But can use deadly force if theres a connection between the property and a physical risk - someone trying to steal ur necklace off ur neck - u can use deadly force if theyre using deadly force as well. If they have gun to ur neck and theyre trying to steal ur necklace u can shoot them.

First-Degree Murder

• Specific-intent crime - F in FIAT. Exam Tip 8: The question will generally tell you if the case involves first-degree murder. Otherwise, assume the question involves common law murder. • First degree murder - Typically defined as a deliberate and pre-meditated murder, or a killing that results during the commission of an inherently dangerous felony (i.e., felony murder is frequently classified as first-degree murder). If it doesn't say its first degree murder, its not. Instead its common law murder.

Mens Rea—The Model Penal Code States of Mind

• The MPC expresses mens rea as: purpose, knowledge, recklessness, and negligence. The MPC also recognizes some strict liability crimes. • Hierarchy of mental states: 1) Purpose—highest level of culpability - comes to looking like specific intent 2) Knowledge - not that u intended to cause result that law condems but u knew what u did would cause result. 3) Recklessness - involves weighing risk 4) Negligence—lowest level of culpability • Look for the mens rea requirement in the statute through words like "knowingly" or "intent to" • If there is no mens rea language, assume the prosecutor must prove recklessness.

Jurisdiction

• The United States has the power to criminalize and to prosecute crimes that: o Occur anywhere inside the boundaries of the US o Occur on ships and planes or high seas or high air; or o Are committed by U.S. nationals abroad - so can have jx over foreign crimes too if its authorized • States can only punish crimes having some connection to the state. For example: o A crime that occurs whole or in part inside the state o Conduct outside the state that involved an attempt to commit a crime inside the state. For example - if someone tries t ocommit fraud and attempts fraud outside of ny but is trying to defraud someone in ny, that's something ny has jx over. o A conspiracy to commit a crime if an overt act occurred within the state. Example 1: Doug lives in Delaware and is visiting South Carolina for vacation. In South Carolina, he meets Ken from Kansas in a bar. Doug hires Ken to kill Doug's wife, Vicki. Ken goes to Delaware, kidnaps Vicki, and her body is later found in Pennsylvania (where she was killed). Can Ken be convicted of murder in Delaware? Obviously answer is yes bc part of crime occurred there - kidnapping leading to homicide. Suppose Ken leaves the bar in South Carolina and rents a car to drive to Delaware. Can Ken be prosecuted for conspiracy to commit murder in South Carolina even if no crime occurred there? Note no other crime - because kidnapping happens in pa, not south Carolina. Yes he can be punished for conspiracy - one of the overt acts - renting car used to drive to where hed kidnap victim occurred there - even tho simply renting car wouldn't be a crime, its overt act in furtherance of the conspiracy. Can Ken be prosecuted in Kansas? No. because none of events connected to crime occurred there. Fact he lives there isn't enough to give Kansas jx over him.

Accomplices

• Theory for holding people other than the principal responsible for the crime committed by the principal; same degree of responsibility as the principal - imp thing about accomplice liability is its not separate crime, but rather different theory about why this d is guilty of a particular crime - murder or arson or like so accomplice is charged with same crime, but just diff theory as to why person is guilty. • Accomplices are people who assist the principal either before or during the commission of a crime • For someone to be liable as accomplice, must act with the intent of assisting the principal to commit the crime; bystanders, even approving ones, are not accomplices Example 22: You go to a bar after the exam and a bar fight breaks out. If you applaud during the fight, you are not liable for assault as an accomplice because you didnt act with intent of assisting principal, you just clapped. • Liable as an accomplice for both the planned crime and any other foreseeable crimes that occur in the course of the criminal act

. Self-Defense

• Two Kinds of force: o Deadly force: Intended or likely to cause death or serious injury (e.g., shooting someone) o Non-deadly force: (e.g., locking a door; pushing someone) 1. Non-deadly force o A victim is entitled to use non-deadly force any time he reasonably fears a threat of imminent unlawful harm. Example 76: If someone is trying to kiss you and you don't want them to, you can swat their hand away, slap their face, or use another type of non-deadly force. 2. Deadly force o A victim is entitled to use deadly force only if he reasonably believes that deadly force will be used against him, or under the MPC, reasonably believes that the crime will result in serious bodily injury. Deadly force requires much bigger threat on part of the person against whom force is being used than nondeadly force does. o Retreat - question of whether obligation on part of D to retreat. § Majority Rule: Retreat is not required even when entitled to use deadly force § Minority Rule: Someone must retreat rather than using deadly force if safe to do so Example 77: Jan sees you coming toward her house with a weapon in your hand. In a minority jurisdiction, if she can get inside, lock the door, and call the police, she cannot go upstairs to get her rifle and blow your head off from a window. § Even in minority jurisdictions, once already inside the house, retreat is never required when the person employing deadly force is in his own home. Exam Tip 12: On the MBE, the question will tell you if you are to assume a duty to retreat. Otherwise, assume that you can use deadly force to resist deadly force or another serious crime.

Defenses - Self-Defense

• Two Kinds of force: o Deadly force: Intended or likely to cause death or serious injury (e.g., shooting someone) o Non-deadly force: (e.g., locking a door; pushing someone) 1. Non-deadly force o A victim is entitled to use non-deadly force any time he reasonably fears a threat of imminent unlawful harm. Example 76: If someone is trying to kiss you and you don't want them to, you can swat their hand away, slap their face, or use another type of non-deadly force. 2. Deadly force o A victim is entitled to use deadly force only if he reasonably believes that deadly force will be used against him, or under the MPC, reasonably believes that the crime will result in serious bodily injury. Deadly force requires much bigger threat on part of the person against whom force is being used than nondeadly force does. o Retreat - question of whether obligation on part of d to retreat. § Majority Rule: Retreat is not required even when entitled to use deadly force § Minority Rule: Someone must retreat rather than using deadly force if safe to do so Example 77: Jan sees you coming toward her house with a weapon in your hand. In a minority jurisdiction, if she can get inside, lock the door, and call the police, she cannot go upstairs to get her rifle and blow your head off from a window. § Even in minority jurisdictions, once already inside the house, retreat is never required when the person employing deadly force is in his own home. Exam Tip 12: On the MBE, the question will tell you if you are to assume a duty to retreat. Otherwise, assume that you can use deadly force to resist deadly force or another serious crime.

transferred intent doctrine

• When a defendant has the requisite mens rea for committing a crime against Victim A, but actually commits the crime against Victim B, the law transfers the intent from Victim A over to Victim B. Example 17: Ralph is a professional jewel thief. He is hired to steal a valuable diamond pin from Mrs. Rich at a charity ball. In preparation, Ralph studies her picture from old newspapers. However, Mrs. Rich has had a lot of work done on her face since the picture. So at the ball, Ralph approaches Miss Humble, who resembles the picture he studied, believing her to be Mrs. Rich, and steals jewelry from her instead. Can Ralph be charged with larceny—a specific-intent crime—against Miss Humble? Yes he intended to commit larceny and it transfers to ms humble from ms rich. Can he be charged with attempted larceny against Mrs. Rich? Yes, because he attempted to commit larceny against her even though he actually committed larceny against ms humble. Note 2: Transferred intent does not apply to attempted crimes, only to completed crimes.

Perjury

• Willful act of falsely promising to tell the truth, either verbally or in writing, about material matters • The person must know what they are saying is false, must intend to say something that is false, and the falsity must go to a material matter to the reason why the statement is being made. Example 74: Pauline is a called as a witness in a burglary case. She says that she was walking down the street and she saw the defendant break into the victim's house and then come out. It would not be a material falsehood for her to say that just before she saw the defendant, she had been at a friend's house not doing much of anything—when in reality, she was at the friend's house sleeping with her friend's husband. This is not a material falsehood, at least not vis-à-vis the burglary case. Because it doenst go in any way to the likelihood that she was where she said she was at the time she saw the burglary. • Subornation of Perjury: A person persuades someone else to commit perjury, such as paying someone to testify falsely. U have to know what ur saying is false, falsehood must be material, and u have to do it under oath otherwise its not perjury.

Perjury

• Willful act of falsely promising to tell the truth, either verbally or in writing, about material matters • The person must know what they are saying is false, must intend to say something that is false, and the falsity must go to a material matter to the reason why the statement is being made. Example 74: Pauline is a called as a witness in a burglary case. She says that she was walking down the street and she saw the defendant break into the victim's house and then come out. It would not be a material falsehood for her to say that just before she saw the defendant, she had been at a friend's house not doing much of anything—when in reality, she was at the friend's house sleeping with her friend's husband. This is not a material falsehood, at least not vis-à-vis the burglary case. Because it doesnt go in any way to the likelihood that she was where she said she was at the time she saw the burglary. • Subornation of Perjury: A person persuades someone else to commit perjury, such as paying someone to testify falsely. U have to know what ur saying is false, falsehood must be material, and u have to do it under oath otherwise its not perjury.


Kaugnay na mga set ng pag-aaral

Exam I Money and the Financial Systems: Ch4

View Set

NASBA ELCP comprehensive exam UWYO

View Set

ATI Postpartum Physiological Assessments and Nursing Care

View Set